You are on page 1of 143

NGND NGUYN TR HIP

B N THI TUYN SINH


VO LP 10 THPT V THPT CHUYN
Mn: TON
Nh xut bn...
1
BIN TP
NGND Nguyn Tr Hip
Ph Gim c S GDT
Ths Nguyn Ngc Lc
Trng Phng GDTrH S GDT
BIN SON
Nguyn Vit Ph
Chuyn vin Phng GDTrH S GDT
Ths L Phi Hng
Gio vin Trng THPT Chuyn H Tnh
Ths Nguyn Hng Cng
Ph hiu trng Trng THPT Phan nh Phng
Phm Quc Phong
Gio vin Trng THPT Hng Lnh
Hong B Dng
Gio vin Trng THPT Mai Knh
Nguyn nh Nhm
Gio vin Trng THPT Cm Xuyn
Bi Hi Bnh
Gio vin Trng THCS L Vn Thim
ng Hi Giang
Gio vin Trng THCS Th trn Cm Xuyn
Nguyn Huy Tin
Chuyn vin Phng GDT Hng Lnh
2
LI NI U
gp phn nh hng cho vic dy - hc cc trng nht
l vic n tp, rn luyn k nng cho hc sinh st vi thc tin gio dc
ca tnh nh nhm nng cao cht lng cc k thi tuyn sinh, S
GDT H Tnh pht hnh B ti liu n thi tuyn sinh vo lp 10
THPT v THPT chuyn gm 3 mn: Ton, Ng vn v Ting Anh.
- Mn Ng vn c vit theo hnh thc ti liu n tp.
V cu trc: H thng kin thc c bn ca nhng bi hc
trong chng trnh Ng vn lp 9 (ring phn mn Ting Vit, kin
thc, k nng ch yu c hc t lp 6,7,8). Cc vn bn vn hc, vn
bn nht dng, vn bn ngh lun c trnh by theo trnh t: tc gi,
tc phm (hoc on trch), bi tp. Cc thi tham kho (18 ) c
bin son theo hng: gm nhiu cu v km theo gi lm bi
(mc ch cc em lm quen v c k nng vi dng thi tuyn sinh
vo lp 10).
V ni dung kin thc, k nng: Ti liu c bin son theo
hng bm Chun kin thc, k nng ca B GDT, trong tp
trung vo nhng kin thc c bn, trng tm v k nng vn dng.
- Mn Ting Anh c vit theo hnh thc ti liu n tp, gm
hai phn: H thng kin thc c bn, trng tm trong chng trnh
THCS th hin qua cc dng bi tp c bn v mt s thi tham kho
(c p n).
- Mn Ton c vit theo hnh thc B n thi, gm hai
phn: mt phn n thi vo lp 10 THPT, mt phn n thi vo lp 10
THPT chuyn da trn cu trc thi ca S. Mi thi u c li
gii tm tt v km theo mt s li bnh.
B ti liu n thi ny do cc thy, c gio l lnh o, chuyn
vin phng Gio dc Trung hc - S GDT; ct cn chuyn mn cc
b mn ca S; cc thy, c gio l Gio vin gii tnh bin son.
3
Hy vng y l B ti liu n thi c cht lng, gp phn quan
trng nng cao cht lng dy - hc cc trng THCS v k thi
tuyn sinh vo lp 10 THPT, THPT chuyn nm hc 2011-2012 v
nhng nm tip theo.
Mc d c s u t ln v thi gian, tr tu ca i ng
nhng ngi bin son, song khng th trnh khi nhng hn ch, sai
st. Mong c s ng gp ca cc thy, c gio v cc em hc sinh
trong ton tnh B ti liu c hon chnh hn.
Chc cc thy, c gio v cc em hc sinh thu c kt qu cao
nht trong cc k thi sp ti!
Trng ban bin tp
Nh gio Nhn dn,
Ph Gim c S GDT H Tnh
Nguyn Tr Hip
4
A - PHN BI
I - N THI TUYN SINH LP 10 THPT
S 1
Cu 1: a) Cho bit a = 2 3 + v b = 2 3 . Tnh gi tr biu thc: P = a + b ab.
b) Gii h phng trnh:
3x + y = 5
x - 2y = - 3

'

.
Cu 2: Cho biu thc P =
1 1 x
:
x - x x 1 x - 2 x 1
_
+

+
,
(vi x > 0, x

1)
a) Rt gn biu thc P.
b) Tm cc gi tr ca x P >
1
2
.
Cu 3: Cho phng trnh: x
2
5x + m = 0 (m l tham s).
a) Gii phng trnh trn khi m = 6.
b) Tm m phng trnh trn c hai nghim x
1
, x
2
tha mn:
1 2
x x 3
.
Cu 4: Cho ng trn tm O ng knh AB. V dy cung CD vung gc
vi AB ti I (I nm gia A v O ). Ly im E trn cung nh BC ( E khc B
v C ), AE ct CD ti F. Chng minh:
a) BEFI l t gic ni tip ng trn.
b) AE.AF = AC
2
.
c) Khi E chy trn cung nh BC th tm ng trn ngoi tip CEF
lun thuc mt ng thng c nh.
Cu 5: Cho hai s dng a, b tha mn: a + b
2 2
. Tm gi tr nh nht
ca biu thc: P =
1 1
a b
+
.
S 2
Cu 1: a) Rt gn biu thc:
1 1
3 7 3 7

+
.
b) Gii phng trnh: x
2
7x + 3 = 0.
Cu 2: a) Tm ta giao im ca ng thng d: y = - x + 2 v Parabol
(P): y = x
2
.
5
b) Cho h phng trnh:
4x + ay = b
x - by = a

'

.
Tm a v b h cho c nghim duy nht ( x;y ) = ( 2; - 1).
Cu 3: Mt xe la cn vn chuyn mt lng hng. Ngi li xe tnh rng
nu xp mi toa 15 tn hng th cn tha li 5 tn, cn nu xp mi toa 16 tn
th c th ch thm 3 tn na. Hi xe la c my toa v phi ch bao nhiu
tn hng.
Cu 4: T mt im A nm ngoi ng trn (O;R) ta v hai tip tuyn AB,
AC vi ng trn (B, C l tip im). Trn cung nh BC ly mt im M,
v MI AB, MK AC (I

AB,K

AC)
a) Chng minh: AIMK l t gic ni tip ng trn.
b) V MP BC (P

BC). Chng minh:



MPK MBC
.
c) Xc nh v tr ca im M trn cung nh BC tch MI.MK.MP
t gi tr ln nht.
Cu 5: Gii phng trnh:
y - 2010 1 x - 2009 1 z - 2011 1 3
x - 2009 y - 2010 z - 2011 4

+ +
S 3
Cu 1: Gii phng trnh v h phng trnh sau:
a) x
4
+ 3x
2
4 = 0
b)
2x + y = 1
3x + 4y = -1

'

Cu 2: Rt gn cc biu thc:
a) A =
3 6 2 8
1 2 1 2
+

+
b) B =
1 1 x + 2 x
.
x 4
x + 4 x 4 x
_

+
,
( vi x > 0, x

4 ).
Cu 3: a) V th cc hm s y = - x
2
v y = x 2 trn cng mt h trc ta
.
b) Tm ta giao im ca cc th v trn bng php tnh.
Cu 4: Cho tam gic ABC c ba gc nhn ni tip trong ng trn (O;R).
Cc ng cao BE v CF ct nhau ti H.
a) Chng minh: AEHF v BCEF l cc t gic ni tip ng trn.
6
b) Gi M v N th t l giao im th hai ca ng trn (O;R) vi BE
v CF. Chng minh: MN // EF.
c) Chng minh rng OA EF.
Cu 5: Tm gi tr nh nht ca biu thc:
P =
2
x - x y + x + y - y + 1
S 4
Cu 1: a) Trc cn thc mu ca cc biu thc sau:
4
3
;
5
5 1
.
b) Trong h trc ta Oxy, bit th hm s y = ax
2
i qua im M
(- 2;
1
4
). Tm h s a.
Cu 2: Gii phng trnh v h phng trnh sau:
a)
2x + 1 = 7 - x
b)
2x + 3y = 2
1
x - y =
6

'

Cu 3: Cho phng trnh n x: x


2
2mx + 4 = 0 (1)
a) Gii phng trnh cho khi m = 3.
b) Tm gi tr ca m phng trnh (1) c hai nghim x
1
, x
2
tha mn:
( x
1
+ 1 )
2
+ ( x
2
+ 1 )
2
= 2.
Cu 4: Cho hnh vung ABCD c hai ng cho ct nhau ti E. Ly I thuc
cnh AB, M thuc cnh BC sao cho:

0
IEM 90
(I v M khng trng vi cc
nh ca hnh vung ).
a) Chng minh rng BIEM l t gic ni tip ng trn.
b) Tnh s o ca gc

IME
c) Gi N l giao im ca tia AM v tia DC; K l giao im ca BN v
tia EM. Chng minh CK BN.
Cu 5: Cho a, b, c l di 3 cnh ca mt tam gic. Chng minh:
ab + bc + ca a
2
+ b
2
+ c
2
< 2(ab + bc + ca ).
7
S 5
Cu 1: a) Thc hin php tnh:
3 2
. 6
2 3
_



,
b) Trong h trc ta Oxy, bit ng thng y = ax + b i qua im
A( 2; 3 ) v im B(-2;1) Tm cc h s a v b.
Cu 2: Gii cc phng trnh sau:
a) x
2
3x + 1 = 0
b)
2
x - 2 4
+ =
x - 1 x + 1 x - 1
Cu 3: Hai t khi hnh cng mt lc trn qung ng t A n B di
120 km. Mi gi t th nht chy nhanh hn t th hai l 10 km nn n
B trc t th hai l 0,4 gi. Tnh vn tc ca mi t.
Cu 4: Cho ng trn (O;R); AB v CD l hai ng knh khc nhau ca
ng trn. Tip tuyn ti B ca ng trn (O;R) ct cc ng thng AC,
AD th t ti E v F.
a) Chng minh t gic ACBD l hnh ch nht.
b) Chng minh ACD
~
CBE
c) Chng minh t gic CDFE ni tip c ng trn.
d) Gi S, S
1
, S
2
th t l din tch ca AEF, BCE v BDF. Chng
minh:
1 2
S S S + .
Cu 5: Gii phng trnh:
( )
3 2
10 x + 1 = 3 x + 2
S 6
Cu 1: Rt gn cc biu thc sau:
a) A =
3 3 3 3
2 . 2
3 1 3 1
_ _
+
+


+
, ,
b) B =
( )
b a
- . a b - b a
a - ab ab - b
_


,
( vi a > 0, b > 0, a

b)
Cu 2: a) Gii h phng trnh:
( )
( )
x - y = - 1 1
2 3
+ = 2 2
x y

'

8
b) Gi x
1
, x
2
l hai nghim ca phng trnh: x
2
x 3 = 0. Tnh gi
tr biu thc: P = x
1
2
+ x
2
2
.
Cu 3:
a) Bit ng thng y = ax + b i qua im M ( 2;
1
2
) v song song vi
ng thng 2x + y = 3. Tm cc h s a v b.
b) Tnh cc kch thc ca mt hnh ch nht c din tch bng 40 cm
2
,
bit rng nu tng mi kch thc thm 3 cm th din tch tng thm 48 cm
2
.
Cu 4: Cho tam gic ABC vung ti A, M l mt im thuc cnh AC (M
khc A v C ). ng trn ng knh MC ct BC ti N v ct tia BM ti I.
Chng minh rng:
a) ABNM v ABCI l cc t gic ni tip ng trn.
b) NM l tia phn gic ca gc

ANI
.
c) BM.BI + CM.CA = AB
2
+ AC
2
.
Cu 5: Cho biu thc A = 2x - 2 xy + y - 2 x + 3. Hi A c gi tr nh
nht hay khng? V sao?
S 7
Cu 1: a) Tm iu kin ca x biu thc sau c ngha: A = x - 1 + 3 - x
b) Tnh:
1 1
3 5 5 1

+
Cu 2: Gii phng trnh v bt phng trnh sau:
a) ( x 3 )
2
= 4
b)
x - 1 1
<
2x + 1 2
Cu 3: Cho phng trnh n x: x
2
2mx - 1 = 0 (1)
a) Chng minh rng phng trnh cho lun c hai nghim phn bit
x
1
v x
2
.
b) Tm cc gi tr ca m : x
1
2
+ x
2
2
x
1
x
2
= 7.
Cu 4: Cho ng trn (O;R) c ng knh AB. V dy cung CD vung
gc vi AB (CD khng i qua tm O). Trn tia i ca tia BA ly im S; SC
ct (O; R) ti im th hai l M.
a) Chng minh SMA ng dng vi SBC.
b) Gi H l giao im ca MA v BC; K l giao im ca MD v AB.
Chng minh BMHK l t gic ni tip v HK // CD.
9
c) Chng minh: OK.OS = R
2
.
Cu 5: Gii h phng trnh:
3
3
x + 1 = 2y
y + 1 = 2x

'

.
S 8
Cu 1: a) Gii h phng trnh:
2x + y = 5
x - 3y = - 1

'

b) Gi x
1
,x
2
l hai nghim ca phng trnh:3x
2
x 2 = 0. Tnh gi
tr biu thc: P =
1 2
1 1
+
x x
.
Cu 2: Cho biu thc A =
a a a 1
:
a - 1
a 1 a - a
_
+

,
vi a > 0, a

1
a) Rt gn biu thc A.
b) Tm cc gi tr ca a A < 0.
Cu 3: Cho phng trnh n x: x
2
x + 1 + m = 0 (1)
a) Gii phng trnh cho vi m = 0.
b) Tm cc gi tr ca m phng trnh (1) c hai nghim x
1
, x
2
tha
mn: x
1
x
2
.( x
1
x
2
2 ) = 3( x
1
+ x
2
).
Cu 4: Cho na ng trn tm O ng knh AB = 2R v tia tip tuyn Ax
cng pha vi na ng trn i vi AB. T im M trn Ax k tip tuyn
th hai MC vi na ng trn (C l tip im). AC ct OM ti E; MB ct
na ng trn (O) ti D (D khc B).
a) Chng minh: AMCO v AMDE l cc t gic ni tip ng trn.
b) Chng minh

ADE ACO
.
c) V CH vung gc vi AB (H

AB). Chng minh rng MB i qua


trung im ca CH.
Cu 5: Cho cc s a, b, c [ ]
0 ; 1
. Chng minh rng: a + b
2
+ c
3
ab bc
ca 1.
S 9

Cu 1: a) Cho hm s y =
( )
3 2
x + 1. Tnh gi tr ca hm s khi x =
3 2 + .
b) Tm m ng thng y = 2x 1 v ng thng y = 3x + m ct
nhau ti mt im nm trn trc honh.
10
Cu 2: a) Rt gn biu thc: A =
3 x 6 x x - 9
:
x - 4
x 2 x 3
_
+
+



,
vi
x 0, x 4, x 9
.
b) Gii phng trnh:
( ) ( )
2
x - 3x + 5 1
x + 2 x - 3 x - 3

Cu 3: Cho h phng trnh:


3x - y = 2m - 1
x + 2y = 3m + 2

'

(1)
a) Gii h phng trnh cho khi m = 1.
b) Tm m h (1) c nghim (x; y) tha mn: x
2
+ y
2
= 10.
Cu 4: Cho na ng trn tm O ng knh AB. Ly im M thuc on
thng OA, im N thuc na ng trn (O). T A v B v cc tip tuyn Ax
v By. ng thng qua N v vung gc vi NM ct Ax, By th t ti C v
D.
a) Chng minh ACNM v BDNM l cc t gic ni tip ng trn.
b) Chng minh ANB ng dng vi CMD.
c) Gi I l giao im ca AN v CM, K l giao im ca BN v DM.
Chng minh IK //AB.
Cu 5: Chng minh rng:
( ) ( )
a + b 1
2
a 3a + b b 3b + a

+
vi a, b l cc s
dng.
S 10
Cu 1: Rt gn cc biu thc:
a) A =
( )
2
3 8 50 2 1
b) B =
2
2
2 x - 2x + 1
.
x - 1 4x
, vi 0 < x < 1
Cu 2:Gii h phng trnh v phng trnh sau:
a)
( )
2 x - 1 y = 3
x - 3y = - 8
+

'

.
b)
x + 3 x 4 0
11
Cu 3: Mt x nghip sn xut c 120 sn phm loi I v 120 sn phm
loi II trong thi gian 7 gi. Mi gi sn xut c s sn phm loi I t hn
s sn phm loi II l 10 sn phm. Hi mi gi x nghip sn xut c bao
nhiu sn phm mi loi.
Cu 4: Cho hai ng trn (O) v
(O )
ct nhau ti A v B. V AC, AD th
t l ng knh ca hai ng trn (O) v
(O )
.
a) Chng minh ba im C, B, D thng hng.
b) ng thng AC ct ng trn
(O )
ti E; ng thng AD ct
ng trn (O) ti F (E, F khc A). Chng minh 4 im C, D, E, F cng nm
trn mt ng trn.
c) Mt ng thng d thay i lun i qua A ct (O) v
(O )
th t ti M
v N. Xc nh v tr ca d CM + DN t gi tr ln nht.
Cu 5: Cho hai s x, y tha mn ng thc:

( ) ( )
2 2
x + x 2011 y + y 2011 2011 + +
Tnh: x + y
S 11
Cu 1: 1) Rt gn biu thc:

2
1 - a a 1 - a
A a
1 - a
1 - a
_ _
+


, ,
vi a 0 v a 1.
2) Gii phng trnh: 2x
2
- 5x + 3 = 0
Cu 2: 1) Vi gi tr no ca k, hm s y = (3 - k) x + 2 nghch bin trn R.
2) Gii h phng trnh:

4x + y = 5
3x - 2y = - 12

'

Cu 3: Cho phng trnh x


2
- 6x + m = 0.
1) Vi gi tr no ca m th phng trnh c 2 nghim tri du.
2) Tm m phng trnh c 2 nghim x
1
, x
2
tho mn iu kin x
1
- x
2
= 4.
Cu 4: Cho ng trn (O; R), ng knh AB. Dy BC = R. T B k tip
tuyn Bx vi ng trn. Tia AC ct Bx ti M. Gi E l trung im
ca AC.
1) Chng minh t gic OBME ni tip ng trn.
2) Gi I l giao im ca BE vi OM. Chng minh: IB.IE = IM.IO.
12
Cu 5: Cho x > 0, y > 0 v x + y 6. Tm gi tr nh nht ca biu thc :
P = 3x + 2y +
6 8
+
x y
.
S 12
Cu 1: Tnh gn biu thc:
1) A = 20 - 45 + 3 18 + 72 .
2) B =
a + a a - a
1 + 1 +
a + 1 1- a
_ _


, ,
vi a 0, a 1.
Cu 2: 1) Cho hm s y = ax
2
, bit th hm s i qua im A (- 2 ; -12).
Tm a.
2) Cho phng trnh: x
2
+ 2 (m + 1)x + m
2
= 0. (1)
a. Gii phng trnh vi m = 5
b. Tm m phng trnh (1) c 2 nghim phn bit, trong c 1
nghim bng - 2.
Cu 3: Mt tha rung hnh ch nht, nu tng chiu di thm 2m, chiu
rng thm 3m th din tch tng thm 100m
2
. Nu gim c chiu di v chiu
rng i 2m th din tch gim i 68m
2
. Tnh din tch tha rung .
Cu 4: Cho tam gic ABC vung A. Trn cnh AC ly 1 im M, dng
ng trn tm (O) c ng knh MC. ng thng BM ct ng trn tm
(O) ti D, ng thng AD ct ng trn tm (O) ti S.
1) Chng minh t gic ABCD l t gic ni tip v CA l tia phn gic
ca gc

BCS
.
2) Gi E l giao im ca BC vi ng trn (O). Chng minh cc
ng thng BA, EM, CD ng quy.
3) Chng minh M l tm ng trn ni tip tam gic ADE.
Cu 5: Gii phng trnh.

2 2
x - 3x + 2 + x + 3 = x - 2 + x + 2x - 3
S 13
Cu 1: Cho biu thc: P =
a a - 1 a a + 1 a +2
- :
a - 2
a - a a + a
_


,
vi a > 0, a 1, a
2.
1) Rt gn P.
13
2) Tm gi tr nguyn ca a P c gi tr nguyn.
Cu 2: 1) Cho ng thng d c phng trnh: ax + (2a - 1) y + 3 = 0
Tm a ng thng d i qua im M (1, -1). Khi , hy tm h s gc ca
ng thng d.
2) Cho phng trnh bc 2: (m - 1)x
2
- 2mx + m + 1 = 0.
a) Tm m, bit phng trnh c nghim x = 0.
b) Xc nh gi tr ca m phng trnh c tch 2 nghim bng 5, t
hy tnh tng 2 nghim ca phng trnh.
Cu 3: Gii h phng trnh:

4x + 7y = 18

3x - y = 1

'

Cu 4: Cho ABC cn ti A, I l tm ng trn ni tip, K l tm ng


trn bng tip gc A, O l trung im ca IK.
1) Chng minh 4 im B, I, C, K cng thuc mt ng trn tm O.
2) Chng minh AC l tip tuyn ca ng trn tm (O).
3) Tnh bn knh ca ng trn (O), bit AB = AC = 20cm, BC = 24cm.
Cu 5: Gii phng trnh: x
2
+ x + 2010 = 2010.
S 14
Cu 1: Cho biu thc
P =
x + 1 2 x 2 + 5 x
+ +
4 - x
x - 2 x + 2
vi x 0, x 4.
1) Rt gn P.
2) Tm x P = 2.
Cu 2: Trong mt phng, vi h ta Oxy, cho ng thng d c phng
trnh:
y m 1 x n ( ) +
.
1) Vi gi tr no ca m v n th d song song vi trc Ox.
2) Xc nh phng trnh ca d, bit d i qua im A(1; - 1) v c h
s gc bng -3.
Cu 3: Cho phng trnh: x
2
- 2 (m - 1)x - m - 3 = 0 (1)
1) Gii phng trnh vi m = -3
2) Tm m phng trnh (1) c 2 nghim tho mn h thc
2 2
1 2
x + x
= 10.
3) Tm h thc lin h gia cc nghim khng ph thuc gi tr ca m.
Cu 4: Cho tam gic ABC vung A (AB > AC), ng cao AH. Trn na mt
phng b BC cha im A, v na ng trn ng knh BH ct AB ti
E, na ng trn ng knh HC ct AC ti F. Chng minh:
14
1) T gic AFHE l hnh ch nht.
2) T gic BEFC l t gic ni tip ng trn.
3) EF l tip tuyn chung ca 2 na ng trn ng knh BH v HC.
Cu 5: Cc s thc x, a, b, c thay i, tha mn h:

2 2 2 2
x + a + b + c = 7 (1)
x + a + b + c = 13 (2)

'

Tm gi tr ln nht v gi tr nh nht ca x.
S 15

Cu 1: Cho M =
x 1 1 2
- : +
x - 1
x - 1 x - x x 1
_
_



+
,
,
vi
x 0, x 1 >
.
a) Rt gn M.
b) Tm x sao cho M > 0.
Cu 2: Cho phng trnh x
2
- 2mx - 1 = 0 (m l tham s)
a) Chng minh rng phng trnh lun c hai nghim phn bit.
b) Gi x
1
, x
2
l hai nghim ca phng trnh trn.
Tm m
2 2
1 2
x + x
- x
1
x
2
= 7
Cu 3: Mt on xe ch 480 tn hng. Khi sp khi hnh c thm 3 xe na
nn mi xe ch t hn 8 tn. Hi lc u on xe c bao nhiu chic,
bit rng cc xe ch khi lng hng bng nhau.
Cu 4: Cho ng trn (O) ng kinh AB = 2R. im M thuc ng trn
sao cho MA < MB. Tip tuyn ti B v M ct nhau N, MN ct AB
ti K, tia MO ct tia NB ti H.
a) T gic OAMN l hnh g ?
b) Chng minh KH // MB.
Cu 5: Tm x, y tho mn 5x - 2
x
(2 + y) + y
2
+ 1 = 0.
S 16
Cu 1: Cho biu thc: K =
x 2x - x
-
x - 1 x - x
vi x >0 v x 1
1) Rt gn biu thc K
2) Tm gi tr ca biu thc K ti x = 4 + 2
3
15
Cu 2: 1) Trong mt phng ta Oxy, ng thng y = ax + b i qua im
M (-1; 2) v song song vi ng thng y = 3x + 1. Tm h s a v b.
2) Gii h phng trnh:
3x 2y 6
x - 3y 2
+

'

Cu 3: Mt i xe nhn vn chuyn 96 tn hng. Nhng khi sp khi hnh


c thm 3 xe na, nn mi xe ch t hn lc u 1,6 tn hng. Hi lc u i
xe c bao nhiu chic.
Cu 4: Cho ng trn (O) vi dy BC c nh v mt im A thay i
trn cung ln BC sao cho AC > AB v AC> BC. Gi D l im chnh gia
ca cung nh BC. Cc tip tuyn ca (O) ti D v C ct nhau ti E. Gi P, Q
ln lt l giao im ca cc cp ng thng AB vi CD; AD vi CE.
1) Chng minh rng: DE//BC
2) Chng minh t gic PACQ ni tip ng trn.
3) Gi giao im ca cc dy AD v BC l F. Chng minh h thc:
1
CE
=
1
CQ
+
1
CF
Cu 5: Cho cc s dng a, b, c. Chng minh rng:

a b c
1 + + 2
a + b b + c c + a
< <
S 17
Cu 1: Cho x
1
=
3 + 5
v x
2
=
3 - 5
Hy tnh: A = x
1
. x
2
; B =
2 2
1 2
x + x
Cu 2: Cho phng trnh n x: x
2
- (2m + 1) x + m
2
+ 5m = 0
a) Gii phng trnh vi m = -2.
b) Tm m phng trnh c hai nghim sao cho tch cc nghim
bng 6.
Cu 3: Cho hai ng thng (d): y = - x + m + 2 v (d): y = (m
2
- 2) x + 1
a) Khi m = -2, hy tm to giao im ca chng.
b) Tm m (d) song song vi (d)
Cu 4: Cho 3 im A, B, C thng hng (B nm gia A v C). V ng trn
tm O ng knh BC; AT l tip tuyn v t A. T tip im T v ng
thng vung gc vi BC, ng thng ny ct BC ti H v ct ng trn ti
K (K

T). t OB = R.
a) Chng minh OH.OA = R
2
.
16
b) Chng minh TB l phn gic ca gc ATH.
c) T B v ng thng song song vi TC. Gi D, E ln lt l giao
im ca ng thng va v vi TK v TA. Chng minh rng
TED cn.
d) Chng minh
HB AB
=
HC AC
Cu 5: Cho x, y l hai s thc tho mn: (x + y)
2
+ 7(x + y) + y
2
+ 10 = 0
Tm gi tr ln nht v gi tr nh nht ca biu thc A = x + y + 1
S 18
Cu 1: Rt gn cc biu thc:
1) 45 20 5 + .
2)
x x x 4
x x 2
+
+
+
vi x > 0.
Cu 2: Mt tha vn hnh ch nht c chu vi bng 72m. Nu tng chiu
rng ln gp i v chiu di ln gp ba th chu vi ca tha vn mi
l 194m. Hy tm din tch ca tha vn cho lc ban u.
Cu 3: Cho phng trnh: x
2
- 4x + m +1 = 0 (1)
1) Gii phng trnh (1) khi m = 2.
2) Tm gi tr ca m phng trnh (1) c 2 nghim x
1
, x
2
tha mn
ng thc
2 2
1 2
x + x
= 5 (x
1
+ x
2
)
Cu 4: Cho 2 ng trn (O) v
(O )
ct nhau ti hai im A, B phn bit.
ng thng OA ct (O),
(O )
ln lt ti im th hai C, D. ng thng
O
A ct (O),
(O )
ln lt ti im th hai E, F.
1. Chng minh 3 ng thng AB, CE v DF ng quy ti mt im I.
2. Chng minh t gic BEIF ni tip c trong mt ng trn.
3. Cho PQ l tip tuyn chung ca (O) v
(O )
(P (O), Q
(O )
).
Chng minh ng thng AB i qua trung im ca on thng PQ.
Cu 5: Gii phng trnh:
1
x
+
2
1
2 x
= 2
S 19
Cu 1: Cho cc biu thc A =
5 7 5 11 11 5
B 5
5 1 11 5 55
, :
+ +
+
+ +
a) Rt gn biu thc A.
17
b) Chng minh: A - B = 7.
Cu 2: Cho h phng trnh
3x + my = 5
mx - y = 1

'

a) Gii h khi m = 2
b) Chng minh h c nghim duy nht vi mi m.
Cu 3: Mt tam gic vung c cnh huyn di 10m. Hai cnh gc vung hn
km nhau 2m. Tnh cc cnh gc vung.
Cu 4: Cho na ng trn (O) ng knh AB. im M thuc na ng
trn, im C thuc on OA. Trn na mt phng b l ng thng AB
cha im M v tip tuyn Ax, By. ng thng qua M vung gc vi MC
ct Ax, By ln lt ti P v Q; AM ct CP ti E, BM ct CQ ti F.
a) Chng minh t gic APMC ni tip ng trn.
b) Chng minh gc

PCQ = 90
0
.
c) Chng minh AB // EF.
Cu 5: Tm gi tr nh nht ca biu thc: P =
4 2
2
x + 2x + 2
x + 1
.
S 20
Cu 1: Rt gn cc biu thc :
a) A =
2 2
-
5 - 2 5 + 2
b) B =
1 x - 1 1 - x
x - : +
x x x + x
_
_



,
,
vi
x 0, x 1. >
Cu 2: Cho phng trnh x
2
- (m + 5)x - m + 6 = 0 (1)
a) Gii phng trnh vi m = 1
b) Tm cc gi tr ca m phng trnh (1) c mt nghim x = - 2
c) Tm cc gi tr ca m phng trnh (1) c nghim x
1
, x
2
tho
mn
2 2
1 2 1 2
x x + x x = 24
Cu 3: Mt phng hp c 360 ch ngi v c chia thnh cc dy c s ch
ngi bng nhau. nu thm cho mi dy 4 ch ngi v bt i 3 dy th
s ch ngi trong phng khng thay i. Hi ban u s ch ngi trong
phng hp c chia thnh bao nhiu dy.
Cu 4: Cho ng trn (O,R) v mt im S ngoi ng trn. V hai tip
tuyn SA, SB ( A, B l cc tip im). V ng thng a i qua S v
18
ct ng trn (O) ti M v N, vi M nm gia S v N (ng thng a
khng i qua tm O).
a) Chng minh: SO AB
b) Gi H l giao im ca SO v AB; gi I l trung im ca MN. Hai
ng thng OI v AB ct nhau ti E. Chng minh rng IHSE l t
gic ni tip ng trn.
c) Chng minh OI.OE = R
2
.
Cu 5: Tm m phng trnh n x sau y c ba nghim phn bit:
x
3
- 2mx
2
+ (m
2
+ 1) x - m = 0 (1).
S 21
Cu 1. 1) Trc cn thc mu s
2
5 1
.
2) Gii h phng trnh :
4
2 3 0
x y
x

'
+

.
Cu 2. Cho hai hm s:
2
x y v
2 + x y
1) V th ca hai hm s ny trn cng mt h trc Oxy.
2) Tm to cc giao im M, N ca hai th trn bng php tnh.
Cu 3. Cho phng trnh ( ) 0 1 1 2 2
2
+ + m x m x vi
m
l tham s.
1) Gii phng trnh khi 2 m .
2) Tm
m
phng trnh c hai nghim
2 1
, x x
tho mn

2 2
1 1 2 2
4 2 4 1 x x x x + +
.
Cu 4. Cho ng trn (O) c ng knh AB v im C thuc ng trn
(C khc A , B ). Ly im D thuc dy BC (D khc B, C). Tia AD ct
cung nh BC ti im E, tia AC ct tia BE ti im F.
1) Chng minh rng FCDE l t gic ni tip ng trn.
2) Chng minh rng DA.DE = DB.DC.
3) Gi I l tm ng trn ngoi tip t gic FCDE, chng minh rng
IC l tip tuyn
ca ng trn (O) .
Cu 5. Tm nghim dng ca phng trnh :
28
9 4
7 7
2
+
+
x
x x .
19
S 22
Cu 1: 1) Gii phng trnh: x
2
- 2x - 15 = 0
2) Trong h trc to Oxy, bit ng thng y = ax - 1 i qua im
M (- 1; 1). Tm h s a.
Cu 2: Cho biu thc: P =

,
_

,
_

1 1 2
1
2
a
a a
a
a a
a
a
vi a > 0, a 1
1) Rt gn biu thc P
2) Tm a P > - 2
Cu 3: Thng ging hai t sn xut c 900 chi tit my; thng hai do ci
tin k thut t I vt mc 15% v t II vt mc 10% so vi thng ging, v
vy hai t sn xut c 1010 chi tit my. Hi thng ging mi t sn
xut c bao nhiu chi tit my?
Cu 4: Cho im C thuc on thng AB. Trn cng mt na mp b AB v
hai tia Ax, By vung gc vi AB. Trn tia Ax ly mt im I, tia vung gc
vi CI ti C ct tia By ti K . ng trn ng knh IC ct IK ti P.
1) Chng minh t gic CPKB ni tip ng trn.
2) Chng minh rng AI.BK = AC.BC.
3) Tnh

APB
.
Cu 5: Tm nghim nguyn ca phng trnh x
2
+ px + q = 0 bit p + q = 198.
S 23
Cu 1.
1) Tnh gi tr ca A = ( ) 5 . 80 5 3 20 + .
2) Gii phng trnh 0 2 7 4
2 4
+ x x .
Cu 2.
1) Tm m ng thng
6 3 + x y
v ng thng
1 2
2
5
+ m x y
ct
nhau ti mt im nm trn trc honh.
2) Mt mnh t hnh ch nht c di ng cho l 13m v chiu
di ln hn chiu rng 7m. Tnh din tch ca hnh ch nht .
Cu 3. Cho phng trnh 0 3 2
2
+ m x x vi
m
l tham s.
1) Gii phng trnh khi 3 m .
20
2) Tm gi tr ca
m
phng trnh trn c hai nghim phn bit
2 1
, x x
tho mn iu kin: 12 2
2 1 2
2
1
+ x x x x .
Cu 4. Cho hai ng trn (O, R) v (O, R) vi R > R ct nhau ti A v B.
K tip tuyn chung DE ca hai ng trn vi D (O) v E (O) sao cho
B gn tip tuyn hn so vi A.
1) Chng minh rng

DAB BDE
.
2) Tia AB ct DE ti M. Chng minh M l trung im ca DE.
3) ng thng EB ct DA ti P, ng thng DB ct AE ti Q.
Chng minh rng PQ song song vi AB.
Cu 5. Tm cc gi tr x
1
3 4
2
+
+
x
x
l s nguyn m.
S 24
Cu 1. Rt gn:
1) A =
5 5
(1 5) .
2 5
+

2) B =
1 1
1 1
x x x x
x x
_ _
+
+ +


+
, ,
vi 0 1 x .
Cu 2. Cho phng trnh ( ) ( ) 0 5 2 3
2
+ + m x m x vi
m
l tham s.
1) Chng minh rng vi mi gi tr ca
m
phng trnh lun c
nghim 2 x .
2) Tm gi tr ca
m
phng trnh trn c nghim
2 2 5 x
.
Cu 3. Mt xe t cn chy qung ng 80km trong thi gian d nh.
V tri ma nn mt phn t qung ng u xe phi chy chm hn vn
tc d nh l 15km/h nn qung ng cn li xe phi chy nhanh hn vn
tc d nh l 10km/h. Tnh thi gian d nh ca xe t .
Cu 4. Cho na ng trn tm O ng knh AB. Ly im C thuc na
ng trn v im D nm trn on OA. V cc tip tuyn Ax, By ca na
ng trn. ng thng qua C, vung gc vi CD ct ct tip tuyn Ax, By
ln lt ti M v N.
1) Chng minh cc t gic ADCM v BDCN ni tip c ng trn.
2) Chng mnh rng

0
90 MDN
.
3) Gi P l giao im ca AC v DM, Q l giao im ca BC v DN.
Chng minh rng PQ song song vi AB.
Cu 5. Cho cc s dng a, b, c. Chng minh bt ng thc:
21

4
a b b c c a a b c
c a b b c c a a b
+ + +
_
+ + + +

+ + +
,
.
S 25
Cu 1. Cho biu thc A =
1 1 2
:
1
1 1
x
x
x x x x
_
_
+




+
,
,
vi a > 0, a 1
1) Rt gn biu thc A.
2) Tnh gi tr ca A khi
2 2 3 x +
.
Cu 2. Cho phng trnh
2
1 0 x ax b + + + vi
b a,
l tham s.
1) Gii phng trnh khi 3 a v 5 b .
2) Tm gi tr ca
b a,
phng trnh trn c hai nghim phn bit
2 1
, x x
tho mn iu kin:

'



9
3
3
2
3
1
2 1
x x
x x
.
Cu 3. Mt chic thuyn chy xui dng t bn sng A n bn sng B cch
nhau 24km. Cng lc , t A mt chic b tri v B vi vn tc dng nc
l 4 km/h. Khi v n B th chic thuyn quay li ngay v gp chic b ti
a im C cch A l 8km. Tnh vn tc thc ca chic thuyn.
Cu 4. Cho ng trong (O, R) v ng thng d khng qua O ct ng trn ti
hai im A, B. Ly mt im M trn tia i ca tia BA k hai tip tuyn MC, MD
vi ng trn (C, D l cc tip im). Gi H l trung im ca AB.
1) Chng minh rng cc im M, D, O, H cng nm trn mt ng trn.
2) on OM ct ng trn ti I. Chng minh rng I l tm ng trn ni
tip tam gic MCD.
3) ng thng qua O, vung gc vi OM ct cc tia MC, MD th t ti P v
Q. Tm v tr ca im M trn d sao cho din tch tam gic MPQ b nht.
Cu 5. Cho cc s thc dng a, b, c tho mn
1
a b c
abc
+ +
.
Tm gi tr nh nht ca biu thc P = ( ) ( )
a b a c + +
.
S 26
Cu 1: 1) Rt gn biu thc:
1 1
2 5 2 5

+
.
22
2) Gii h phng trnh:
3x + y = 9
x - 2y = - 4

'

.
Cu 2: Cho biu thc P =
1 1 x
:
x + x x 1 x + 2 x 1
_


+ +
,
vi x > 0.
1) Rt gn biu thc P.
2) Tm cc gi tr ca x P >
1
2
.
Cu 3: Cho phng trnh n x: x
2
x + m = 0 (1)
1) Gii phng trnh cho vi m = 1.
2) Tm cc gi tr ca m phng trnh (1) c hai nghim x
1
, x
2
tha
mn: (x
1
x
2
1)
2
= 9( x
1
+ x
2
).
Cu 4: Cho t gic ABCD c hai nh B v C trn na ng trn ng knh
AD, tm O. Hai ng cho AC v BD ct nhau ti E. Gi H l hnh chiu vung
gc ca E xung AD v I l trung im ca DE. Chng minh rng:
1) Cc t gic ABEH, DCEH ni tip c ng trn.
2) E l tm ng trn ni tip tam gic BCH.
2) Nm im B, C, I, O, H cng thuc mt ng trn.
Cu 5: Gii phng trnh:
( )
( )
2
x + 8 x + 3 x 11x + 24 1 5 + +
.
S 27
Cu 1: Rt gn cc biu thc sau:
1) A =
1 2
20 80 45
2 3
+
2) B =
5 5 5 5
2 . 2
5 1 5 1
_ _
+
+


+
, ,
Cu 2: 1) Gii h phng trnh:
2x - y = 1 - 2y
3x + y = 3 - x

'

2) Gi x
1
, x
2
l hai nghim ca phng trnh: x
2
x 3 = 0.
Tnh gi tr biu thc P =
1 2
1 1
x x
+
.
Cu 3. Mt xe la i t Hu ra H Ni. Sau 1 gi 40 pht, mt xe la
khc i t H Ni vo Hu vi vn tc ln hn vn tc ca xe la th nht l
23
5 km/h. Hai xe gp nhau ti mt ga cch H Ni 300 km. Tm vn tc ca
mi xe, gi thit rng qung ng st Hu-H Ni di 645km.
Cu 4. Cho na ng trn tm O ng knh AB. C l mt im nm gia
O v A. ng thng vung gc vi AB ti C ct na ng trn trn ti I. K
l mt im bt k nm trn on thng CI (K khc C v I), tia AK ct na
ng trn (O) ti M, tia BM ct tia CI ti D. Chng minh:
1) ACMD l t gic ni tip ng trn.
2) ABD ~ MBC
3) Tm ng trn ngoi tip tam gic AKD nm trn mt ng
thng c nh khi K di ng trn on thng CI.
Cu 5: Cho hai s dng x, y tha mn iu kin x + y = 1.
Hy tm gi tr nh nht ca biu thc: A =
2 2
1 1
x y xy
+
+
S 28
Cu 1: 1) Gii h phng trnh:
2x + y = 7
x - 3y = - 7

'

2) Gi x
1
, x
2
l hai nghim ca phng trnh: 3x
2
x 2 = 0.
Tnh gi tr biu thc P = x
1
2
+ x
2
2
.
Cu 2: Cho biu thc A =
a a a 1
:
a - 1
a 1 a + a
_



+
,
vi a > 0, a

1.
1) Rt gn biu thc A.
2) Tm cc gi tr ca a A < 0.
Cu 3: Cho phng trnh n x: x
2
2mx - 1 = 0 (1)
1) Chng minh rng phng trnh cho lun c hai nghim phn
bit x
1
v x
2
.
2) Tm cc gi tr ca m : x
1
2
+ x
2
2
x
1
x
2
= 7.
Cu 4: Cho na ng trn tm O ng knh AB = 2R v tia tip tuyn Ax
cng pha vi na ng trn i vi AB. T im M trn Ax k tip tuyn
th hai MC vi na ng trn (C l tip im). AC ct OM ti E; MB ct
na ng trn (O) ti D (D khc B).
1) Chng minh: AMDE l t gic ni tip ng trn.
2) MA
2
= MD.MB
3) V CH vung gc vi AB (H

AB). Chng minh rng MB i qua


trung im ca CH.
24
Cu 5: Gii phng trnh:
4 1 5
x - x + 2x -
x x x
+
S 29
Cu 1: a) Cho ng thng d c phng trnh:
y mx 2m 4 +
. Tm m
th hm s i qua gc ta .
b) Vi nhng gi tr no ca m th th hm s
2 2
y m m x ( ) i
qua im A(-1; 2).
Cu 2: Cho biu thc P =

,
_

,
_

+
+
a a a
3
1
3
1
3
1
vi a > 0 v a

9.
a) Rt gn biu thc P
b) Tm cc gi tr ca a P >
2
1
.
Cu 3: Hai ngi cng lm chung mt cng vic th hon thnh trong 4 gi.
Nu mi ngi lm ring, hon thnh cng vic th thi gian ngi th
nht t hn thi gian ngi th hai l 6 gi. Hi nu lm ring th mi ngi
phi lm trong bao lu hon thnh cng vic.
Cu 4: Cho na ng trn ng knh BC = 2R. T im A trn na ng
trn v AH BC. Na ng trn ng knh BH, CH ln lt c tm O
1
;
O
2
ct AB, AC th t ti D v E.
a) Chng minh t gic ADHE l hnh ch nht, t tnh DE bit R =
25 v BH = 10
b) Chng minh t gic BDEC ni tip ng trn.
c) Xc nh v tr im A din tch t gic DEO
1
O
2
t gi tr ln
nht. Tnh gi tr .
Cu 5: Gii phng trnh: x
3
+ x
2
- x = -
1
3
.
S 30
Cu 1. 1) Gii phng trnh: 0 75 3 + x .
2) Gii h phng trnh

'

+

4 2
1 2 3
y x
y x
.
25
Cu 2. Cho phng trnh ( ) 0 3 2
2
+ + m x m x (1) vi
m
l tham s.
1) Gii phng trnh khi 2 m .
2) Chng t phng trnh (1) c nghim vi mi gi tr ca m. Gi
2 1
, x x
l cc nghim ca phng trnh (1). Tm gi tr nh nht ca biu thc
sau: A =
2 1
x x
.
Cu 3.
1) Rt gn biu thc P =
3
2
9 25 4
2
a a a
a a
+
+
vi 0 a > .
2) Khong cch gia hai bn sng A v B l 48 km. Mt can xui
dng t bn A n bn B, ri quay li bn A. Thi gian c i v v l 5 gi
(khng tnh thi gian ngh). Tnh vn tc ca can trong nc yn lng, bit
rng vn tc ca dng nc l 4 km/h.
Cu 4. Cho tam gic vung ABC ni tip trong ng trn tm O ng
knh AB. Trn tia i ca tia CA ly im D sao cho CD = AC.
1) Chng minh tam gic ABD cn.
2) ng thng vung gc vi AC ti A ct ng trn (O) ti E (E

A). Tn tia i ca tia EA ly im F sao cho EF = AE. Chng minh rng


ba im D, B, F cng nm trn mt ng thng.
3) Chng minh rng ng trn i qua ba im A, D, F tip xc vi
ng trn (O).
Cu 5. Cho cc s dng
c b a , ,
. Chng minh bt ng thc:
2 >
+
+
+
+
+ b a
c
a c
b
c b
a
.
S 31
Cu 1: Tnh:
a) A 20 3 18 45 72 + .
b)
B 4 7 4 7 + +
.
c)
C x 2 x 1 x 2 x 1 + +
vi x > 1
Cu 2: Cho hm s y = (2m - 1)x - m + 2
a) Tm m hm s nghch bin trn R.
b) Tm m th hm s i qua A (1; 2)
26
Cu 3: Hai ngi th cng lm cng vic trong 16 gi th xong. Nu ngi
th nht lm 3 gi, ngi th hai lm 6 gi th h lm c
4
1
cng vic.
Hi mi ngi lm mt mnh th trong bao lu lm xong cng vic?
Cu 4: Cho ba im A, B, C c nh thng hng theo th t . V ng
trn (O; R) bt k i qua B v C (BC

2R). T A k cc tip tuyn AM, AN


n (O) (M, N l tip im). Gi I, K ln lt l trung im ca BC v MN;
MN ct BC ti D. Chng minh:
a) AM
2
= AB.AC
b) AMON; AMOI l cc t gic ni tip ng trn.
c) Khi ng trn (O) thay i, tm ng trn ngoi tip OID lun
thuc mt ng thng c nh.
Cu 5: Tm cc s nguyn x, y tha mn phng trnh: (2x +1)y = x +1.
S 32
Cu 1: 1) Rt gn biu thc: P = ( 7 3 2)( 7 3 2) + + .
2) Trong mp to Oxy, tm m ng thng (d):
2
y m 1 x 1 ( ) +
song song vi ng thng
d y 3x m 1 ( ): +
.
Cu 2: Cho phng trnh x
2
+ (2m + 1) x + m
2
+ 1 = 0 (1)
a) Gii phng trnh (1) khi m = 1
b) Tm m phng trnh (1) c 2 nghim m.
Cu 3: Cho a, b l cc s dng tho mn ab = 1. Tm gi tr nh nht ca
biu thc: A = (a + b + 1)(a
2
+ b
2
) +
b a +
4
.
Cu 4: Qua im A cho trc nm ngoi ng trn (O) v 2 tip tuyn AB,
AC (B, C l cc tip im), ly im M trn cung nh BC, v MH BC;
MI AC; MK AB.
a) Chng minh cc t gic: BHMK, CHMI ni tip ng trn.
b) Chng minh MH
2
= MI.MK
c) Qua M v tip tuyn vi ng trn (O) ct AB, AC ti P, Q. Chng
minh chu vi APQ khng ph thuc vo v tr im M.
27
Cu 5: Chng minh nu
a 2 >
th h phng trnh:
5
2 2
x 2y a (1)
x y 1 (2)

'
+

v
nghim.
S 33
Cu 1: a) Gii h phng trnh:
x 3y 10
2x y 1
+

'
+

.
b) Vi gi tr no ca m th hm s y = (m + 2) x - 3 ng bin trn tp
xc nh.
Cu 2: Cho biu thc A =

,
_

+ + +

,
_

1
2
1
1
:
1
2
1
a a a a
a
a
a
a
vi
a > 0, a 1
a) Rt gn biu thc A.
b) Tnh gi tr ca A khi a = 2011 - 2 2010 .
Cu 3: Cho phng trnh: k (x
2
- 4x + 3) + 2(x - 1) = 0.
a) Gii phng trnh vi k = -
2
1
.
b) Chng minh rng phng trnh lun c nghim vi mi gi tr ca k.
Cu 4: Cho hai ng trn (O; R) v (O; R) tip xc ngoi ti A. V tip
tuyn chung ngoi BC (B, C th t l cc tip im thuc (O; R) v (O; R)).
a) Chng minh

BAC
= 90
0
.
b) Tnh BC theo R, R.
c) Gi D l giao im ca ng thng AC v ng trn (O) (D

A),
v tip tuyn DE vi ng trn (O) (E

(O)). Chng minh BD = DE.


Cu 5: Cho hai phng trnh: x
2
+ a
1
x + b
1
= 0 (1) , x
2
+ a
2
x + b
2
= 0 (2)
Cho bit a
1
a
2
> 2 (b
1
+ b
2
) . Chng minh t nht mt trong hai phng trnh
cho c nghim.
S 34
Cu 1: Rt gn biu thc: P =
2 2
) 1 1 ( ) 1 1 ( + + a a
vi a > 1
Cu 2: Cho biu thc: Q =

,
_

,
_

1
1
1
1
2
1
2
2
x
x
x
x
x
x
.
1) Tm tt c cc gi tr ca x Q c ngha. Rt gn Q.
28
2) Tm tt c cc gi tr ca x Q = - 3 x - 3.
Cu 3: Cho phng trnh x
2
+ 2 (m - 1)
x
+ m + 1 = 0 vi m l tham s.
Tm tt c cc gi tr ca m phng trnh c ng 2 nghim phn bit.
Cu 4: Gii phng trnh:
26 2 19 6 3
2 2
+ + + x x x x
= 8 - x
2
+ 2x .
Cu 5: Cho ng trn (O), ng knh AB, d
1
, d
2
l cc cc ng thng
ln lt qua A, B v cng vung gc vi ng thng AB. M, N l cc im
ln lt thuc d
1
, d
2
sao cho

MON
= 90
0
.
1) Chng minh ng thng MN l tip tuyn ca ng trn (O).
2) Chng minh AM . AN =
4
2
AB
.
3) Xc nh v tr ca M, N din tch tam gic MON t gi tr nh nht.
S 35
Cu 1: Rt gn A =
3
9 6
2
+
+ +
x
x x
vi x 3 .
Cu 2: a) Gii phng trnh
2
x 2x 4 2 +
.
b) Vit phng trnh ng thng (d) i qua 2 im A(1; 2) v B(2; 0).
Cu 3: Cho phng trnh: (x
2
- x - m)(x - 1) = 0 (1)
a) Gii phng trnh khi m = 2.
b) Tm m phng trnh c ng 2 nghim phn bit.
Cu 4: T im M ngoi ng trn (O; R) v hai tip tuyn MA, MB
(tip im A; B) v ct tuyn ct ng trn ti 2 im C v D khng i qua
O. Gi I l trung im ca CD.
a) Chng minh 5 im M, A, I, O, B cng thuc mt ng trn.
b) Chng minh IM l phn gic ca

AIB
.
Cu 5: Gii h phng trnh:
4 4
3 3 2 2
x y 1
x y x y

'
+ +

.
S 36
Cu 1: a) Tnh
2 2
(1 5) (1 5) + +
.
b) Gii phng trnh: x
2
+ 2x - 24 = 0.
29
Cu 2: Cho biu thc: P =
a
a
a
a
a
a

+
+

+
+
+
9
7 3
3
1
3
2
vi a > 0, a

9.
a) Rt gn.
b) Tm a P < 1.
Cu 3: Cho phng trnh: x
4
- 5x
2
+ m = 0 (1)
a) Gii phng trnh khi m = 4.
b) Tm m phng trnh (1) c ng 2 nghim phn bit.
Cu 4: Cho ng trn (O), t im A ngoi ng trn v ng thng AO
ct ng trn (O) ti B, C (AB < AC). Qua A v ng thng khng i qua
(O) ct ng trn (O) ti D; E (AD < AE). ng thng vung gc vi AB
ti A ct ng thng CE ti F.
a) Chng minh t gic ABEF ni tip ng trn.
b) Gi M l giao im th hai ca FB vi ng trn (O), chng minh
DM AC.
c) Chng minh: CE . CF + AD . AE = AC
2
.
Cu 5: Tm gi tr nh nht ca hm s: y =
x x
1
1
2
+

, vi 0 < x < 1
S 37
Cu 1: Cho biu thc: M = 1
1 1
2 2
+ +
+
+

+ +

x
x x
x x
x x
x x
Rt gn biu thc M vi x 0.
Cu 2: a) Gii h phng trnh:
3x 5y 18
x 2y 5

'
+


b) Trong mt phng to Oxy, vi gi tr no ca a, b th ng thng (d):
y = ax + 2 - b v ng thng (d): y = (3 - a)x + b song song vi nhau.
Cu 3: Cho phng trnh: x
2
- 2x + m = 0 (1)
a) Gii phng trnh khi m = - 3.
b) Tm m phng trnh (1) c 2 nghim x
1
, x
2
tho mn:
2
2
2
1
1 1
x x
+
= 1.
Cu 4: ChoABC c 3 gc nhn, trc tm l H v ni tip ng trn (O).
V ng knh AK.
a) Chng minh t gic BHCK l hnh hnh hnh.
b) V OM BC (M

BC). Chng minh H, M, K thng hng v


AH = 2.OM.
30
c) Gi A, B, C l chn cc ng cao thuc cc cnh BC, CA, AB ca
ABC. Khi BC c nh hy xc nh v tr im A tng S = AB + BC
+ CA t gi tr ln nht.
Cu 5: Tm gi tr nh nht ca biu thc: y =
2
2
x x 1
x 2x 2
+ +
+ +
.
S 38
Cu 1: Cho biu thc: P =
x
x x
x x
x x +
+
+
+ 2
1
1
2
vi x > 0.
a) Rt gi biu thc P.
b) Tm x P = 0.
Cu 2: a) Gii phng trnh: x +
1 1
2
x
b) Gii h phng trnh:
6x 6y 5xy
.
4 3
1
x y
+

'

Cu 3: Cho phng trnh: x


2
- 2(m - 1)x + m + 1= 0.(1)
a) Gii phng trnh khi m = - 1.
b) Tm m phng trnh (1) c 2 nghim x
1
, x
2
tho mn
4
1
2
2
1
+
x
x
x
x
.
Cu 4: ABC cn ti A. V ng trn (O; R) tip xc vi AB, AC ti B, C.
ng thng qua im M trn BC vung gc vi OM ct tia AB, AC ti D, E.
a) Chng minh 4 im O, B, D, M cng thuc mt ng trn.
b) MD = ME.
Cu 5: Gii phng trnh: x
2
+ 3x + 1 = (x + 3)
1
2
+ x
S 39
Cu 1:
1) Tnh: 48 - 2 75 + 108
2) Rt gn biu thc: P=
1 1 1
- . 1 -
1 - x 1 + x x
_ _

, ,
vi x 1 v x >0
Cu 2: 1) Trn h trc ta Oxy, ng thng y = ax + b i qua 2 im M
(3; 2) v N (4; -1).
Tm h s a v b.
31
2) Gii h phng trnh:
2x + 5y = 7
3x - y = 2

'

Cu 3: Cho phng trnh: x


2
- 2mx - 6m = 0 (1)
1). Gii phng trnh (1) khi m = 2
2) Tm m phng trnh (1) c 1 nghim gp 2 ln nghim kia.
Cu 4: Cho ng trn (O), ng knh AB c nh, im I nm gia A v O
sao cho AI =
2
3
AO. K dy MN vung gc vi AB ti I, gi C l im ty
thuc cung ln MN sao cho C khng trng vi M, N v B. Ni AC ct MN ti E.
1) Chng minh t gic IECB ni tip .
2) Chng minh h thc: AM
2
= AE.AC.
3) Hy xc nh v tr ca im C sao cho khong cch t N n tm
ng trn ngoi tip tam gic CME l nh nht.
Cu 5: Cho x v y l hai s tha mn ng thi : x 0 , y 0, 2x + 3y 6
v 2x + y 4.
Tm gi tr nh nht v gi tr ln nht ca biu thc K = x
2
- 2x y.
S 40
Cu 1. Trong h trc ta Oxy, cho ng thng d c phng trnh: 3x + 4y = 2.
a) Tm h s gc ca ng thng d.
b) Vi gi tr no ca tham s m th ng thng d
1
: y = (m
2
-1)x + m
song song vi ng thng d.
Cu 2. Tm a, b bit h phng trnh
ax by 3
bx ay 11
+

'

c nghim
x 3
y 1

'

.
Cu 3. Cho phng trnh:
2
(1 3)x 2x 1 3 0 + + (1)
a) Chng t phng trnh (1) lun c 2 nghim phn bit.
b) Gi 2 nghim ca phng trnh (1) l
1 2
x , x
. Lp mt phng trnh
bc 2 c 2 nghim l
1
1
x
v
2
1
x
.
Cu 4. Bn trong hnh vung ABCD v tam gic u ABE . V tia Bx thuc
na mt phng cha im E, c b l ng thng AB sao cho Bx vung gc
vi BE. Trn tia Bx ly im F sao cho BF = BE.
a) Tnh s o cc gc ca tam gic ADE.
b) Chng minh 3 im: D, E, F thng hng.
32
c) ng trn tm O ngoi tip tam gic AEB ct AD ti M. Chng
minh ME // BF.
Cu 5. Hai s thc x, y tho mn h iu kin :
3 2
2 2 2
x 2y 4y 3 0 (1)
x x y 2y 0 (2)
+ +

'
+

.
Tnh gi tr biu thc P =
2 2
x y + .
33
II - N THI TUYN SINH LP 10 CHUYN TON
S 1
Cu 1: Gii cc phng trnh:
a)
2
2
4 2
x 4 x - 9 0
x x
_ _
+

, ,
b)
( )
( )
2
x + 5 x + 2 1 x 7x + 10 3 + +
Cu 2:
a) Cho 3 s a, b, c khc 0 tha mn: abc = 1 v

3 3 3
3 3 3
a b c b c a
b c a a b c
+ + + + .
Chng minh rng trong 3 s a, b, c lun tn ti mt s l lp phng ca mt
trong hai s cn li.
b) Cho x =
3 3
84 84
1 1
9 9
+ +
. Chng minh x c gi tr l mt s nguyn.
Cu 3: Cho cc s dng x, y, z tha mn: x + y + z 3.Tm gi tr ln nht
ca biu thc:
A =
( )
2 2 2
1 x 1 y 1 z 2 x y z + + + + + + + +
.
Cu 4: Cho ng trn ( O; R ) v im A nm ngoi ng trn sao cho
OA = R
2
. T A v cc tip tuyn AB, AC vi ng trn (B, C l cc tip
im). Ly D thuc AB; E thuc AC sao cho chu vi ca tam gic ADE bng
2R.
a) Chng minh t gic ABOC l hnh vung.
b) Chng minh DE l tip tuyn ca ng trn (O; R).
c) Tm gi tr ln nht ca din tch ADE.
Cu 5: Trn mt phng cho 99 im phn bit sao cho t 3 im bt k trong
s chng u tm c 2 im c khong cch nh hn 1. Chng minh rng tn
ti mt hnh trn c bn knh bng 1 cha khng t hn 50 im.
S 2
Cu 1: a) Tm cc s hu t x, y tha mn ng thc:
x (
3 3
2011 2010) y( 2011 2010) 2011 2010 + + +
34
b) Tm tt c cc s nguyn x > y > z > 0 tho mn:
xyz + xy + yz + zx + x + y + z = 2011.
Cu 2: a) Gii phng trnh: 2(x
2
+ 2) = 5
1
3
+ x
.
b) Cho a, b, c

[0; 2] v a + b + c = 3. Chng minh a


2
+ b
2
+ c
2
< 5.
Cu 3: Tm tt c cc s hu t x sao cho gi tr ca biu thc x
2
+ x + 6 l
mt s chnh phng.
Cu 4: Cho ng trn (O) ngoi tip ABC c H l trc tm. Trn cung
nh BC ly im M.
Gi N, I, K ln lt l hnh chiu ca M trn BC, CA, AB. Chng minh:
a) Ba im K, N, I thng hng.
b)
MN
BC
MI
AC
MK
AB
+
.
c) NK i qua trung im ca HM.
Cu 5: Tm GTLN v GTNN ca biu thc: P = 2x
2
- xy - y
2
vi x, y tho
mn iu kin sau:
x
2
+ 2xy + 3y
2
= 4.
S 3
Cu 1: a) Cho a, b, c l 3 s tng i mt khc nhau v tho mn:

a b c
+ + = 0
b - c c - a a - b
Chng minh rng:
2 2 2
a b c
+ + = 0
(b - c) (c - a) (a - b)

b) Tnh gi tr ca biu thc:
A =
2
2 4 4
4 4
2 1
1 + +
2010 2010 - 2010 1 + 2010
2010
+ -
1 - 2010 2010 1 + 2010
_


,
Cu 2: a) Cho a, b, c l di 3 cnh tam gic, chng minh:

2 2 2
1 1 1 a + b + c
+ +
a + bc b + ac c + ab 2abc

.
b) Cho biu thc: A = x - 2 xy +3y - 2 x + 1. Tm gi tr nh nht
ca A.
35
Cu 3: a) Gii phng trnh: 2 x - 1 + 3 5 - x = 2 13 .
b) Cho hm s y = f(x) vi f(x) l mt biu thc i s xc nh vi
mi s thc x khc
khng. Bit rng: f(x) + 3f
1
x
_

,
= x
2
x 0. Tnh gi tr ca f(2).
Cu 4: Cho lc gic u ABCDEF. Gi M l trung im ca EF, K l
trung im ca BD. Chng minh tam gic AMK l tam gic u.
Cu 5: Cho t gic li ABCD c din tch S v im O nm trong t gic
sao cho:OA
2
+ OB
2
+ OC
2
+ OD
2
= 2S. Chng minh ABCD l hnh vung c
tm l im O.
S 4
Cu 1: a) Cho x v y l 2 s thc tho mn x
2
+ y
2
= 4. Tm gi tr ln nht
ca biu thc : A =
xy
x + y + 2
.
b) Cho x, y, z l 3 s thc dng tho mn x
2
+ y
2
+ z
2
= 2. Chng minh:

3 3 3
2 2 2 2 2 2
2 2 2 x + y + z
+ + + 3
x + y y + z z + x 2 xyz

.
Cu 2: a) Gii phng trnh: x
2
+ 9x + 20 = 2 3x + 10 .
b) Tm x, y tho mn:
2 2 2
2 3
x y - 2x + y = 0
2x - 4x + 3 = - y

'

.
Cu 3: a) Chng minh rng nu:
2 4 2 2 2 4
3 3
x + x y + y + x y = a
th
3 2 2 3 2
3
x + y = a .
b) Chng minh rng nu phng trnh x
4
+ ax
3
+ bx
2
+ ax +1 = 0 c
nghim th 5(a
2
+ b
2
) 4.
Cu 4: Cho na ng trn tm (O) ng knh AB = 2R v bn knh OC
vung gc vi AB. Tm im M trn na ng trn sao cho 2MA
2
= 15MK
2
,
trong K l chn ng vung gc h t M xung OC.
Cu 5: Cho hnh thang ABCD (AB//CD). Gi E v F ln lt l trung im
ca BD v AC. Gi G l giao im ca ng thng i qua F vung gc vi
AD vi ng thng i qua E vung gc vi BC. So snh GD v GC.
36
S 5
Cu 1: 1) Gii phng trnh: x
2
+
2
2
81x
= 40
(x + 9)
.
2) Gii phng trnh:
x
2
- 2x + 3(x - 3)
x + 1
x - 3
= 7.
Cu 2: 1) Tm gi tr nh nht biu thc: A =
2
5 - 3x
1 - x
.
2) Cho a, b, c l di 3 cnh ca tam gic. Chng minh:

2 2 2 2 2 2
a + b + b + c + c + a 2 (a + b + c).
Cu 3: Gii h phng trnh:
2
2 2
y - xy + 1 = 0 (1)
x + 2x + y +2y + 1 = 0 (2)

'

Cu 4: Cho hnh thang ABCD c 2 y BC v AD (BC

AD). Gi M, N l 2
im ln lt trn 2 cnh AB v DC sao cho
AM CN
=
AB CD
. ng thng MN
ct AC v BD tng ng vi E v F. Chng minh EM = FN.
Cu 5: Cho ng trn tm (O) v dy AB, im M chuyn ng trn ng
trn. T M k MH vung gc vi AB (H

AB). Gi E, F ln lt l hnh chiu


vung gc ca H trn MA, MB. Qua M k ng thng vung gc vi EF ct
AB ti D.
1) Chng minh ng thng MD lun i qua 1 im c nh khi M thay
i trn ng trn.
2) Chng minh:
2
2
MA AH AD
=
MB BD BH
.
S 6
Cu 1: Tnh gi tr biu thc: A =

1 1 1
+ + +
1 + 2 2 + 3 24 + 25

.
37
Cu 2: a) Cho cc s khc khng a, b, c. Tnh gi tr ca biu thc:
M = x
2011
+ y
2011
+ z
2011
Bit x, y, z tho mn iu kin:
2 2 2 2 2 2
2 2 2 2 2 2
x + y + z x y z
= + +
a + b + c a b c
b) Chng minh rng vi a >
1
8
th s sau y l mt s nguyn dng.
x =
3 3
a + 1 8a - 1 a + 1 8a - 1
a + + a - .
3 3 3 3
Cu 3: a) Cho a, b, c > 0 tho mn:
1 35 4c
+
1 + a 35 + 2b 4c + 57

. Tm gi
tr nh nht ca A = a.b.c.
b) Gi s a, b, c, d, A, B, C, D l nhng s dng v

a b c d
= = =
A B C D
. Chng minh rng:
aA + bB + cC + dD = (a + b + c + d) (A +B + C + D)
Cu 4: Cho tam gic ABC c ba gc nhn. Gi M, N, P, Q l bn nh ca
mt hnh ch nht (M v N nm trn cnh BC, P nm trn cnh AC v Q nm
trn cnh AB).
a) Chng minh rng: Din tch hnh ch nht MNPQ c gi tr ln nht
khi PQ i qua trung im ca ng cao AH.
b) Gi s AH = BC. Chng minh rng, mi hnh ch nht MNPQ u
c chu vi bng nhau.
Cu 5: Cho tam gic ABC vung cn A, ng trung tuyn BM. Gi D l
hnh chiu ca C trn tia BM, H l hnh chiu ca D trn AC. Chng minh
rng AH = 3HD.
B - PHN LI GII
38
I - LP 10 THPT
S 1
Cu 1: a) Ta c: a + b = ( 2 3 + ) + ( 2 3 ) = 4
a.b = ( 2 3 + )( 2 3 = 1. Suy ra P = 3.
3x + y = 5 6x + 2y = 10 7x = 7 x = 1
b)
x - 2y = - 3 x - 2y = - 3 y = 5 - 3x y = 2


' ' ' '

.
Cu 2:
1 1 x
a) P = :
x - x x 1 x - 2 x 1
_
+

+
,

( ) ( )
( )
2
x 1
1 x
.
x
x x 1 x x 1
_


+


,


( )
( ) ( ) ( )
2
x 1 x 1 x 1
1 x x - 1
.
x
x x. x
x x 1
+
+

b) Vi x > 0, x

1 th ( )
x - 1 1
2 x - 1 x
x 2
> > x > 2 .
Vy vi x > 2 th P >
1
2
.
Cu 3: a) Vi m = 6, ta c phng trnh: x
2
5x + 6 = 0
= 25 4.6 = 1 . Suy ra phng trnh c hai nghim: x
1
= 3; x
2
= 2.
b) Ta c: = 25 4.m
phng trnh cho c nghim th 0
25
m
4

(*)
Theo h thc Vi-t, ta c x
1
+ x
2
= 5 (1); x
1
x
2
= m (2).
Mt khc theo bi ra th
1 2
x x 3
(3). T (1) v (3) suy ra x
1
= 4; x
2
= 1
hoc x
1
= 1; x
2
= 4 (4)
T (2) v (4) suy ra: m = 4. Th li th tho mn.
39
Cu 4:
a) T gic BEFI c:

0
BIF 90
(gt) (gt)

0
BEF BEA 90
(gc ni tip chn na ng
trn)
Suy ra t gic BEFI ni tip ng trn ng
knh BF
b) V AB CD nn

AC AD
,
suy ra

ACF AEC
.
Xt ACF v AEC c gc A chung v


ACF AEC
.
Suy ra: ACF ~ vi AEC
AC AE
AF AC

2
AE.AF = AC
F
E
I O
D
C
B
A
c) Theo cu b) ta c

ACF AEC
, suy ra AC l tip tuyn ca ng trn
ngoi tip CEF (1).
Mt khc

0
ACB 90
(gc ni tip chn na ng trn), suy ra AC CB
(2). T (1) v (2) suy ra CB cha ng knh ca ng trn ngoi tip
CEF, m CB c nh nn tm ca ng trn ngoi tip CEF thuc CB c
nh khi E thay i trn cung nh BC.
Cu 5: Ta c (a + b)
2
4ab = (a - b)
2
0

(a + b)
2
4ab

( )
( ) ( )
a + b
4 1 1 4
ab a + b b a a + b
+
( )
4
P
a + b

, m a + b
2 2
( )
4 4
a + b
2 2

P 2
. Du = xy ra
( )
2
a - b 0
a = b = 2
a + b = 2 2


'

. Vy: min P =
2
.
Li bnh:
Cu IIb
Cc bn tham kho thm mt li gii sau
1) Ta c a = 1. = 25 4m. Gi x
1
,

x
2
l cc nghim nu c ca phng trnh.
T cng thc
1,2
2
b
x
a
t


1 2
| |
| |
x x
a

. Vy nn phng trnh c
hai nghim x
1
,

x
2
tho mn |x
1
x
2
| = 3
1 2
| | 3
| |
x x
a



1 a

= 9
25 4m = 9 m = 4 .
40
2) C th bn dang bn khon khng thy iu kin 0. Xin ng, bi
|x
1
x
2
| = 3 = 9. iu bn khon y cng lm ni bt u im ca
li gii trn. Li gii gim thiu ti a cc php ton, iu y ng
hnh gim bt nguy s sai st.
Cu IVb

chng minh mt ng thc ca tch cc on thng ngi ta
thng gn cc on thng y vo mt cp tam gic ng dng. Mt th
thut d nhn ra cp tam gic ng dng l chuyn "hnh thc" ng
thc on thng dng tch v dng thng. Khi mi tam gic c
xt s c cnh hoc l nm cng mt v, hoc cng nm t thc, hoc
cng nm mu thc.
Trong bi ton trn AE.AF = AC
2

AC AE
AF AC

. ng thc mch bo
ta xt cc cp tam gic ng dng ACF (c cnh nm v tri) v ACE
(c cnh nm v phi).
Khi mt on thng l trung bnh nhn ca hai on thng cn li,
chng hn AE.AF = AC
2
th AC l cnh chung ca hai tam gic, cn AE
v AF khng cng nm trong mt tam gic cn xt.
Trong bi ton trn AC l cnh chung ca hai tam gic ACE v
ACF
Cu IVc
Nu () l ng thng c nh cha tm ca ng trn bin thin
c cc c im sau:
+ Nu ng trn c hai im c nh th () l trung trc ca on
thng ni hai im c nh y.
+ Nu ng trn c mt im c nh th () l ng thng i qua
im v
hoc l () ('),
hoc l () // ('),
hoc l () to vi (') mt gc khng i
(trong (') l mt ng thng c nh c sn).
Trong bi ton trn, ng trn ngoi tip CEF ch c mt im C l
c nh. Li thy CB CA m CA c nh nn phn on c th
CB l ng thng phi tm. l iu dn dt li gii trn.
41

Cu V
Vic tm GTNN ca biu thc P bao gi cng vn hnh theo s "b
dn": P B, (trong ti liu ny chng ti s dng B - ch ci u ca
ch b hn).
1) Gi thit a + b 2 2 ang ngc vi s "b dn" nn ta phi
chuyn ho a + b 2 2
1 1
2 2
a b

+
.
T m li gii nh gi P theo
1
a b +
.
2)
1 1 4
a b a b
+
+
vi a > 0, b > 0 l mt bt ng thc ng nh. Tuy
l mt h qu ca bt ng
C-si, nhng n c vn dng rt nhiu. Chng ta cn gp li n
trong mt s sau.
3) Cc bn tham kho li gii khc ca bi ton nh l mt cch
chng minh bt ng thc trn.
Vi hai s a > 0, b > 0 ta c
1 1 2 2.2 4 4
2
2 2
Co si Co si
P
a b a b a b
ab

+
+ +
. Du ng thc c khi a
= b =
2
. Vy minP =
2
.
S 2
Cu 1: a)
( ) ( )
( ) ( )
3 7 3 7
1 1 2 7
7
2
3 7 3 7
3 7 3 7
+

+
+
b) = 49 4.3 = 37; phng trnh c 2 nghim phn bit:
1 2
7 37 7 37
x ; x
2 2
+
.
Cu 2: a) Honh giao im ca ng thng (d) v Parabol (P) l nghim
ca phng trnh: - x + 2 = x
2

x
2
+ x 2 = 0. Phng trnh ny c tng cc
h s bng 0 nn c 2 nghim l 1 v 2.
+ Vi x = 1 th y = 1, ta c giao im th nht l (1;1)
+ Vi x = - 2 th y = 4, ta c giao im th hai l (- 2; 4)
42
Vy (d) giao vi (P) ti 2 im c ta l (1;1) v (- 2; 4)
b) Thay x = 2 v y = -1 vo h cho ta c:

( )
a = 2 + b
8 - a = b a = 5
8 - 2 + b b 2 + b = a b = 3


' ' '

.
Th li : Thay a = 5 v b = 3 vo h cho th h c nghim duy nht (2; - 1).
Vy a = 5; b = 3 th h cho c nghim duy nht (2; - 1).
Cu 3: Gi x l s toa xe la v y l s tn hng phi ch
iu kin: x

N
*
, y > 0.
Theo bi ra ta c h phng trnh:
15x = y - 5
16x = y + 3

'

. Gii ra ta c: x = 8, y =
125 (tha mn)
Vy xe la c 8 toa v cn phi ch 125 tn hng.
Cu 4:
a) Ta c:

0
AIM AKM 90
(gt), suy ra t gic AIMK ni tip ng trn
ng knh AM.
b) T gic CPMK c

0
MPC MKC 90
(gt). Do CPMK l t gic ni
tip

MPK MCK
(1). V KC l tip tuyn ca (O) nn ta c:

MCK MBC
(cng chn

MC
) (2). T (1) v (2) suy ra

MPK MBC
(3)
c)
Chng minh tng t cu b ta c BPMI l t gic
ni tip.
Suy ra:

MIP MBP
(4). T (3) v (4) suy ra

MPK MIP
.
Tng t ta chng minh c

MKP MPI
.
Suy ra: MPK
~
MIP

MP MI
MK MP

MI.MK = MP
2

MI.MK.MP = MP
3
.
Do MI.MK.MP ln nht khi v ch khi MP
ln nht (4)
- Gi H l hnh chiu ca O trn BC, suy ra OH l
hng s (do BC c nh).
Li c: MP + OH OM = R

MP R OH.
Do MP ln nht bng R OH khi v ch khi
O, H, M thng hng hay M nm chnh gia cung
nh BC (5). T (4) v (5) suy ra max
(MI.MK.MP) = ( R OH )
3

M nm chnh gia
cung nh BC.
H
O
P
K
I
M
C
B
A
43
Cu 5: t x - 2009 a; y - 2010 b; z - 2011 c
(vi a, b, c > 0). Khi phng trnh cho tr thnh:
2 2 2
a - 1 b - 1 c - 1 3
a b c 4
+ +
2 2 2
1 1 1 1 1 1 1 1 1
0
4 a a 4 b b 4 c c
_ _ _
+ + + + +

, , ,
2 2 2
1 1 1 1 1 1
0
2 a 2 b 2 c
_ _ _
+ +

, , ,

a = b = c = 2
Suy ra: x = 2013, y = 2014, z = 2015.
Li bnh:
Cu IVc
Li bnh sau s 1 cho thy: Nu c AE.AF.AC = AC
3
AE.AF
= AC
2
th thng AC l cnh chung ca hai tam gic ACE v ACF.
Quan st hnh v ta thy MP l cnh chung ca hai tam gic MPI
v MPK, nn ta phn on MI.MK.MP= MP
3
.
Nu phn on y l ng th GTLN ca MI.MK.MP chnh l
GTLN ca MP. l iu dn dt li gii trn.
Cu IIa
Li nhn
Honh giao im ca hai th (d): y = kx + b v (P) : y = ax
2
l
nghim ca phng trnh ax
2
= kx + b (1). S nghim ca phng trnh
(1) bng s giao im ca th hai hm s trn.
Cu V
1) Vic t a, b, c thay cho cc cn thc l cch lm d nhn
bi ton, Vi mi s dng a, b, c ta lun c

2 2 2
1 1 1 3
4
a b c
a b c

+ +
. (1)
Thay v t cu hi khi no th du ng thc xy ra, ngi ta t
bi ton gii phng trnh

2 2 2
1 1 1 3
4
a b c
a b c

+ +
. (2)
44
Vai tr ca a, b, c u bnh ng nn trong (1) ta ngh n nh gi
2
1 1
4
a
a

.
Tht vy
2
1 1
4
a
a


2
1 1
0
4
a
a



2
2
( 2)
0
a
a

. Du ng thc
c khi v ch khi a = 2. Tng t ta cng c
2
1 1
4
b
b

,
2
1 1
4
c
c

. Du
ng thc c khi v ch khi b = 2, c = 2.
2) Mi gi tr ca bin cn bng bt ng thc c gi l im
ri ca bt ng thc y.
Theo , bt ng thc (1) cc bin a, b, c u c chung mt im
ri l a = b = c = 2.
Khi vai tr ca cc bin trong bi ton chng minh bt ng thc
bnh ng vi nhau th cc bin y c chung mt im ri.
Phng trnh din t du bng trong bt ng thc c gi l
"phng trnh im ri".
3) Phng trnh (2) thuc dng "phng trnh im ri"
Ti im ri a = b = c = 2 ta c
2 2 2
1 1 1 1
4
a b c
a b c


.
iu ct ngha im mu cht ca li gii l tch
3 1 1 1
4 4 4 4
+ +
:
(2)
2 2 2
1 1 1 1 1 1
0
4 4 4
a b c
a b c

_ _ _
+ +

, , ,
.
4) Phn ln cc phng trnh cha hai bin tr ln trong chng
trnh THCS u l "phng trnh im ri".
S 3
Cu 1: a) t x
2
= y, y 0. Khi phng trnh cho c dng: y
2
+ 3y 4
= 0 (1).
Phng trnh (1) c tng cc h s bng 0 nn (1) c hai nghim y
1
= 1; y
2
= -
4. Do y 0 nn ch c y
1
= 1 tha mn. Vi y
1
= 1 ta tnh c x = t 1. Vy
phng trnh c nghim l x = t 1.
b)
2x + y = 1 8x + 4y = 4 5x = 5 x = 1
3x + 4y = -1 3x + 4y = -1 2x + y = 1 y = - 1


' ' ' '

45
Cu 2:
( ) ( )
3 1 2 2 1 2
3 6 2 8
a) A = 3 2
1 2 1 2 1 2 1 2
+
+

+ +
1 1 x + 2 x
b) B = .
x 4
x + 4 x 4 x
_

+
,

( ) ( )
2
1 1 x( x + 2)
= .
( x 2) x
x 2 x 2
_


+
+
,

( ) ( )
x 2 x 2
1 1 4
=
x - 4 x - 4
x 2 x 2
+

+
Cu 3:
a) V th cc hm s y = - x
2
v y = x 2.
b) Honh giao im ca ng thng y = x
2 v parabol
y = - x
2
l nghim ca phng trnh:- x
2
= x
2

x
2
+ x 2 = 0
Suy ra cc giao im cn tm l: L( 1; -1 ) v
K ( - 2; - 4 )
(xem hnh v).
O
Cu 4:
a) T gic AEHF c:

0
AEH AFH 90
(gt). Suy ra AEHFl t gic ni tip.
- T gic BCEF c:

0
BEC BFC 90
(gt). Suy ra BCEF l t gic ni tip.
b) T gic BCEF ni tip suy ra:

BEF BCF
(1). Mt khc

BMN BCN
=

BCF

(gc ni tip cng chn

BN
) (2). T (1) v (2) suy ra:

BEF BMN

MN // EF.
c) Ta c:

ABM ACN
( do BCEF ni tip)

AM AN

AM = AN, li
c OM = ON nn suy ra OA l ng trung trc ca MN OA MN , m
MN song song vi EF nn suy ra OA EF .
Cu 5: K: y > 0 ; x R. Ta c: P =
2
x - x y + x + y - y + 1
( )
2
2
y 1
y 3y 3
= x - x( y - 1) + + - +
4 4 2 4

2
2
y 1 3 1 2 2
x - y
2 4 3 3 3
_

_
+ +



,
,
. Du = xy ra
- 1
x =
3
1
y =
9

'

.
46
Suy ra:
2
Min P =
3 .
S 4
Cu 1:
a)
( )
2
4 4 3 4 3
3
3
3

;
( )
( ) ( )
5 5 1
5
5 1
5 1 5 1
+

+
=
( )
2
5 5 5 5
4
5 1
+ +

.
b) Thay x = - 2 v y =
1
4
vo hm s y = ax
2
ta c:

2
1 1 1
a.(-2) 4a = a =
4 4 16

.
Cu 2:
( )
2
2
7 - x 0
x 7 (1)
a) 2x + 1 = 7 - x
x 16x + 48 = 0 2x + 1 = 7 - x


' '

Gii phng trnh: x


2
16x + 48 = 0 ta c hai nghim l 4 v 12. i chiu
vi iu kin (1) th ch c x = 4 l nghim ca phng trnh cho.
b)
1
2x + 3y = 2 10x = 5 x =
4x + 6y = 4
2
1 1
1 6x - 6y = 1 x - y = y = x -
y =
6 6
3



' ' ' '

.
Cu 3: a) Vi m = 3 ta c phng trnh: x
2
6x + 4 = 0.
Gii ra ta c hai nghim: x
1
=
2
3 5; x 3 5 + .
b) Ta c:
/
= m
2
4
Phng trnh (1) c nghim

/
m 2
0
m -2

(*).
Theo h thc Vi-t ta c: x
1
+ x
2
= 2m v x
1
x
2
= 4. Suy ra: ( x
1
+ 1 )
2
+ ( x
2
+ 1
)
2
= 2

x
1
2
+ 2x
1
+ x
2
2
+ 2x
2
= 0

(x
1
+ x
2
)
2
2x
1
x
2
+ 2(x
1
+ x
2
) = 0

4m
2
8 +
4m = 0

m
2
+ m 2 = 0

1
2
m 1
m 2

.
i chiu vi iu kin (*) ta thy ch c nghim m
2
= - 2 tha mn. Vy m =
- 2 l gi tr cn tm.
Cu 4:
47
a) T gic BIEM c:

0
IBM IEM 90
(gt); suy ra t gic BIEM ni tip
ng trn ng knh IM.
b) T gic BIEM ni tip suy ra:

0
IME IBE 45
(do ABCD l hnh vung).
c) EBI v ECM c:

0
IBE MCE 45
, BE = CE ,

BEI CEM
( do

0
IEM BEC 90
)

EBI = ECM (g-c-g)

MC =
IB; suy ra MB = IA
V CN // BA nn theo nh l Thalet,
ta c:
MA MB
MN MC

=
IA
IB
. Suy ra IM
song song vi BN
(nh l Thalet o)

0
BKE IME 45
(2). Li c

0
BCE 45
(do ABCD l hnh vung).
Suy ra

BKE BCE
BKCE l t
gic ni tip.
Suy ra:

0
BKC BEC 180 +
m

0
BEC 90
; suy ra

0
BKC 90
; hay CK BN .
I
E
M
N
B
C
A D
K
Cu 5:
Ta c: ( ) ( ) ( )
2 2 2
a - b b - c c - a 0 + + ( )
( )
2 2 2
2 a b c 2 ab + bc + ca + +

2 2 2
a b c ab + bc + ca + +
(1).
V a, b, c l di 3 cnh ca mt tam gic nn ta c: a
2
< a.(b+ c)

a
2
< ab
+ ac.
Tng t: b
2
< ab + bc; c
2
< ca + bc. Suy ra: a
2
+ b
2
+ c
2
< 2(ab + bc + ca) (2).
T (1) v (2) suy ra iu phi chng minh.
S 5
Cu 1: a)
3 2 3 2 3 2
. 6 . 6 . 6 .6 .6 3 2 1
2 3 2 3 2 3
_



,
48
b) V ng thng y = ax + b i qua im A(2; 3) nn thay x = 2 v y = 3 vo
phng trnh ng thng ta c: 3 = 2a + b (1). Tng t: 1 = -2a + b (2).
T ta c h:
1
2a + b = 3 2b = 4 a =
2
- 2a + b = 1 2a + b = 3
b = 2




' ' '

.
Cu 2: a) Gii phng trnh: x
2
3x + 1 = 0. Ta c: = 9 4 = 5
Phng trnh c hai nghim: x
1
=
3 5
2
+
; x
2
=
3 5
2

.
b) iu kin: x
t 1.
( ) ( )
2 2 2 2
x x + 1 - 2 x - 1
x - 2 4 4
+ = + =
x - 1 x + 1 x - 1 x - 1 x - 1 x - 1

x(x + 1) 2(x 1) = 4

x
2
x 2 = 0

1
2
x 1
x 2

.
i chiu vi iu kin suy ra phng trnh cho c nghim duy nht x = 2.
Cu 3: Gi vn tc ca t th nht l x (km/h). Suy ra vn tc ca t th
hai l: x 10 (km/h) (k: x > 10).
Thi gian t th nht v t th hai chy t A n B ln lt l
120
x
(h)
v
120
x - 10
(h).
Theo bi ra ta c phng trnh:
120 120
0, 4
x x - 10

Gii ra ta c x = 60 (tha mn).Vy vn tc ca t th nht l 60 km/h v
t th hai l 50 km/h.
Cu 4:
a) T gic ACBD c hai ng cho
AB v CD bng nhau v ct nhau ti
trung im ca mi ng, suy ra
ACBD l hnh ch nht
b) T gic ACBD l hnh ch nht suy
ra:
F
E
O
D
C
B
A

0
CAD BCE 90
(1). Li c
1
CBE
2

s

BC
(gc to bi tip tuyn v dy
cung);
1
ACD
2

s

AD
(gc ni tip), m

BC AD
(do BC = AD)

CBE ACD
(2). T (1) v (2) suy ra ACD ~ CBE .
49
c) V ACBD l hnh ch nht nn CB song song vi AF, suy ra:

CBE DFE
(3). T (2) v (3) suy ra

ACD DFE
do t gic CDFE ni tip c
ng trn.
d) Do CB // AF nn CBE ~ AFE, suy ra:
2
1
2
S EB
S EF

1
S EB
S EF
. Tng t ta c
2
S BF
S EF
. T suy ra:
1 2
S S
1
S S
+
1 2
S S S + .
Cu 5: k: x
3
+ 1 0 x -1 (1).
t: a =
x + 1
; b =
2
x - x + 1
,( a0; b>0) (2)

a
2
+ b
2
= x
2
+ 2.
Khi phng trnh cho tr thnh: 10.ab = 3.(a
2
+ b
2
)
( ) ( )
a - 3b 3a - b 0

a = 3b hoc b = 3a.
+) Nu a = 3b th t (2) suy ra:
x + 1
= 3
2
x - x + 1

9x
2
10x + 8 = 0
(v nghim).
+) Nu b = 3a th t (2) suy ra: 3
x + 1
=
2
x - x + 1

9x + 9 = x
2
x + 1

x
2
10x 8 = 0. Phng trnh c hai nghim x
1
= 5 33 + ; x
2
= 5 33
(tha mn (1)).
Vy phng trnh cho c hai nghim x
1
= 5 33 + v x
2
= 5 33 .
Li bnh:
Cu IV
1) chng minh ng thc (*) v din tch cc tam gic (chng
hn
1 2
S S S +
(*))
Bn c th ngh n mt trong ba cch sau :
Nu ba tam gic tng ng c mt cnh bng nhau th bin i (*)
v ng thc cc ng cao tng ng h
1
, h
2
, h chng minh (chng
hn(*) h
1
+ h
2
= h).
Nu ba tam gic tng ng c mt ng cao bng nhau th bin
i (*) v ng thc cc cnh tng ng a
1
, a
2
, a chng minh (chng
hn(*) a
1
+ a
2
= a).
Nu hai trng hp trn khng xy ra th bin i (*) v ng thc
t s din tch chng minh (chng hn(*)
1 2
1
S S
S S
+ ). Thng
ng thc v t s din tch tam gic l ng thc v t s cc cnh tng
ng trong cc cp tam gic ng dng.
50
2) Trong bi ton trn, hai kh nng u khng xy ra. iu dn
chng ta n li gii vi cc cp tam gic ng dng.
Cu V
cc bn c cch nhn khi qut, chng ti khai trin bi ton trn mt
bnh din mi.
Vit li
3
10 1 x +
= 3(x
2
+ 2)
2
10 ( 1)( 1) x x x + + = 3[(x + 1) + x
2

x + 1) (1)
Phng trnh (1) c dng .P(x) + .Q(x) +
. ( ) ( ) P x Q x
= 0 ( 0,
0, 0) (2)
(ph ng trnh ng c p i v i P(x) v Q(x)). t
( ) . ( ) Q x t P x , (3)
ph ng trnh (1) c a v t
2
+ t + = 0.
(4)
Sau khi tm c t t (4), th vo (3) tm x.
S 6
Cu 1:

( ) ( )
( ) ( )
3 3 1 3 3 1
3 3 3 3
a) A = 2 . 2 2 2
3 1 3 1 3 1 3 1
2 3 2 3 1.
_ _
+
_ _
+

+ +



+ +
, ,
, ,
+

( )
( ) ( )
( )
( )
b a b a
b) - . a b - b a - . ab a - b
a - ab ab - b
a a b b a b
b . ab a . ab
b - a. a > 0, b > 0, a b
a b
_
_





,
,

Cu 2:
a) k: x 0 v
y 0.
(*)
Rt y t phng trnh (1) ri th vo phng trnh (2) ta c:
2
2 3
2 2x 3x - 2 = 0
x x + 1
+
x 2
1
x
2

.
+ Vi x = 2, suy ra y = x + 1 = 3 (tho mn (*))
+ Vi x =
1
2

, suy ra y = x +1 =
1
2
(tho mn (*))
51
Vy h cho c hai nghim: (2; 3) v
1 1
;
2 2
_


,
.
b) Phng trnh x
2
x 3 = 0 c cc h s a, c tri du nn c hai nghim
phn bit x
1
; x
2
.
p dng h thc Vi-t, ta c: x
1
+ x
2
= 1 v x
1
x
2
= - 3.
Do : P = x
1
2
+ x
2
2

= (x
1
+ x
2
)
2
2x
1
x
2
= 1 + 6 = 7.
Cu 3:
a) Vit ng thng 2x + y = 3 v dng y = - 2x + 3.
V ng thng y = ax + b song song vi ng thng trn, suy ra a = - 2 (1)
V ng thng y = ax + b i qua im M (2;
1
2
) nn ta c:
1
2a + b
2

(2).
T (1) v (2) suy ra a = - 2 v b =
9
2
.
b) Gi cc kch thc ca hnh ch nht l x (cm) v y (cm)
( x; y > 0).
Theo bi ra ta c h phng trnh:
( ) ( )
xy = 40
xy = 40
x + 3 y + 3 xy + 48 x + y = 13

' '

.
Suy ra x, y l hai nghim ca phng trnh: t
2
13t + 40 = 0 (1).
Gii phng trnh (1) ta c hai nghim l 8 v 5.
Vy cc kch thc ca hnh ch nht l 8 cm v 5 cm.
Cu 4:
a) Ta c:

0
MAB 90
(gt)(1).

0
MNC 90
(gc ni tip
chn na ng trn)

0
MNB 90
(2)
T (1) v (2) suy ra ABNM l t gic ni tip.
Tng t, t gic ABCI c:

0
BAC BIC 90

ABCI l t gic ni tip ng trn.


I
N
M
C
B
A
b) T gic ABNM ni tip suy ra

MNA MBA
(gc ni tip cng chn cung
AM) (3).
T gic MNCI ni tip suy ra

MNI MCI
(gc ni tip cng chn cung MI) (4).
T gic ABCI ni tip suy ra

MBA MCI
(gc ni tip cng chn cung AI) (5).
T (3),(4),(5) suy ra

MNI MNA

NM l tia phn gic ca

ANI
.
52
c) BNM v BIC c chung gc B v

0
BNM BIC 90

BNM ~ BIC (g.g)
BN BI
BM BC

BM.BI = BN . BC .
Tng t ta c: CM.CA = CN.CB.
Suy ra: BM.BI + CM.CA = BC
2
(6).
p dng nh l Pitago cho tam gic ABC vung ti A ta c:
BC
2
= AB
2
+ AC
2
(7).
T (6) v (7) suy ra iu phi chng minh.
Cu 5: A = 2 - 2 - 2 3 + + x xy y x .
Trc ht ta thy biu thc A c ngha khi v ch khi:
0
0

'

x
xy
(1).
T (1) ta thy nu x = 0 th y nhn mi gi tr ty thuc R (2).
Mt khc, khi x = 0 th A = y + 3 m y c th nh ty nn A cng c th
nh ty . Vy biu thc A khng c gi tr nh nht.
Li bnh:
Cu IVc
a) Bit bao k c a v khi bt gp ng thc
BM . BI + CM . CA = AB
2
+ AC
2
. (1)
Phi chng
2
2
. (2)
. (3)
BM BI AB
CM CA AC

'

T cng theo tng v c (1).


Nu c (1) th AB phi l cnh chung mt cp tam gic ng dng. Tic
rng iu y khng ng. Tng t cng khng c (2).
AB
2
+ AC
2
= BC
2
v y nn (1) BM.BI +
CM.CA = BC
2
(3)
Kh nng
2
2
. .
. (1 )
BM BI k BC
CM CA k BC

'

(vi 0 < k < 1), t cng theo tng v


c (1) cng khng xy ra v BC khng phi l cnh chung ca mt cp
tam gic ng dng.
BN + NC = BC v y nn (1) BM.BI + CM.CA
= BC(BN + NC)


BM.BI + CM.CA = BC.BN +

BC.NC (4)
53
iu y dn dt chng ta n li gii trn.
b) Mong thi gian ng lng qun phn tch : PQ
2
= PQ(PK + KQ)
l mt cch chng minh ng thc dng : PX.PY + QM.QN = PQ
2
.
( y K l mt im thuc on thng PQ).
Cu V
Cnh bo. Cc bn cng theo di mt li gii sau :
Biu thc A c ngha khi v ch khi
0
0
x
y

'

. Bin i
( ) ( )
2 2
1 2 A x y x + + .
Suy ra minA = 2, t c khi x = y = 1 (!).
K t qu bi ton sai th r. Nh ng ci sai v t duy
m i ng bn h n.
1) iu kin xc nh ca P(x; y) cha ng thi x v xy l
0 0
0
x x
D
y y
>

' '


U

Do vy tm GTLN, GTNN P(x; y) cn phi xt c lp hai trng


hp
0 x
y

'


v
0
0
x
y
>

'


2) Khng th gp chung
0 0
0
x x
y y
>

' '


U

thnh
0
0
x
y

'

3) Do cho rng iu kin xc nh ca P(x; y) l


0
0
0
y
x
D
y

'

(b st
0
0
0
y
x
D
y
<

'
<

)
Vy nn A = 2 l GNNN ca A trn
0 y
D

, cha kt lun l
GTNN ca A trn D.
4) Nhn y lin tng n phng trnh ( ) ( ) 0 P x Q x . (1)
Bin i ng (1)
( ) 0
( ) 0
( ) 0
Q x
Q x
P x

>

'

. Cch bin i sau l sai (1)


( ) 0
( ) 0
Q x
P x

'

.
54
S 7
Cu 1: a) Biu thc A c ngha
- 1 0
1 3
3 - 0


'

x
x
x
.
b)
( ) ( ) ( ) ( )
1 1 3 5 5 1
3 5 5 1
3 5 3 5 5 1 5 1
+

+
+ +
=
( ) ( )
3 5 5 1
3 5 5 1
1
9 5 5 1 4
+
+


.
Cu 2: a) ( x 3 )
2
= 4

x 3 = 2
5
1

x
x
.
Vy phng trnh c 2 nghim x = 5; x = 1
b) k:
1
x
2

.
- 1 1 - 1 1 (2 - 2) - (2 1)
- 0 0
2 1 2 2 1 2 2(2 1)
+
< < <
+ + +
x x x x
x x x
( )
3 1
0 2x + 1 > 0 x > -
2 2x + 1 2

<
.
Cu 3: a) Ta c
/
= m
2
+ 1 > 0, m R. Do phng trnh (1) lun c hai
nghim phn bit.
b) Theo nh l Vi-t th: x
1
+ x
2
= 2m v x
1
.x
2
= - 1.
Ta c: x
1
2
+ x
2
2
x
1
x
2
= 7

(x
1
+ x
2
)
2
3x
1
.x
2
= 7

4m
2
+ 3 = 7

m
2
= 1

m = 1.
Cu 4:
a) SBC v SMA c:

BSC MSA
,

SCB SAM
(gc ni tip cng chn

MB
).
SBC SMA ~ .
b) V AB CD nn

AC AD
.
Suy ra

MHB MKB
(v cng
bng

1
(sdAD sdMB)
2
+
t
gic BMHK ni tip c ng
trn

0
HMB HKB 180 +
(1).
55
Li c:

0
HMB AMB 90
(2)
(gc ni tip chn na ng trn).
T (1) v (2) suy ra

0
HKB 90
, do HK // CD (cng vung gc vi AB).
c) V ng knh MN, suy ra

MB AN
.
Ta c:
1
OSM ASC
2

(s

AC
- s

BM
);
1
OMK NMD
2

s

ND
=
1
2
(s

AD
- s

AN
);
m

AC AD
v

MB AN
nn suy ra

OSM OMK
OSM OMK ~ (g.g)
2 2
OS OM
OK.OS = OM R
OM OK

.
Cu 5: Gii h phng trnh:
3
3
1 2 (1)
1 2 (2)
+

'
+

x y
y x
Ly pt (1) tr pt (2) ta c: x
3
y
3
= 2(y x)

(x y)(x
2
xy + y
2
+ 2) = 0

x y = 0

x = y.
( do x
2
xy + y
2
+ 2 =
2
2
y 3y
x - 2 0
2 4
_
+ + >

,
)
Vi x = y ta c phng trnh: x
3
2x + 1 = 0

(x 1)(x
2
+ x 1) = 0

-1+ 5 -1- 5
x = 1; x = ; x=
2 2
.
Vy h cho c 3 nghim l:
( )
1 5 1 5 1 5 1 5
1;1 , ; , ;
2 2 2 2
_ _
+ +


, ,
.
S 8
Cu 1:
2 5 6 3 15 7 14 2
a)
- 3 - 1 - 3 - 1 5 - 2 1
+ +


' ' ' '


x y x y x x
x y x y y x y
b) Phng trnh 3x
2
x 2 = 0 c cc h s a v c tri du nn lun c hai
nghim phn bit x
1
v x
2
.
Theo h thc Vi-t ta c: x
1
+ x
2
=
1
3
v x
1
.x
2
=
2
3

.
Do P =
2 1
1 2 1 2
1 1 1 2 1
:
3 3 2
+
_
+

,
x x
x x x x
.
56
Cu 2:

( )
a a a 1 a 1
a) A = : . a 1 a 1
a 1 a ( a - 1) ( a - 1)( a 1) a 1 ( a - 1)
_ _
+



+
, ,

b) A < 0
a > 0, a 1
0 a < 1
a 1

<
'
<

.
Cu 3: a) Vi m = 0 ta c phng trnh x
2
x + 1 = 0
V = - 3 < 0 nn phng trnh trn v nghim.
b) Ta c: = 1 4(1 + m) = -3 4m.
phng trnh c nghim th 0

- 3 4m0

4m
- 3
3 m
4

(1).
Theo h thc Vi-t ta c: x
1
+ x
2
= 1 v x
1
.x
2
= 1 + m
Thay vo ng thc: x
1
x
2
.( x
1
x
2
2 ) = 3( x
1
+ x
2
), ta c:
(1 + m)(1 + m 2) = 3

m
2
= 4

m = 2.
i chiu vi iu kin (1) suy ra ch c m = -2 tha mn.
Cu 4:
a) V MA, MC l tip tuyn nn:

0
MAO MCO 90

AMCO l t
gic ni tip ng trn ng knh MO.

0
ADB 90
(gc ni tip chn na ng
trn)

0
ADM 90
(1)
Li c: OA = OC = R; MA = MC (tnh
cht tip tuyn). Suy ra OM l ng
trung trc ca AC

0
AEM 90
(2).
x
N
I
H
E
D
M
C
O B
A
T (1) v (2) suy ra MADE l t gic ni tip ng trn ng knh MA.
b) T gic AMDE ni tip suy ra:

ADE AME AMO
(gc ni tip cng
chn cung AE) (3)
T gic AMCO ni tip suy ra:

AMO ACO
(gc ni tip cng chn cung
AO) (4).
T (3) v (4) suy ra

ADE ACO
c) Tia BC ct Ax ti N. Ta c

0
ACB 90
(gc ni tip chn na ng trn)

0
ACN 90
, suy ra ACN vung ti C. Li c MC = MA nn suy ra c
MC = MN, do MA = MN (5).
57
Mt khc ta c CH // NA (cng vung gc vi AB) nn theo nh l Ta-lt th
IC IH BI
MN MA BM
_


,
(6).
T (5) v (6) suy ra IC = IH hay MB i qua trung im ca CH.
Cu 5: V b, c [ ]
0;1
nn suy ra
2 3
b b; c c . Do :
a + b
2
+ c
3
ab bc ca a + b + c ab bc ca (1).
Li c: a + b + c ab bc ca = (a 1)(b 1)(c 1) abc + 1 (2)
V a, b, c [ ]
0 ; 1
nn (a 1)(b 1)(c 1) 0 ; abc0
Do t (2) suy ra a + b + c ab bc ca 1 (3).
T (1) v (3) suy ra a + b
2
+ c
3
ab bc ca 1.

S 9
Cu 1: a) Thay x = 3 2 + vo hm s ta c:
y =
( ) ( ) ( )
2
2
3 2 3 2 1 3 2 1 0 + + + .
b) ng thng y = 2x 1 ct trc honh ti im c honh x =
1
2
; cn
ng thng y = 3x + m ct trc honh ti im c honh x =
m
3

. Suy ra
hai ng thng ct nhau ti mt im trn trc honh
m 1 -3
m =
3 2 2

.
Cu 2: a) A =
3 x 6 x x - 9
:
x - 4
x 2 x 3
_
+
+



,
( ) ( )
( ) ( )
x 3 x 3
3( x 2) x
:
x 2 x 3
x 2 x 2
_
+
+

+


+
,
3 x 1 1
.
x 2 x 3 x 2
_
+



+
,
, vi
x 0, x 4, x 9
.
b) iu kin: x 3 v x - 2 (1).
2 2
2
x 3x 5 1 x 3x 5 x 2
(1) x 3x 5 x 2
(x 2)(x 3) x 3 (x 2)(x 3) (x 2)(x 3)
+ + +
+ +
+ + +

x
2
4x + 3 = 0. Gii ra ta c: x
1
= 1 (tha mn); x
2
= 3 (loi do (1)).
Vy phng trnh cho c nghim duy nht x = 1.
Cu 3: a) Thay m = 1 vo h cho ta c:
58
3x - y = 1 6x - 2y = 2 7x = 7 x = 1
x + 2y = 5 x + 2y = 5 x + 2y = 5 y = 2


' ' ' '

.
Vy phng trnh c nghim (1; 2).
b) Gii h cho theo m ta c:
3x - y = 2m - 1 6x - 2y = 4m - 2 7x = 7m x = m
x + 2y = 3m + 2 x + 2y = 3m + 2 x + 2y = 3m + 2 y = m + 1


' ' ' '

Nghim ca h cho tha mn x
2
+ y
2
= 10

m
2
+ (m + 1)
2
= 10

2m
2
+ 2m 9 = 0.
Gii ra ta c:
1 2
1 19 1 19
m ; m
2 2
+
.
Cu 4:
a) T gic ACNM c:

0
MNC 90
(gt)

0
MAC 90
( tnhcht tip tuyn).

ACNM l t gic ni tip ng trn ng knh MC. Tng t t gic


BDNM ni tip ng trn ng knh MD.
b) ANB v CMD c:

ABN CDM
(do t gic BDNM ni tip)

BAN DCM
(do t gic ACNM ni tip)

ANB ~ CMD (g.g)


c) ANB ~ CMD


CMD ANB
=
90
0
(do

ANB
l gc ni tip chn na
ng trn (O)).
Suy ra

0
IMK INK 90

IMKN l
t gic ni tip ng trn ng knh
IK

IKN IMN
(1).
T gic ACNM ni tip

IMN NAC
(gc ni tip cng
chn cung NC) (2).
K
I
y
x
D
C
N
M
O B
A
Li c:
1
NAC ABN (
2

s

AN
) (3).
T (1), (2), (3) suy ra

IKN ABN
IK // AB (pcm).
59
Cu 5: Ta c:
( ) ( ) ( ) ( )
a + b 2(a + b)
(1)
a 3a + b b 3b + a 4a 3a + b 4b 3b + a

+ +
p dng bt ng thc C-si cho cc s dng ta c:
( ) ( )
( ) ( )
4a + (3a + b) 7a + b
4a 3a + b 2
2 2
4b + (3b + a) 7b + a
4b 3b + a 3
2 2


T (2) v (3) suy ra: ( ) ( ) ( )
4a 3a + b 4b 3b + a 4a + 4b 4 +
T (1) v (4) suy ra:
( ) ( )
a + b 2(a + b) 1
4a + 4b 2
a 3a + b b 3b + a

+
. Du bng xy ra khi v ch khi a = b.
Li nhn
Cu V
Cc bn c s dng bt ng thc C-si lm ton nh mt nh
l (khng phi chng minh)
Bt ng thc C-si ch p dng cho cc s khng m. C th l :
+ Vi hai s a 0, b 0 ta c
2
a b
ab
+

, du ng thc c khi
v ch khi a = b.
+ Vi ba s a 0, b 0, c 0 ta c
3
3
a b c
abc
+ +

, du ng
thc c khi v ch khi a = b = c.
S 10
Cu 1:
( ) ( )
2
a) A = 3 8 50 2 1 6 2 5 2 2 1 = 2 2 1 1
b)
( )
2
2
2 2 2
x - 1 x - 1
2 x - 2x + 1 2 2
B = . .
x - 1 4x x - 1 2 x x - 1 2 x

V 0 < x < 1 nn ( )
x - 1 x - 1 ; x x
( )
( )
- 2 x - 1
1
B =
2x x - 1 x

.
60
Cu 2: a)
( )
2 x - 1 y = 3 2x y = 5 2x y = 5 x = 1
2x - 6y = - 16 7y = 21 y = 3
x - 3y = - 8
+ + +


' ' ' '

b)
x + 3 x 4 0
t
x
= t (t 0) (1)
Khi phng trnh cho tr thnh: t
2
+ 3t 4 = 0 (2)
Phng trnh (2) c tng cc h s bng 0; suy ra (2) c hai nghim: t
1
= 1
(tha mn (1)); t
2
= - 4 (loi do (1)).
Thay t
1
= 1 vo (1) suy ra x = 1 l nghim ca phng trnh cho.
Cu 3: Gi x l s sn phm loi I m x nghip sn xut c trong 1 gi(x
> 0).
Suy ra s sn phm loi II sn xut c trong mt gi l x + 10.
Thi gian sn xut 120 sn phm loi I l
120
x
(gi)
Thi gian sn xut 120 sn phm loi II l
120
x + 10
(gi)
Theo bi ra ta c phng trnh:
120 120
7
x x + 10
+
(1)
Gii phng trnh (1) ta c x
1
= 30 (tha mn); x
2
=
40
7

(loi).
Vy mi gi x nghip sn xut c 30 sn phm loi I v 40 sn phm loi
II.
Cu 4:
c) Ta c

0
CMA DNA 90
(gc ni tip chn na ng trn); suy ra
CM // DN hay CMND l hnh thang.
a) Ta c

ABC
v

ABD
ln
lt l cc gc ni tip chn
na ng trn (O) v (O
/
)

0
ABC ABD 90
Suy ra C, B, D thng hng.
b) Xt t gic CDEF c:

0
CFD CFA 90
(gc ni
tip chn na ng trn (O))

0
CED AED 90
(gc ni
tip chn na ng trn (O
/
)

0
CFD CED 90
suy ra
CDEF l t gic ni tip.
d
K
I
N
M
F
E
O
/
O
C
D
B
A
61
Gi I, K th t l trung im ca MN v CD. Khi IK l ng trung bnh
ca hnh thang CMND. Suy ra IK // CM // DN (1) v CM + DN = 2.IK (2)
T (1) suy ra IK MN

IK

KA (3) (KA l hng s do A v K c nh).


T (2) v (3) suy ra: CM + DN 2KA. Du = xy ra khi v ch khi IK =
AK

d AK ti A.
Vy khi ng thng d vung gc AK ti A th (CM + DN) t gi tr ln
nht bng 2KA.
Cu 5: Ta c:
( ) ( )
2 2
x + x 2011 y + y 2011 2011 + +
(1) (gt)
( ) ( )
2 2
x + x 2011 x - x 2011 2011 + +
(2)
( ) ( )
2 2
y + y 2011 y - y 2011 2011 + +
(3)
T (1) v (2) suy ra:
( ) ( )
2 2
y + y 2011 x - x 2011 + +
(4)
T (1) v (3) suy ra:
( ) ( )
2 2
x + x 2011 y - y 2011 + +
(5)
Cng (4) v (5) theo tng v v rt gn ta c:
x + y = - (x + y)

2(x + y) = 0

x + y = 0.
S 11
Cu 1: 1) Rt gn
A =
( ) ( )
( ) ( )
2
1 - a 1 + a + a
1 - a
+ a
1 - a
1 - a 1 + a
1 1
1 1
1 1
] ]
=
( )
( )
( )
( )
2
2 2
1 1
1 + 2 a + a . = 1 + a . = 1.
1 + a 1 + a
2) Gii phng trnh: 2x
2
- 5x + 3 = 0
Phng trnh c tng cc h s bng 0 nn phng trnh c 2 nghim phn
bit x
1
= 1, x
2
=
3
2
.
Cu 2: 1) Hm s nghch bin khi trn R khi v ch khi 3 - k < 0

k > 3
62
I
E
x
M
O
C
B
A
2) Gii h:
2
x =
4x + y = 5 8x +2y = 10 11x = - 2
11

3x - 2y = - 12 3x - 2y = -12 4x + y = 5 63
y =
11




' ' ' '

Cu 3: 1) Phng trnh c 2 nghim tri du khi: m < 0


2) Phng trnh c 2 nghim x
1
, x
2

= 9 - m 0

m 9
Theo h thcVit ta c
1 2
1 2
x + x = 6 (1)
x . x = m (2)

'

Theo yu cu ca bi ra x
1
- x
2
= 4 (3)
T (1) v (3)

x
1
= 5, thay vo (1)

x
2
= 1
Suy ra m = x
1
.x
2
= 5 (tho mn)
Vy m = 5 l gi tr cn tm.

Cu 4:
a) Ta c E l trung im ca AC

OE AC
hay

OEM
= 90
0
.
Ta c Bx AB

ABx
=90
0
.
nn t gic CBME ni tip.
b) V t gic OEMB ni tip



OMB = OEB
(cung chn

OB
),

EOM = EBM
(cng chn cung EM)
EIO ~ MIB (g.g)

IB.IE = M.IO
Cu 5: Ta c : P = 3x + 2y +
6 8 3 3 3 6 y 8
+ = ( x + y) + ( x + ) + ( + )
x y 2 2 2 x 2 y
Do ( )
3 3 3 3
x + y = x + y . 6 = 9.
2 2 2 2

3x 6 3x 6
+ 2 . = 6
2 x 2 x
,
y 8 y 8
+ 2 . = 4
2 y 2 y

Suy ra P 9 + 6 + 4 = 19
63
Du bng xy ra khi
x + y = 6
x = 2
3x 6
=
y = 4 2 x
y 8
=
2 y

' '

Vy min P = 19.
Li bnh:
Cu V
Vic tm GTNN ca biu thc P bao gi cng vn hnh theo s
"b dn": P B, (trong ti liu ny chng ti s dng B - ch ci u
ca ch b hn).
1) Do gi thit cho x + y 6, thun theo s "b dn": P B,
iu y mch bo ta biu th P theo (x + y). thc hin c iu y ta
phi kh
6
x
v
8
y
.
Do c x > 0; y > 0 nn vic kh c thc hin d dng bng cch
p dng bt ng thc C-si cho cc tng cp s Ax v
6
x
, By v
8
y
.
Bi l m li gii "kho lo" tch
3 3
3
2 2
x x x +
,
3 1
2
2 2
y y y +
.
2) Tuy nhin mu cht li gii nm s "kho lo" ni trn. Cc s
3
2
,
1
2
c ngh ra bng cch no?
Vi mi s thc a < 2, ta c

6 8
3 2 P x y
x y
+ + +
=
6 8
( ) (3 ) (2 ) a x y a x a y
x y
1
1
+ + + + +
1
1
]
]
(1)

6 2 6(3 ) 2 8(2 ) P a a a + +
(2)
Ta c
6
(3 ) 2 6(3 ) a x a
x
+
, du ng thc c khi
6
3
x
a

; (3)

8
(2 ) 2 8(2 ) a y a
y
+
, du ng thc c khi
8
2
y
a

. ; (4)
64
(2) tr thnh ng thc buc phi c x + y = 6
6 8
6
3 2 a a
+

(5)
Thy rng
3
2
a
l mt nghim ca (5). Thay
3
2
a
vo (2) ta c s
phn tch nh l i gi i trnh by. Cc s
3
2
,
1
2
c ngh
ra nh th .
3) Ph ng trnh (3) l ph ng trnh " k t i m r i ". Ng i
ta khng c n bi t ph ng trnh " k t i m r i " c bao
nhiu nghi m. Ch c n bi t (c th l on) c m t
nghi m c a n l cho l i gi i thnh cng. (Vi c gi i
ph ng trnh " k t i m r i " nhi u khi ph c t p v c ng
khng c n thi t.)
S 12
Cu 1: Rt gn biu thc
1) A = 20 - 45 + 3 18 + 72 = 5 . 4 - 9 . 5 + 3 9 . 2 + 36 . 2
= 2 5 - 3 5 + 9 2 + 6 2 = 15 2 - 5
2) B =
a + a a - a
1 + 1 +
a + 1 1 - a
_ _


, ,
vi a 0, a 1
=
a ( a + 1) a ( a - 1)
1 + 1 -
a + 1 a - 1
_ _


, ,
= (1 + a ) (1 - a ) = 1 - a
Cu 2: 1) th hm s i qua im M (- 2; -12) nn ta c: - 12 = a

. (- 2)
2

4a = -12

a = - 3. Khi hm s l y = - 3x
2
.
2) a) Vi m = 5 ta c phng trnh: x
2
+ 12x + 25 =0.
= 6
2
-25 = 36 - 25 = 11
x
1
=
- 6 - 11
; x
2
= - 6 + 11
b) Phng trnh c 2 nghim phn bit khi:
> 0

(m + 1)
2
- m
2
> 0

2m + 1 > 0

m >
- 1
2
(*)
Phng trnh c nghim x = - 2

4 - 4 (m + 1) + m
2
= 0

m
2
- 4m = 0


m = 0
m = 4

(tho mn iu kin (*))


Vy m = 0 hoc m = 4 l cc gi tr cn tm.
65
O
k
s
e
m
d
c
b
a
Cu 3:
Gi chiu di ca tha rung l x, chiu rng l y. (x, y > 0, x tnh bng m)
Din tch tha rung l x.y
Nu tng chiu di thm 2m, chiu rng thm 3 m th din tch tha
rung lc ny l: (x + 2) (y + 3)
Nu gim c chiu di v chiu rng 2m th din tch tha rung cn li l
(x-2) (y-2).
Theo bi ra ta c h phng trnh:

(x + 2) (y + 3) = xy + 100
(x - 2) (y - 2) = xy - 68

'

xy + 3x + 2y + 6 = xy + 100

xy - 2x - 2y + 4 = xy - 68

'

3x + 2y = 94 x = 22 x = 22

2x + 2y = 72 x + y = 36 y = 14


' ' '

.
Vy din tch tha rung l: S = 22 .14= 308 (m
2
).
Cu 4: 1) Ta c

0
BAC = 90 (gt)

0
MDC = 90
(gc ni tip chn na ng
trn)
A, D nhn BC di gc 90
0
, t gic
ABCD ni tip
V t gic ABCD ni tip.



ADB = ACB

(cng chn cung AB). (1)
Ta c t gic DMCS ni tip


ADB = ACS

(cng b vi

MDS
). (2)
T (1) v (2)

BCA = ACS
.
2) Gi s BA ct CD ti K. Ta c BD CK, CA BK.

M l trc tm KBC. Mt khc

MEC
= 90
0
(gc ni tip chn na
ng trn)

K, M, E thng hng, hay BA, EM, CD ng quy ti K.


3) V t gic ABCD ni tip



DAC = DBC
(cng chn

DC
). (3)
Mt khc t gic BAME ni tip



MAE = MBE
(cng chn

ME
). (4)
T (3) v (4)

DAM = MAE
hay AM l tia phn gic

DAE
.
Chng minh tng t:

ADM = MDE
hay DM l tia phn gic

ADE
.
Vy M l tm ng trn ni tip ADE.
Cu 5: Ta c: x
2
- 3x + 2 = (x - 1) (x - 2), x
2
+ 2x - 3 = (x - 1) (x + 3)
66
iu kin: x 2 (*)
Phng trnh cho
(x - 1) (x - 2) - (x - 1) (x + 3) + x + 3 - x - 2 = 0
x - 1 ( x - 2 - x + 3) - ( x - 2 - x + 3) = 0
( ) ( )
x - 2 - x + 3 x - 1 - 1 = 0
x - 2 = x + 3 (VN)
2
x - 1 - 1 = 0

x
(tho mn k (*))
Vy phng trnh cho c nghim duy nht l x = 2.
Li bnh:
Cu IVb
chng minh ba ng thng ng quy, mt phng php thng
dng l chng minh ba ng thng y hoc l ba ng cao, hoc l ba
ng trung tuyn, hoc l ba ng phn gic ca mt tam gic.
S 13
Cu 1:
1) iu kin: a 0, a 1, a 2
Ta c:
( ) ( )
( )
( ) ( )
( )
a - 1 a + a + 1 a + 1 a - a + 1
a + 2
P = - :
a - 2
a a - 1 a a + 1
1
1
1
]
a + a + 1 - a + a - 1 a + 2
= :
a - 2
a

2 (a - 2)
=
a + 2
2) Ta c: P =
2a - 4 2a + 4 - 8 8
= = 2 -
a + 2 a + 2 a + 2
P nhn gi tr nguyn khi v ch khi 8 M(a + 2)
a + 2 = 1 a = - 1; a = - 3
a + 2 = 2 a = 0 ; a = - 4

a + 2 = 4 a = 2 ; a = - 6
a + 2 = 8 a = 6 ; a = - 10
t


t


t

t

Cu 2:
1) ng thng i qua im M (1; -1) khi a + (2a - 1) . (- 1) + 3 = 0

a - 2a + 4 = 0

a = 4
Suy ra ng thng l 4x + 7y + 3 = 0
- 4 3
7y = - 4x - 3 y = x -
7 7

67
nn h s gc ca ng thng l
4
7

2) a) Phng trnh c nghim x = 0 nn: m + 1 = 0 m 1 .


b) Phng trnh c 2 nghim khi:
= m
2
- (m - 1) (m + 1) 0

m
2
- m
2
+ 1 0, ng m.
Ta c x
1
.x
2
= 5

m + 1
m - 1
= 5

m + 1 = 5m - 5
3
4m = 6 m =
2

.
Vi m =
3
2
ta c phng trnh :
1
2
x
2
- 3x +
5
= 0
2

x
2
- 6x + 5 = 0
Khi x
1
+ x
2
=
- b
= 6
a
Cu 3: H cho
4x + 7y = 18 25x = 25 x = 1

21x - 7y = 7 3x - y = 1 y = 2


' ' '

.
Cu 4:
1) Theo gi thit ta c:

1 2 3 4
B = B , B = B
M

0
1 2 3 4
B + B + B + B = 180

0
2 3
B B 90 +
Tng t

0
2 3
C + C = 90
Xt t gic BICK c
) )
0
B + C = 180

4 im B, I, C, K thuc ng
trn tm O ng knh IK.
2) Ni CK ta c OI = OC = OK (v
ICK vung ti C)

IOC cn ti O



OIC = ICO.
(1)
Ta li c

1 2
C = C (gt). Gi H l giao
im ca AI vi BC.
2
1
2
3
4 4
1
3
K
I
H
B
C
A
O
Ta c AH BC. (V ABC cn ti A).
Trong IHC c

0 0
HIC + ICH = 90 OCI + ICA = 90 .
Hay

0
ACO = 90
hay AC l tip tuyn ca ng trn tm (O).
3) Ta c BH = CH = 12 (cm).
Trong vung ACH c AH
2
= AC
2
- CH
2
= 20
2
- 12
2
= 256

AH = 16
Trong tam gic ACH, CI l phn gic gc C ta c:
68
IA AC AH - IH AC 20 5
= = = =
IH CH IH CH 12 3

(16 - IH) . 3 = 5 . IH

IH = 6
Trong vung ICH c IC
2
= IH
2
+ HC
2
= 6
2
+ 12
2
= 180
Trong vung ICK c IC
2
= IH . IK
2
IC 180
IK = = = 30
IH 6
, OI = OK = OC = 15 (cm)
Cu 5:
Ta c
2
x + x + 2010 = 2010 (1) iu kin: x - 2010
(1)
2
1 1
x + x + - x - 2010 + x + 2010 - = 0
4 4

2 2
1 1
x + - x +2010 - = 0
2 2
_ _


, ,

1 1
x + = x + 2010 - . (2)
2 2
1 1
x + = - x + 2010 + . (3)
2 2

Gii (2) : (2)


2
x 1 0
(x 1) x 2010 (4)
+

'
+ +

(4)

(x + 1)
2
= x + 2010

x
2
+ x - 2009 = 0
= 1 + 4 . 2009 = 8037
1 2
- 1 + 8037 -1 - 8037
x = ; x =
2 2
(loi)
Gii (3): (3)


2
2010 x 0
x x 2010
x x 2010 (5)

+
'
+

(5)
2
x x 2010 0 . = 1 + 4 . 2010 = 8041,
1 2
1 + 8041 1 - 8041
x = ; x =
2 2
(loi nghim x
1
)
Vy phng tnh c 2 nghim:
1 8037 1 8041
x ; x
2 2
+
.
Li bnh:
Cu V
Bng cch thm bt
1
( )
4
x +
, s nhy cm y trnh by li gii ngn
gn.
69
Khng cn mt s kho lo no c, bn cng c mt li gii trn tru
theo cch sau :
t 2010 x y + , y 0 bi ton c a v gii h
2
2
2010
2010
x y
y x
+

'
+

.
y l h phng trnh h i xng kiu 2 quen thuc bit cch gii.
Ch : Phng trnh cho c dng
(ax + b)
2
= ' ' p a x b + + qx + r , (a 0, a' 0, p 0)
t :
' ' , khi ' 0;
' ' , khi ' 0.
a x b ay b pa
a x b ay b pa

+ + >

+ + <

Thng phng trnh tr thnh h i xng kiu 2.


S 14
Cu 1: 1) Ta c :
x + 1 2 x 2 + 5 x
P = + -
x - 4
x - 2 x +2
P =
( x+1) ( x +2) + 2 x ( x - 2) - 2 - 5 x
( x - 2) ( x + 2)
=
=
x + 3 x +2 + 2x - 4 x - 2 - 5 x
( x +2) ( x - 2)
=
3x - 6 x 3 x ( x 2) 3 x
= =
( x + 2) ( x - 2) ( x + 2) ( x - 2) x +2

2) P = 2 khi
3 x
= 2 3 x = 2 x +4 x = 4 x = 16
x +2

Cu 2: 1) d song song vi trc Ox khi v ch khi
m 1 0 m 1
n 0 n 0

' '


.
2) T gi thit, ta c:
m 1 3 m 2
1 m 1 n n 2

' '
+

.
Vy ng thng d c phng trnh:
y 3x 2 +
Cu 3: 1) Vi m = - 3 ta c phng trnh: x
2
+ 8x = 0

x (x + 8) = 0

x = 0

x = - 8

2) Phng trnh (1) c 2 nghim khi:


0 (m - 1)
2
+ (m + 3) 0

m
2
- 2m + 1 + m + 3 0
70
o
2
o
1
o
e
f
h
c
b
a

m
2
- m + 4 > 0


2
1 15
(m ) 0
2 4
+ >
ng m
Chng t phng trnh c 2 nghim phn bit m
Theo h thc Vi t ta c:
1 2
1 2
x + x = 2(m - 1) (1)
x - x = - m - 3 (2)

'

Ta c
2 2
1 2
x + x
= 10

(x
1
+ x
2
)
2
- 2x
1
x
2
= 10

4 (m - 1)
2
+ 2 (m + 3) = 10

4m
2
- 6m + 10 = 10
m = 0
2m (2m - 3) = 0
3
m =
2

3) T (2) ta c m = -x
1
x
2
- 3 th vo (1) ta c:
x
1
+ x
2
= 2 (- x
1
x
2
- 3 - 1) = - 2x
1
x
2
- 8

x
1
+ x
2
+ 2x
1
x
2
+ 8 = 0
y l h thc lin h gia cc nghim khng ph thuc m.
Cu 4: 1) T gi thit suy ra

0 0
CFH = 90 , HEB = 90 . (gc ni tip
chn na ng trn)
Trong t gic AFHE c:
$
0
A = F = E = 90 AFHE
l hnh ch nht.
2) V AEHF l hnh ch nht

AEHF ni tip

AFE = AHE
(gc ni
tip chn

AE
) (1)
Ta li c

AHE = ABH
(gc c cnh tng ng ) (2)
T (1) v (2)

AFE = ABH
m
0
CFE + AFE = 180

0
CFE + ABH = 180 .
Vy t gic BEFC ni tip.
3) Gi O
1
, O
2
ln lt l tm ng trn ng knh HB v ng knh HC.
Gi O l giao im AH v EF. V AFHE l hnh ch nht. OF = OH FOH
cn ti O

OFH = OHF
. V CFH vung ti F

O
2
C = O
2
F = O
2
H


HO
2
F cn ti O
2
.

2 2
O FH = O HF m

0
2
O HF + FHA = 90 .

0
2
O FH + HFO = 90 . Vy EF l tip tuyn ca ng trn tm O
2
.
Chng minh tng t EF l tip tuyn ca ng trn tm O
1
.
Vy EF l tip tuyn chung ca 2 na ng trn.
Cu 5: Tm GTLN, GTNN ca x tho mn.
71
2 2 2 2
x + a +b + c = 7 (1)
x + a + b + c = 13 (2)

'

T (1)

a + b + c = 7 - x.. T (2)

a
2
+ b
2
+ c
2
= 13 - x
2
.
Ta chng minh: 3(a
2
+ b
2
+ c
2
) (a + b + c)
2
.

3a
2
+ 3b
2
+ 3c
2
- a
2
- b
2
- c
2
- 2ab - 2ac - 2bc 0

(a - b)
2
+ (b - c)
2
+ (c - a)
2
0 (pcm)
Suy ra 3 (13 - x
2
) (7 - x)
2
.

3 (13 - x
2
) 49 - 14x + x
2
.

4x
2
- 14x + 10 0

1 x
5
2
.
5 3
x khi a b c , x 1 khi a b c 2
2 2

.
Vy max x =
5
2
, min x = 1.
S 15
Cu 1: a) M =
x 1 1 2
- : +
x - 1
x - 1 x - x x + 1
_
_



,
,
=
( ) ( ) ( ) ( )
x 1 x - 1 2
- : +
x - 1 x ( x - 1)
x - 1 x + 1 x - 1 x +1
1
1
1
1
1
]
]
=
( ) ( ) ( ) ( )
( ) ( )
x - 1 x + 1
x - 1 x + 1 x - 1
: = .
x + 1
x x - 1 x - 1 x +1 x x - 1
=
x - 1
x
.
b) M > 0

x - 1 > 0 (v x > 0 nn
x
> 0)

x > 1. (tho mn)


Cu 2: a) Ta thy: a = 1; b = - 2m; c = - 1, r rng: a. c = 1 . (-1) = -1 < 0

phng trnh lun c hai nghim phn bit vi mi m


b) V phng trnh lun c 2 nghim phn bit. Theo h thc Vi-t, ta c:
72
a
h
b
o
n
m
k
1 2
1 2
b
x + x = - 2m
a

c
x . x = = - 1
a

'

do :
( )
2
2 2
1 2 1 2 1 2 1 2
x + x - x x = 7 x + x - 3x x = 7

(2m)
2
- 3 . ( -1) = 7

4m
2
= 4

m
2
= 1

m = t 1.
Cu 3: Gi x (chic) l s xe lc u (x nguyn, dng)
S xe lc sau l: x + 3 (chic)
Lc u mi xe ch:
480
x
(tn hng), sau mi xe ch:
480
x + 3
(tn hng)
Ta c phng trnh:
480 480
- = 8
x x +3

x
2
+ 3x - 180 = 0
Gii phng trnh ta c x
1
= - 15 (loi); x
2
= 12 (TMK)
Vy on xe lc u c 12 chic.
Cu 4: a)

AMB
= 90
0
(gc ni tip chn na
ng trn (O))

AM MB (1)
MN = BN (t/c 2 tip tuyn ct nhau), OM = OB

ON l ng trung trc ca on thng MB

ON MB (2)
T (1) v (2)

AM // ON

OAMN l hnh
thang.
b) NHK c HM NK; KB NH.
suy ra O l trc tm NHK

ON KH (3)
T (2) v (3)

KH // MB
Cu 5: 5x - 2
x
(2 + y) + y
2
+ 1 = 0 (1).
iu kin: x 0
t
x
= z, z

0, ta c phng trnh:
5z
2
- 2(2 + y)z + y
2
+ 1 = 0
Xem (2) l phng trnh bc hai n z th phng trnh c nghim khi 0
= (2 + y)
2
- 5(y
2
+ 1) = - (2y - 1)
2
0 vi y
phng trnh c nghim th = 0
1
y =
2

73
Th vo (1) ta tm c x =
1
4
. Vy x =
1
4
v
1
y =
2
l cc gi tr cn tm.
Li bnh:
Cu V
1) gii mt phng trnh cha hai n, ta xem mt trong hai n l
tham s. Gii phng trnh vi n cn li.
2) Cc bn tham kho thm mt li gii khc :
Ta c 5x 2 (2 ) x y + + y
2
+ 1 = 0 (4x 4 x + 1) + y
2
+ 2y x + x = 0

2 2
(2 1) ( ) 0 x y x + 2 1 0 x y x
1 1
( ; )
4 2
x y
.
Qua bi n i ta th y 5 x 2 (2 ) x y + + y
2
+ 1 0 v i m i y,
v i m i x > 0 .
Trnh by li gii ny chng ti mun nghim li Li bnh sau cu 5 2
rng: phn ln cc phng trnh cha hai bin tr ln trong chng
trnh THCS u l "phng trnh im ri". Bin i v tng cc biu
thc cng du l cch gii c trng ca "phng trnh im ri".
S 16
Cu 1:
1) K =
x x (2 x - 1)
-
x - 1 x ( x - 1)
=
x - 2 x + 1
= x - 1
x - 1
2) Khi x = 4 + 2 3 , ta c: K =
4 2 3 +
- 1 =
( )
2
3 +1 -1= 3 +1-1= 3
Cu 2:
1) ng thng y = ax + b song song vi ng thng y = 3x + 1 nn a = 3.
V ng thng y = ax + b i qua im M (-1;2) nn ta c:2 = 3.(-1) + b
b= 5 (t/m v b 1 )
Vy: a = 3, b = 5 l cc gi tr cn tm.
2) Gii h phng trnh:
3x + 2y = 6
x - 3y = 2

'

3 (3y + 2) + 2y = 6
x = 3y + 2

'

11y 0 x 2
x 3y 2 y 0



' '
+

.
74
q
o
p
e
d
c
b
a
Ba 3:
Gi x l s xe lc u ( x nguyn dng, chic)
S xe lc sau l : x+3 (chic)
Lc u mi xe ch :
96
x
(tn hng)
Lc sau mi xe ch :
96
x + 3
( tn hng)
Ta c phng trnh :
96
x
-
96
x + 3
= 1,6

x
2
+ 3x -180 = 0
Gii phng trnh ta c: x
1
= -15 ; x
2
=12.
Vy on xe lc u c: 12 (chic).
Cu 4:
1)

CDE
=
2
1
S

DC
=
2
1
S

BD = BCD

DE// BC (2 gc v tr so le trong)
2)

APC
=
2
1
s

(AC - DC) = AQC

T gic PACQ ni tip (v



APC = AQC)
3) T gic APQC ni tip

CPQ = CAQ (cng chn

CQ)

CAQ = CDE (cng chn

DC
)
Suy ra

CPQ = CDE DE // PQ
Ta c :
DE
PQ
=
CE
CQ
(v DE//PQ) (1) ,
DE
FC
=
QE
QC
(v DE// BC) (2)
Cng (1) v (2) :
DE DE CE + QE CQ
+ = = = 1
PQ FC CQ CQ

1 1 1
+ =
PQ FC DE

(3)
ED = EC (t/c tip tuyn); t (1) suy ra PQ = CQ
Thay vo (3) ta c :
1 1 1
+ =
CQ CF CE


Cu 5 : Ta c
a
a + b + c
<
a
b + a
<
a + c
a + b + c
(1)
b
a + b + c
<
b
b + c
<
b + a
a + b + c
(2)

c
a + b + c
<
c
c + a
<
c + b
a + b + c
(3)
75
Cng tng v (1), (2), (3), ta c : 1 <
a
a + b
+
b
b + c
+
c
c + a
< 2, pcm.
S 17
Cu 1:
A = x
1
.x
2
=
( ) ( ) ( )
2
2
3 + 5 . 3 - 5 = 3 + 5 3 - 5 = 3 - 5 = 9 - 5 = 4 = 2
B =
( ) ( )
2 2
2 2
1 2
x x = 3 + 5 + 3 - 5 = 3 + 5 + 3 - 5 = 6 +
Cu 2: a) m = - 2, phng trnh l: x
2
+ 3x - 6 = 0; = 33> 0, phng trnh
c hai nghim
phn bit x
1, 2
=
- 3 33
2
t
b) Ta c =
[ ]
2
2
- (2m +1 - 4 (m + 5m) = 4m
2
+ 4m + 1 - 4m
2
- 20m = 1 - 16m.
Phng trnh c hai nghim

1 - 16m 0
1
m
16

Khi h thc Vi-t ta c tch cc nghim l m
2
+ 5m.
M tch cc nghim bng 6, do m
2
+ 5m = 6

m
2
+ 5m - 6 = 0
Ta thy a + b + c = 1 + 5 + (-6) = 0 nn m
1
= 1; m
2
= - 6.
i chiu vi iu kin m
1
16
th m = - 6 l gi tr cn tm.
Cu 3: a) Khi m = - 2, ta c hai ng thng y = - x - 2 + 2 = - x v y =
(4 - 2)x + 1 = 2x + 1
Ta c to giao im ca 2 ng thng trn l nghim ca h
y = - x
y = 2x + 1

'

- x = 2x + 1
1
x = -
3

. T tnh c :
1
y
3

.
Vy ta giao im l A(
1 1
; )
3 3

.
b) Hai ng thng (d), ( d
) song song khi v ch khi
2
m = 1
m - 2 = - 1
m = 1
m - 1
m + 2 1
t


' '


Vy m = 1 th hai ng thng cho song song vi nhau..
76
e
h
t
k
o
d
c
b
a
Cu 4: a) Trong tam gic vung ATO c:
R
2
= OT
2
= OA . OH (H thc lng trong
tam gic vung)
b) Ta c

ATB = BCT
(cng chn cung TB)

BCT = BTH
(gc nhn c cnh tng ng
vung gc).

ATB = BTH
hay TB l tia phn gic
ca gc ATH.
c) Ta c ED // TC m TC TB nn ED TB. TED c TB va l ng cao
va l ng phn gic nn TED cn ti T.
d) BD // TC nn
HB BD BE
= =
HC TC TC
(v BD = BE) (1)
BE // TC nn
BE AB
=
TC AC
(2)
T (1) v (2) suy ra:
HB AB
=
HC AC
Cu 5: T gi thit: (x + y)
2
+ 7(x + y) + y
2
+ 10 = 0
( ) ( )
2 2
2
2
7 7 7
x +y + 2. x +y . + - + 10 = - y 0
2 2 2
_ _


, ,
2 2
7 9 7 9
x + y + - 0 x + y +
2 4 2 4
_ _


, ,
.
Gii ra c - 4 x + y + 1 - 1.
A = -1 khi x = - 2 v y = 0, A = - 4 khi x = -5 v y = 0.
Vy gi tr nh nht ca A l - 4 v gi tr ln nht ca A l - 1.
Li bnh:
Cu V
Bi ton cho c hai cch gii.
Cch 1. Bi n i gi thi t v d ng ( mA + n)
2
= k
2
[g(x,
y)]
2
, t m suy ra
(mA + n)
2
k
2
k n mA k + n minA, maxA.
Cch 2. T A = x + y +1 y = A x 1, th vo gi thit c phng trnh
bc hai i vi x. T 0 ta tm c minA, maxA .
S 18
Cu 1: Rt gn biu thc:
77
I
Q
O O'
F
H
P
E
D
C
B
A
1) 45 20 5 + =
2 2
3 .5 2 .5 5 +
= 3 5 2 5 5 + = 4 5
2)
4
2
x x x
x x
+
+
+
=
( 1) ( 2)( 2)
2
x x x x
x x
+ +
+
+
= 1 2 x x + + = 2 1 x
Cu 2: Gi x l chiu di, y l chiu rng ca hnh ch nht
(iu kin: x > 0, y > 0, x, y tnh bng mt)
Theo bi ra ta c: 2 (x + y) = 72

x +y = 36 (1)
Sau khi tng chiu di gp 3, chiu rng gp i, ta c :
2 (3 x + 2y) = 194

3x + 2y = 97 (2)
Ta c h PT :
x + y = 36
3x + 2y = 97

'

Gii h ta c:
x = 25
y = 11

'

i chiu iu kin bi ton ta thy x, y tha mn.


Vy din tch tha vn l: S = xy = 25.11 = 275 (m
2
)
Cu 3:
1) Khi m = 2, PT cho tr thnh: x
2
- 4x + 3 = 0
Ta thy: a +b + c = 1 - 4 +3 = 0
Vy PT cho c 2 nghim: x
1
= 1; x
2
= 3
2) iu kin phng trnh cho c nghim l:
, 2
b' - ac 0

2
2 (m 1) 0 +

3 - m 0

m 3 (1)
p dng h thc Vi t ta c :
1 2
1 2
x x 4
x x m 1
+

'
+

2 2
1 2
x + x
= 5 (x
1
+ x
2
)

(x
1
+ x
2
)
2
- 2x
1
x
2
= 5 (x
1
+ x
2
)

4
2
- 2 (m +1) = 5.4

2 (m + 1) = - 4

m = - 3
Kt hp vi iu kin (1) , ta c m = - 3
Cu 4 :
1. Ta c:

ABC
= 1v (gc ni tip chn
na ng trn)

ABF
= 1v (gc ni tip chn na ng
trn) nn B, C, F thng hng.. AB, CE v
DF l 3 ng cao ca tam gic ACF nn
chng ng quy.
2. Do

0
IEF IBF 90
suy ra BEIF ni
tip ng trn.
3. Gi H l giao im ca AB v PQ
Ta chng minh c cc tam gic AHP
78
v PHB ng dng
HP HA
HB HP

HP
2
= HA.HB
Tng t, HQ
2
= HA.HB. Vy HP = HQ hay H l trung im PQ.
Cu 5:
iu kin x

0 v 2 - x
2
> 0

0 v
x
<
2
(*)
t y =
2
2 - x
> 0
Ta c:
2 2
x + y = 2 (1)
1 1
2 (2)
x y

'
+

T (2) ta c : x + y = 2xy. Thay vo (1) C : xy = 1 hoc xy = -


1
2
* Nu xy = 1 th x + y = 2. Gii ra, ta c :
x 1
y 1

'

.
* Nu xy = -
1
2
th x + y = -1. Gii ra, ta c :
1 3 1 3
x x
2 2
; .
1 3 1 3
y y
2 2

+



' '
+




.
i chiu k (*), phng trnh cho c 2 nghim : x = 1 ; x

=
- 1 - 3
2
.
Li nhn .
Cu IV.1
Lin h vi li bnh sau cu 4b 12
S 19
Cu 1: a) A =
5 5 7 11 11 1
5 7 11
5 1 11
( ) ( )
.
+ +
+ + +
+
b) B =
5 5 11
5 5 11
5
( )
.
+
+ .
Vy A - B = 5 7 11 + + 5 11 = 7, pcm.
Cu 2: a) Vi m = 2 ta c h
79
y
x
m
p
q
b
a
3x + 2y = 5 y = 2x - 1 y = 2x - 1 x = 1

2x - y = 1 3x + 2(2x - 1) = 5 7x = 7 y = 1


' ' ' '

Vy h phng trnh c nghim (x; y) = (1; 1).
b) H c nghim duy nht khi:
3 m

m 1

m
2
- 3 vi mi m
Vy h phng trnh lun c nghim duy nht vi mi m.
Cu 3: Gi cnh gc vung nh l x.
Cnh gc vung ln l x + 2
iu kin: 0 < x < 10, x tnh bng m.
Theo nh l Pitago ta c phng trnh: x
2
+ (x + 2)
2
= 10
2
.
Gii phng trnh ta c x
1
= 6 (t/m), x
2
= - 8 (loi).
Vy cnh gc vung nh l 6m; cnh gc vung ln l 8m.
Cu 4: a) Ta c

0
PAC = 90

0
PAC + PMC = 180

nn t gic APMC ni tip
b) Do t gic APMC ni tip nn

MPC MAC (1)
D thy t gic BCMQ ni tip suy ra

MQC MBC (2)
Li c

0
MAC MBC 90 +
(3). T (1), (2), (3)
ta c :

0 0
MPC MBC 90 PCQ 90 + .
c) Ta c

BMQ = BCQ (T gic BCMQ ni tip)

BMQ = AMC (Cng ph
vi BMC)

EMC = EFC
(T gic CEMF ni tip). Nn

BCQ = EFC hay
AB // EF.
Cu 5: P = x
2
+ 1 +
2
1
x + 1

( )
2
2
1
2 x + 1
x + 1
, P = 2

x
2
+ 1 =
2
1
x + 1
x = 0. Vy min P = 2.
S 20
Cu 1: a)
( ) ( )
( )
2
2
2( 5 +2) - 2( 5 - 2) 2 5 +4 - 2 5 + 4 8
A = = = = 8
5 - 4
5 - 2 5 +2
5 - 2
.
80
b) Ta c:
( ) ( )
( )
( )
( )
( )
( )
( )
2
x - 1 x + 1 +1 - x x x +1
x - 1 x - 1
B = : =
x x x - 1 + 1 - x
x x +1
x - 1 x +1 x +1
=
x
x x - 1

Cu 2: x
2
- (m + 5)x - m + 6 = 0 (1)
a) Khi m = 1, ta c phng trnh x
2
- 6x + 5 = 0
a + b + c = 1 - 6 + 5 = 0

x
1
= 1; x
2
= 5
b) Phng trnh (1) c nghim x = - 2 khi:
(-2)
2
- (m + 5) . (-2) - m + 6 = 0

4 + 2m + 10 - m + 6 = 0

m = - 20
c) = (m + 5)
2
- 4(- m + 6) = m
2
+ 10m + 25 + 4m - 24 = m
2
+ 14m + 1
Phng trnh (1) c nghim khi = m
2
+ 14m + 1 0 (*)
Vi iu kin trn, p dng nh l Vi-t, ta c:
S = x
1
+ x
2
= m + 5; P = x
1
. x
2
= - m + 6. Khi :
2 2
1 2 1 2 1 2 1 2
x x x x 24 x x x x 24 ( ) + +
m 6 m 5 24 ( )( ) + +

2
m m 6 0 m 3 m 2 ; .
Gi tr m = 3 tho mn, m = - 2 khng tho mn iu kin. (*)
Vy m = 3 l gi tr cn tm.
Cu 3: Gi x l s dy gh trong phng lc u (x nguyn, x > 3)
x - 3 l s dy gh lc sau.
S ch ngi trn mi dy lc u:
360
x
(ch), s ch ngi trn mi dy lc
sau:
360
x - 3
(ch)
Ta c phng trnh:
360 360
- = 4
x - 3 x
Gii ra c x
1
= 18 (tha mn); x
2
= - 15 (loi)
Vy trong phng c 18 dy gh.
Cu 4: a) SAB cn ti S (v SA = SB - theo t/c 2 tip tuyn ct nhau)
nn tia phn gic SO cng l ng cao SO AB
81
b)

0
SHE = SIE = 90 IHSE
ni tip ng trn ng knh SE.
c) SOI ~ EOH (g.g)
OI SO
=
OH OE

OI . OE = OH . OS = R
2
(h thc lng trong tam gic vung SOB)
Cu 5: (1)

x
3
- 2mx
2
+ m
2
x + x - m = 0,

x (x
2
- 2mx + m
2
) + x - m = 0

x (x - m)
2
+ (x - m) = 0

(x - m) (x
2
- mx + 1) = 0
2
x = m

x - mx + 1 = 0 (2)


phng trnh cho c ba nghim phn bit th (2) c hai nghim phn
bit khc m.
D thy x = m khng l nghim ca (2). Vy (2) c hai nghim phn bit khi
v ch khi
= m
2
- 4 > 0
m > 2

m < - 2

.
Vy cc gi tr m cn tm l:
m > 2
m < - 2

.
S 21
Cu 1.
1) A =
( )
( )( )
( )
2
1 5
4
1 5 2
1 5 1 5
1 5 2
1 5
2 +

+
+

.
2) Ta c h

'



4
3 2
x y
x

'



2
11
2
3
y
x
.
Cu 2.
1) V th
2
x y thng qua bng gi tr
x -2 -1 0 1 2
y 4 1 0 1 4
V th
2 + x y
qua cc im A(0, 2) v B(-2,0).
82
-2 -1 1 2 3
-1
1
2
3
4
5
x
y
M
N
A
B
O
2) Phng trnh honh giao im ca hai th
2
2
+ x x hay 0 2
2
x x .
Phng trnh ny c nghim:
1 1
1 1
y x
v
4 2
2 2
y x
.
Vy hai th ct nhau ti hai im M(-1, 1) v N(2, 4).
Cu 3.
1) Vi 2 m , ta c phng trnh: 0 1 3 2
2
+ + x x . Cc h s ca phng
trnh tho mn 0 1 3 2 + + c b a nn phng trnh c cc nghim:
1
1
x
,
2
1
2
x
.
2) Phng trnh c bit thc ( ) ( ) ( ) 0 3 2 1 . 2 . 4 1 2
2 2
m m m nn
phng trnh lun c hai nghim
2 1
, x x
vi mi
m
.
Theo nh l Viet, ta c:

'

+
2
1
.
2
1 2
2 1
2 1
m
x x
m
x x
.
iu kin bi 1 4 2 4
2
2 2 1
2
1
+ + x x x x

( ) 1 6 4
2 1
2
2 1
+ x x x x . T ta
c: ( ) ( ) 1 1 3 2 1
2
m m

0 3 7 4
2
+ m m .
Phng trnh ny c tng cc h s
0 3 ) 7 ( 4 + + + + c b a
nn phng
trnh ny c cc nghim
4
3
, 1
2 1
m m
. Vy cc gi tr cn tm ca
m
l
4
3
, 1 m m
.
83
Cu 4. 1) T gic FCDE c 2 gc i :

o
FED FCD 90
(gc ni tip chn
na ng trn). Suy ra t gic FCDE ni tip.
2) Xt hai tam gic ACD v BED c:

0
90 ACD BED
,

ADC BDE
(i
nh) nn ACD BED. T ta c t
s :
. .
DC DE
DC DB DADE
DA DB

.
3) I l tm ng trn ngoi tip t gic
FCDE tam gic ICD cn

ICD IDC FEC
(chn cung

FC
).
Mt khc tam gic OBC cn nn

OCB OBC DEC
(chn cung

AC

ca (O)). T

0
90 ICO ICD DCO FEC DEC FED + +
IC CO hay IC l tip tuyn ca
ng trn (O).
Cu 5. t
2
1
28
9 4
+
+
y
x
,
2
1
y
ta c
4
1
28
9 4
2
+ +
+
y y
x

2
1
7 7
2
+ + x y y
.
Cng vi phng trnh ban u ta c h:

'

+ +
+ +
2
1
7 7
2
1
7 7
2
2
x y y
y x x
.
Tr v cho v ca hai phung trnh ta thu c
( ) ( ) x y y x y x + 7 7
2 2


( ) 0 8 7 7 ) ( + + y x y x

0 y x
(v
0 > x v
2
1
y
nn
) 0 8 7 7 > + + y x
hay
y x
.
Thay vo mt phng trnh trn ta c
0
2
1
6 7
2
+ x x

14
50 6
14
50 6
x
x
. i chiu vi iu kin ca x, y ta c nghim l
14
50 6 +
x .
84
D
O
F
B A
C
E
I
Li bnh:
Cu V
Chc chn s hi ng sau php t n ph
4 9 1
28 2
x
y
+
+ c s
"mch bo" no khng?
Ta c 7x
2
+ 7x =
4 9
28
x +

2
1 4 9 1
7
2 28 4
x
x
+
_
+ +

,

Di hnh thc mi phng trnh cho thuc dng
(ax + b)
2
= ' ' p a x b + + qx + r , (a 0, a' 0, p 0)
M t l n L i bnh sau cu 5 13 ch d n cch t
n ph nh trn.
S 22
Cu 1: 1) x
2
- 2x - 15 = 0 , ' = 1 - (-15) = 16 ,
'
= 4
Vy phng trnh c 2 nghim x
1
= 1 - 4 = - 3; x
2
= 1 + 4 = 5
2. ng thng y = ax - 1 i qua im M (- 1; 1) khi v ch khi: 1 = a (-1) -1
<=> a = - 2. Vy a = - 2
Cu 2: 1) P =
( ) ( ) ( ) ( )
( ) ( )
a a a 1 a a a 1
a 1
.
2 a
a 1 a 1
+ +

+

=
( ) ( )
a
a
a a
a a
a a a a a a a a a a a
2
2
. 4
) 1 ( 2
1

+
.
Vy P = - 2 a .
2) Ta c: P 2

- 2 a > - 2

a < 1

0 < a < 1
Kt hp vi iu kin P c ngha, ta c: 0 < a < 1
Vy P > -2 a khi v ch khi 0 < a < 1
Cu 3: Gi x, y s chi tit my ca t 1, t 2 sn xut trong thng ging (x, y

N
*
),
ta c x + y = 900 (1) (v thng ging 2 t sn xut c 900 chi tit). Do ci
tin k thut nn thng hai t 1 sn xut c: x + 15%x, t 2 sn xut c:
y + 10%y.
C hai t sn xut c: 1,15x + 1,10y = 1010 (2)
T (1), (2) ta c h phng trnh:
85
x y 900 1,1x 1,1y 990 0, 05x 20
1,15x 1,1y 1010 1,15x 1,1y 1010 x y 900
+ +


' ' '
+ + +

<=> x = 400 v y = 500 (tho mn)
Vy trong thng ging t 1 sn xut c 400 chi tit my, t 2 sn xut c
500 chi tit my.
Cu 4: 1) Ta c

IPC
= 90
0
(v gc ni tip
chn na ng trn) =>

CPK
= 90
0
.
Xt t gic CPKB c:

K B +
= 90
0
+ 90
0
= 180
0
=> CPKB l t gic ni tip ng trn (pcm)
2) Xt AIC vBCK c

A B
= 90
0
;

ACI BKC
(2 gc c cnh tng ng vung gc)
=> AIC ~ BCK (g.g) =>
BK
AC
BC
AI

=> AI.BK = AC.BC.


3) Ta c:

PAC PIC
(v 2 gc ni tip cng chn cung PC )

PBC PKC
(v 2 gc ni tip cng chn cung PC )
Suy ra

0
PAC PBC PIC PKC 90 + +
(v ICK vung ti C).=>

APB
= 90
0
.
Cu 5: Tm nghim nguyn ca phng trnh x
2
+ px + q = 0 bit p + q= 198.
Phng trnh c nghim khi 0 <=> p
2
+ 4q 0; gi x
1
, x
2
l 2 nghim.
- Khi theo h thc Vit c x
1
+ x
2
= - p v x
1
x
2
= q
m p + q = 198 => x
1
x
2
- (x
1
+ x
2
) = 198
<=> (x
1
- 1)(x
2
- 1) = 199 = 1 . 199 = (- 1)(-199) ( V x
1
, x
2

Z )
Nn ta c :
x
1
- 1 1 -1 199 -199
x
2
- 1 199 -199 1 -1
x
1
2 0 200 -198
x
2
200 -198 2 0
Vy phng trnh c cc nghim nguyn: (2; 200); (0; -198); (200; 2); (-198;
0)
S 23
Cu 1.
1) A = ( ) 5 . 80 5 3 20 + =
( )
2 5 3 5 4 5 . 5 3 5. 5 15 +
.
86
x
y
P
A
B
C
I
K
2) t
2
x t , 0 t phng trnh tr thnh 0 2 7 4
2
+ t t .
Bit thc 81 ) 2 .( 4 . 4 7
2

Phng trnh c nghim
4
1
1
t
,
2
2
t
(loi).
Vi
4
1
t
ta c
4
1
2
x

2
1
t x
. Vy phng trnh c nghim
2
1
t x
.
Cu 2.
1) Ta gi
) (
1
d
,
) (
2
d
ln lt l cc ng thng c phng trnh
6 3 + x y
v
1 2
2
5
+ m x y
. Giao im ca
) (
1
d
v trc honh l A(2,
0). Yu cu ca bi ton c tho mn khi v ch khi
) (
2
d
cng i qua A


1 2 2 .
2
5
0 + m
3 m .
2) Gi x l chiu rng ca hnh ch nht (n v m, x > 0)
chiu di ca hnh ch nht l x + 7 (m).
V ng cho l 13 (m) nn theo nh l Piatago ta c :
( )
2
2 2
13 x x 7 + +
2
2x 14x 49 169 + +


2
x 7x 60 0 +
x 5
x 12

. Ch c nghim x 5 tho mn.


Vy mnh t c chiu rng 5m, chiu di 12m v din tch l S = 5.12 = 60 (m
2
).
Cu 3.
1) Khi 3 m phng trnh tr thnh 0 2
2
x x
( ) 0 2 x x
0 x ;
2 x .
2) Phng trnh c hai nghim phn bit
2 1
, x x


( ) 0 3 1 ' > m
4 < m .
Khi theo nh l Vi-et ta c:
2
2 1
+ x x
(1) v
3
2 1
m x x
(2).
iu kin bi ton 12 2
2 1 2
2
1
+ x x x x


( ) 12 2
2 2 1 1
+ x x x x


12 2 2
2 1
x x
(do (1))
6
2 1
x x
(3).
T (1) v (3) ta c:
4 , 2
2 1
x x
. Thay vo (3) ta c:
( ) 3 4 . 2 m

5 m , tho mn iu kin.
Vy 5 m .
Cu 4.
1) Ta c

DAB
=
1
2
s

DB
(gc ni tip) v

BDE
=
1
2
s

DB
(gc gia tip
tuyn v dy cung). Suy ra

DAB BDE
.
87
2) Xt hai tam gic DMB v AMD c:

DMA
chung,

DAM BDM
nn
DMB AMD

MD MA
MB MD

hay
2
. MD MAMB .
Tng t ta cng c: EMB AME
ME MA
MB ME

hay
2
. ME MAMB .
T : MD = ME hay M l trung im ca DE.
3) Ta c

DAB BDM
,

EAB BEM



PAQ PBQ + =

0
180 DAB EAB PBQ BDM BEM DBE + + + +
t gic APBQ ni tip

PQB PAB . Kt hp vi

PAB BDM
suy ra

PQB BDM . Hai gc ny v tr so le trong nn PQ song song vi AB.
A
B
O
O'
M
D
E
P
Q
Cu 5. t
1
3 4
2
+
+

x
x
y
.
Khi ta c ( ) 3 4 1
2
+ + x x y

( ) 0 3 4 .
2
+ y x x y (1).
Ta tm iu kin ca y (1) c nghim.
Nu
0 y
th (1) c nghim
3
4
x
.
Nu
0 y
, (1) c nghim

( ) 0 3 2 '
2
y y

0 4 3
2
y y

4 1 y
.
Kt hp li th (1) c nghim
4 1 y
.
Theo gi thit
y
l s nguyn m
1 y
. Khi thay vo trn ta c
2 x .
Li bnh:
88
Cu V
1) T cch gii bi ton trn ta suy biu thc
2
4 3
1
x
y
x
+

+
c GTNN
bng 1 v GTLN bng 4.
2) Phng php gii bi ton trn cng l phng phng php tm
GTNN, GTLN ca cc biu thc dng
2
2
' ' '
ax bx c
P
a x b x c
+ +

+ +
(vi b'
2
4ac
< 0), chng hn
2
2
20 10 3
3 2 1
x x
P
x x
+ +

+ +
;
2 2
2 2
8 7 x xy y
Q
x y
+

+
vi x
2
+ y
2
> 0;
F = x
2
+ 2xy y
2
vi 4x
2
+ 2xy + y
2
= 3.
S 24
Cu 1.
1) A =
5(1 5) (1 5) 1 5
(1 5) (1 5) 2
2 2
2 5
+ +
.
2) B =
( ) ( )
( ) ( )
1 1
1 1 1 1 1
1 1
x x x x
x x x
x x
_ _
+

+ + +

+
, ,
.
Cu 2.
1) Thay 2 x vo v tri ca phng trnh ta c:
( )
2
2 3 .2 2( 5) 4 6 2 2 10 0 m m m m + + + +
ng vi mi m
nn phng trnh c nghim 2 x vi mi m
2) V phng trnh lun c nghim 2 x nn n c nghim
2 2 5 x

th theo nh l Vi-et ta c: ( ) ( ) 5 2 2 2 5 2 m


5 2 2 5 m


2 2 10 m
.
Cu 3.
Gi x (km/h) l vn tc d nh ca xe, x > 15.
Thi gian d nh ca xe l
80
x
.
Thi gian xe i trong mt phn t qung ng u l
20
15 x
, thi gian xe i
trong qung ng cn li l
60
10 x +
.
89
Theo bi ra ta c
80
x
=
20
15 x
+
60
10 x +
(1).
Bin i (1)

4 1 3
15 10 x x x
+
+

( ) ( ) ( )
4 15 10 4 35 x x x x +

15 600 x

x = 40 (tho mn iu kin).
T thi gian d nh ca xe l
80
2
40

gi.
Cu 4.
1) Ta c v Ax l tip tuyn ca na ng trn nn

0
90 MAD
. Mt khc
theo gi thit

0
90 MCD
nn suy ra t gic ADCM ni tip.
Tng t, t gic BDCN cng ni tip.
2) Theo cu trn v cc t gic ADCM v BDCN ni tip nn:

DMC DAC
,

DNC DBC
.
Suy ra

0
90 DMC DNC DAC DBC + +
. T

0
90 MDN
.
3) V

0
90 ACB MDN
nn t gic CPDQ ni tip. Do

CPQ CDQ CDN .
Li do t gic CDBN ni tip nn

CDN CBN
. Hn na ta c

CBN CAB
, suy ra

CPQ CAB hay PQ song song vi AB.
Cu 5. Vi cc s dng x, y ta c: ( )
2
4 x y xy +
4 x y
xy x y
+

+

1 1 4
x y x y
+
+
p dng bt ng thc trn ta, c:
1 1 1 1 1 1 a b b c c a
a b c
c a b b c c a a b
+ + +
_ _ _
+ + + + + + +

, , ,
90
4 4 4
. . . a b c
b c c a a b
+ +
+ + +
=
4
a b c
b c c a a b
_
+ +

+ + +
,
Vy bt ng thc c chng minh.
Li bnh:
Cu II.1
Thay cu II.1 bi cu : Chng minh phng trnh c nghim khng
ph thuc gi tr ca m, ta c mt bi ton "thng minh hn".
Bi n i ph ng trnh v d ng m(x 2) = x
2
+ 3x 10 .
(1)
Xem (1) l phng trnh i vi m. Th th (1) c nghim khng ph
thuc m khi v ch khi x 2 = x
2
+ 3x 10 = 0 x = 2.
Vy c x = 2 l nghim c nh khng ph thuc vo m ca phng
trnh cho.
Vn nghim c nh cn c bn thm li bnh sau cu Cu
I4b, 32.
S 25
Cu 1.
1) Ta c A =
( )
1 1
:
1
1
x x
x
x x
_
_
+




,
,
=
1 1 1
.
1
x x x
x x x
+

+
.
2)
2 2 3 x +

( )
2
2 1 x +

2 1 x + nn A =
2 2 2
2
2 1
+

+
.
Cu 2. 1) Khi 3 a v 5 b ta c phng trnh:
0 4 3
2
+ x x
. Do a + b
+ c = 0 nn
phng trnh c nghim
4 , 1
2 1
x x
.
2) Phng trnh c hai nghim phn bit
2 1
, x x


2
4( 1) 0 a b + > (*)
Khi theo nh l Vi-et, ta c
1 2
1 2
1
x x a
x x b
+

'
+

(1).
Bi ton yu cu

'



9
3
3
2
3
1
2 1
x x
x x

( ) ( )
1 2
3
1 2 1 2 1 2
x x 3
x x 3x x x x 9

'
+

'



2
3
2 1
2 1
x x
x x
(2).
91
T h (2) ta c: ( ) ( )
2 2
2
1 2 1 2 1 2
4 3 4( 2) 1 x x x x x x + + + , kt hp vi
(1) c
2
1
1 2
a
b

'
+

1, 3
1, 3
a b
a b

.
Cc gi tr ny u tho mn iu kin (*) nn chng l cc gi tr cn tm.
Cu 3.
Gi x (km/h) l vn tc thc ca chic thuyn (x > 4).
Vn tc ca chic thuyn khi xui dng l x + 4 (km/m).
Vn tc ca chic thuyn khi ngc dng l x 4 km.
Thi gian chic thuyn i t A n B l
24
4 x +
.
Thi gian chic thuyn quay v t B n C l
16
4 x
.
Thi gian chic b i c
8
2
4

(gi).
Ta c phng trnh:
24
4 x +
+
16
4 x
= 2 (1).
Bin i phng trnh: (1)
( ) ( )
12( 4) 8( 4) 4 4 x x x x + + +

2
20 0 x x

( 20) 0 x x

0
20
x
x

.
i chiu vi iu kin ta thy ch c nghim x = 20 tho mn. Vy vn tc
thc ca chic thuyn l 20km/h.
Cu 4.
1) V H l trung im ca AB nn OH AB hay

0
90 OHM
. Theo tnh
cht ca tip tuyn ta li c OD DM hay

0
90 ODM
. Suy ra cc im M,
D, O, H cng nm trn mt ng trn.
2) Theo tnh cht tip tuyn, ta c MC = MD MCD cn ti M MI l
mt ng phn gic ca

CMD
. Mt khc I l im chnh gia cung nh

CD
nn

1
2
DCI
s

DI
=
1
2
s

CI
=

MCI

CI l phn gic ca

MCD
. Vy I l tm ng trn ni tip tam gic
MCD.
92
3) Ta c tam gic MPQ cn M, c MO l ng cao nn din tch ca n
c tnh:
1
2 2. . . ( )
2
OQM
S S ODQM R MD DQ +
. T S nh nht
MD + DQ nh nht. Mt khc, theo h thc lng trong tam gic vung
OMQ ta c
2 2
. DM DQ OD R khng i nn MD + DQ nh nht DM
= DQ = R. Khi OM =
2 R
hay M l giao im ca d vi ng trn tm
O bn knh
2 R
.
d
I
B
A
O
M
C
D
H
Q
P
Cu 5.
T gi thit ta c: ( )
1 abc a b c + +
. Do , p dng bt ng thc Csi,
P = ( ) ( )
a b a c + +
=
2
a ab ac bc + + + = ( )
a a b c bc + + +

( )
2 a a b c bc + +
= 2.
ng thc xy ra
( )
1
a a b c bc
a b c
abc
+ +

'
+ +


( )
1
1
a a b c
bc
+ +

'

.
H ny c v s nghim dng, chng hn ta chn b = c = 1 a = 2 1 .
Vy gi tr nh nht ca biu thc P l 2.
S 26
Cu 1:
93
1)
( ) ( )
( ) ( )
2 5 2 5
1 1 2 5
2 5
1
2 5 2 5
2 5 2 5
+

+
+
2)
3x + y = 9 6x + 2y = 18 7x = 14 x = 2
x - 2y = - 4 x - 2y = - 4 y = 9 - 3x y = 3


' ' ' '

.
Cu 2:
1)
1 1 x
P = :
x + x x 1 x + 2 x 1
_


+ +
,

( ) ( )
( )
2
x 1
1 x
.
x
x x 1 x x 1
_
+



+ +
,

( )
( ) ( ) ( )
2
x 1 1 x x 1
1 x 1 - x
.
x
x x. x
x x 1
+ +


+
.
2) Vi x > 0 th ( )
1 - x 1
2 1 - x x
x 2
> >

2
3x > - 2 x <
3

.
Vy vi
2
0 x <
3
<
th P >
1
2
.
Cu 3:
1) Vi m = 1, ta c phng trnh: x
2
x + 1 = 0
V = - 3 < 0 nn phng trnh trn v nghim.
2) Ta c: = 1 4m. phng trnh c nghim th 0

1 4m0

1
m
4

(1).
Theo h thc Vi-t ta c: x
1
+ x
2
= 1 v x
1
.x
2
= m
Thay vo ng thc: ( x
1
x
2
1 )
2
= 9( x
1
+ x
2
), ta c:
(m 1)
2
= 9

m
2
2m 8 = 0

m = - 2
.
m = 4

.
i chiu vi iu kin (1) suy ra ch c m = -2 tha mn.
Cu 4:
1) T gic ABEH c:

0
B = 90
(gc ni tip trong na ng trn);

0
H = 90

(gi thit)
nn t gic ABEH ni tip c.
Tng t, t gic DCEH c

0
C = H = 90
, nn ni tip c.
2) Trong t gic ni tip ABEH, ta c:
94

EBH = EAH
(cng chn cung

EH
)
Trong (O) ta c:

EAH = CAD = CBD

(cng chn cung

CD
).
Suy ra:

EBH = EBC
, nn BE l tia phn
gic ca gc

HBC
.
Tng t, ta c:

ECH = BDA = BCE
, nn
CE l tia phn gic ca gc

BCH
.
Vy E l tm ng trn ni tip tam gic
BCH.
B
II
OO
HH
EE
DD
CC
BB
A
3) Ta c I l tm ca ng trn ngoi tip tam gic vung ECD, nn

BIC = 2EDC
(gc ni tip v gc tm cng chn cung

EC
). M

EDC = EHC
, suy ra

BIC = BHC
.
+ Trong (O),

BOC = 2BDC = BHC
(gc ni tip v gc tm cng chn
cung

BC
).
+ Suy ra: H, O, I trn cung cha gc

BHC
dng trn on BC, hay 5
im B, C, H, O, I cng nm trn mt ng trn.
Cu 5: K: x - 3 (1)
t ( )
x + 8 a; x + 3 b a 0; b 0
(2)
Ta c: a
2
b
2
= 5; ( ) ( )
2
x 11x + 24 x + 8 x + 3 ab +
Thay vo phng trnh cho ta c:
(a b)(ab + 1) = a
2
b
2

(a b)(1 a)(1 b) = 0
x + 8 x + 3 (vn)
a - b = 0
x = - 7
1 - a = 0 x + 8 1
x = - 2
1 - b = 0
x + 3 1

i chiu vi (1) suy ra phng trnh cho c nghim duy nht x = - 2.


S 27
Cu 1:
1) A =
1 2
4.5 16.5 9.5
2 3
+
= 5 4 5 2 5 + = 5 .
2)
5 5 5 5
B = 2 . 2
5 1 5 1
_ _
+
+


+
, ,

95

( ) ( )
( ) ( )
5 5 1 5 5 1
2 2 2 5 2 5 1
5 1 5 1
_ _
+

+ +

+
, ,
Cu 2:
1)
2x - y = 1 - 2y 2x + y = 1 2x = 2 x = 1
3x + y = 3 - x 4x + y = 3 y = 1 - 2x y = - 1


' ' ' '

2) Phng trnh x
2
x 3 = 0 c a, c tri du nn c hai nghim phn bit x
1
; x
2
.
p dng h thc Vi-t, ta c: x
1
+ x
2
= 1 v x
1
x
2
= - 3.
Do : P =
1 2
1 2 1 2
x x 1 1 1 1
x x x x 3 3
+
+

.
Cu 3: Gi x (km/h) l vn tc ca xe la th nht i t Hu n H Ni.
Khi vn tc ca xe la th hai i t H Ni l: x + 5 (km/h) (K: x > 0)
Theo gi thit, ta c phng trnh:
300 5 345
5 3 x x
+
+
( ) ( )
2
900 5 5 1035 5 22 1035 0 x x x x x x + + +
Gii phng trnh ta c:
1
23 x
(loi v x > 0) v
2
45 0 x >
.
Vy vn tc xe la th nht l: 45 km/h v vn tc xe la th hai l: 50 km/h
Cu 4:
1) Ta c:

0
AMB 90
(gc ni
tip chn na ng trn)

0
AMD 90
. T gic
ACMD
c

0
AMD ACD 90
, suy ra
ACMD ni tip ng trn
ng knh AD.
2) ABD v MBC c:

B
chung v

BAD BMC
(do
ACMD l t gic ni tip).
Suy ra: ABD ~ MBC (g g)
S
EE
DD
MM
II
CC
KK
OO
BB
A
3) Ly E i xng vi B qua C th E c nh v

EDC BDC
, li c:

BDC CAK
(cng ph vi

B
), suy ra:

EDC CAK
. Do AKDE l t
gic ni tip. Gi O l tm ng trn ngoi tip AKD th O cng l tm
ng trn ngoi tip t gic AKDE nn O
A = O
E, suy ra O
thuc
ng trung trc ca on thng AE c nh.
96
Cu 5:
A =
2 2
1 1
x y xy
+
+
=
2 2
1 1 1
x y 2xy 2xy
+ +
+
p dng bt ng thc Csi cho hai s dng ta c:
1
x + y 2 xy 1 2 xy 1 4xy 2
2xy

(1)
ng thc xy ra khi x = y.
Tng t vi a, b dng ta c:
1 1 1 2 4
2 2.
a b ab a + b a + b
+ (*)
p dng bt ng thc (*) ta c:
( )
2 2 2
1 1 4
4
x y 2xy
x + y
+
+
(2)
Du ng thc xy ra khi x
2
+ y
2
= 2xy

x = y.
T (1) v (2) suy ra: A 6 . Du "=" xy ra
1
x = y =
2
. Vy minA = 6.
S 28
Cu 1:
1)
2x + y = 7 6x + 3y = 21 7x = 14 x = 2
x - 3y = - 7 x - 3y = - 7 y = 7 - 2x y = 3


' ' ' '

2) Phng trnh 3x
2
x 2 = 0 c cc h s a v c tri du nn lun c hai
nghim phn bit x
1
v x
2
.
Theo h thc Vi-t ta c: x
1
+ x
2
=
1
3
v x
1
.x
2
=
2
3

.
Do P =
( )
2
2 2
1 2 1 2 1 2
x x x x 2x x + + =
1 4 13
9 3 9
+ .
Cu 2.
1)
a a a 1
A = :
a 1 a ( a + 1) ( a - 1)( a 1)
_



+ +
,

( )
a 1
. a 1 a 1
a 1 a + 1
_
+


+
,
97
2) A < 0
a > 0, a 1
0 a < 1
a 1

<
'
<

.
Cu 3:
1) Ta c
= m
2
+ 1 > 0, m R. Do phng trnh (1) lun c hai
nghim phn bit.
2) Theo nh l Vi-t th: x
1
+ x
2
= 2m v x
1
.x
2
= - 1. Ta c: x
1
2
+ x
2
2
x
1
x
2
= 7

(x
1
+ x
2
)
2
3x
1
.x
2
= 7

4m
2
+ 3 = 7

m
2
= 1

m = 1 t .
Cu 4:
1)

0
ADB 90
(gc ni tip chn na
ng trn)

0
ADM 90
(1)
Li c: OA = OC = R; MA = MC (tnh
cht tip tuyn). Suy ra OM l ng
trung trc ca AC

0
AEM 90
(2).
T (1) v (2) suy ra MADE l t gic
ni tip ng trn ng knh MA.
x
N
I
H
E
D
M
C
O B
A
2) Xt MAB vung ti A c AD MB, suy ra: MA
2
= MB.MD (h thc
lng trong tam gic vung)
3) Ko di BC ct Ax ti N, ta c

0
ACB 90
(gc ni tip chn na ng
trn)

0
ACN 90
, suy ra ACN vung ti C. Li c MC = MA nn suy ra
c MC = MN, do MA = MN (5).
Mt khc ta c CH // NA (cng vung gc vi AB) nn theo nh l Ta-lt th
IC IH BI
MN MA BM
_


,
(6) vi I l giao im ca CH v MB.
T (5) v (6) suy ra IC = IH hay MB i qua trung im ca CH.
Cu 5: iu kin:
1 5
0, - 0, 2 - 0. x x x
x x
(*)
4 1 5 4 1 5
- 2 - - - - 2 - + + x x x x x x
x x x x x x
98
4
4 4 1
- - 1 0
1 5 1 5
- 2 - - 2 -
_


_

+


,
+ +

,
x
x
x x
x x
x x x x
x x x x

4
- 0 x
x
(v
1
1 0
1 5
- 2 -
+ >
+ x x
x x
)
2 t x .
i chiu vi iu kin (*) th ch c x = 2 tha mn.
S 29
Cu 1: a) ng thng d i qua gc ta khi v ch khi
2m 4 0 m 2.
b) th hm s
2 2
y m m x ( ) i qua im A(-1; 2)
2 2
2 m m 1 ( ).( )
2
m m 2 0 m 1 m 2 ;
Cu 2:
a) P =
( )( ) a
a
a a
a a
a a a
3
.
3 3
3 3 3
1 .
3
1
3
1
+
+ +

,
_

,
_

+
+

.
=
3
2
). 3 )( 3 (
) 3 .( 2
+

a a a a
a a
. Vy P =
3
2
+ a
.
b) Ta c:
3
2
+ a
>
2
1

a + 3 < 4

a < 1 0 a 1. < < .


Vy P >
2
1
khi v ch khi 0 < a < 1.
Cu 3: Gi x, y l thi gian mi ngi cn mt mnh hon thnh cng
vic (x, y > 0 tnh bng gi). Trong 1 gi mi ngi lm c
x
1
;
y
1
cng
vic, c 2 lm trong 1 gi c
x
1
+
y
1
=
4
1
cng vic.(v hai ngi hon
thnh cng vic trong 4 gi). Do ngi th nht lm t hn ngi th hai l 6
gi nn y - x = 6.
Ta c h phng trnh.
99
y x 6
y x 6 (1)
1 1 1
1 1 1
(2)
x y 4
x x 6 4

+

' '
+
+

+

Gii (2): (2) <=> x(x + 6) = 4 (x + x + 6) <=> x


2
- 2x - 24 = 0
<=> x = 6 (t/m); x = - 4 (loi v x > 0). Thay vo (1) c y = 12
Vy hon thnh cng vic ngi th nht cn 6 gi, ngi th hai cn 12 gi.
Cu 4:
a) Ta c

BAC
= 90
0
(v gc ni tipchn na ng trn)
Tng t c

BDH CEH
= 90
0
Xt t gic ADHE c

A ADH AEH
= 90
0
=> ADHE l hnh ch nht.
T DE = AH m AH
2
= BH.CH (H thc lng trong tam gic vung)
hay AH
2
= 10 . 40 = 20
2
(BH = 10; CH = 2.25 - 10 = 40) => DE = 20
b) Ta c:

BAH
=

C
(gc c cnh tng ng vung gc) m

DAH ADE
(1)
(V ADHE l hnh ch nht) =>

C ADE
do

C BDE +
= 180
0
nn t gic
BDEC ni tip ng trn.
c) V O
1
D = O
1
B =>O
1
BD cn ti O
1
=>

1
B BDO (2)
T (1), (2) =>

1
ADE BDO +

B BAH +
= 90
0
=> O
1
D //O
2
E
Vy DEO
2
O
1
l hnh thang vung ti D v E.
Ta c S
ht
=
2
1 2 1 2 1 2
1 1 1
(O D O E).DE O O .DE O O
2 2 2
+
(V O
1
D + O
2
E = O
1
H + O
2
H = O
1
O
2
v
DE < O
1
O
2
)
2 2
2
ht 1 2
1 BC R
S O O
2 8 2
. Du "=" xy
ra khi v ch khi DE = O
1
O
2

DEO
2
O
1
l hnh ch nht
A l im chnh gia cung BC. Khi max
1 2
O DEO
S
=
2
2
R
.
Cu 5: Gii phng trnh: x
3
+ x
2
- x = -
1
3
(1)
100
O
1
O
2
D
O
B
C
H
A
E
(1) <=> 3x
3
+ 3x
2
- 3x = - 1 <=> 4x
3
= x
3
- 3x
2
+ 3x - 1 <=> 4x
3
= (x - 1)
3

<=>
3
4 x
= x - 1 <=> x(
3
4 1
) = 1 <=> x =
3
4 1
1

.
Vy phng trnh ch c 1 nghim x =
3
4 1
1

.
Li bnh:
Cu III
Ta thng gp bi ton :" Hai my cy cng cy mt cnh ng;
hai vi nc cng chy vo mt b; hai hp tc cng o mt con
mng; hai ngi cng lm chung mt cng vic) v.v" . Ta gi bi
bi trn thuc loi ton "Lm chung mt vic"
Mt s lu khi gii bi ton ny l
a) Khi lng cng vic phi hon thnh c quy c bng 1
(n v).
(Nng sut) (thi gian) = (khi lng lm c).
(Nng sut chung) = (tng cc nng sut ring).
(Bn c th t m ti sao li quy c khi lng cng vic l 1. Cng
vic hon tt ngha l hon thnh 100% khi lng cng vic. Bi 100%
= 1, l iu dn ti quy c trn)
b) Bi ton c th trnh by li gii bng h phng trnh hai n
hoc bng phng trnh mt n.
c) Trong bi ton trn (theo cc k hiu dng trong li gii) th :
Cc nng sut ring l
1
x
v
1
y
Nng sut chung : Mt mt c tnh l
1 1
x y
+
, mt mt gi thit
cho l
1
4
. Vy nn c phng trnh
1 1 1
4 x y
+
S 30
Cu 1.
1) Phng trnh tng ng vi 75 3 x

3 5 3 x

5 x
2) H phng trnh

'

+

8 2 4
1 2 3
y x
y x

'



1 2 3
7 7
y x
x

'



2
1
y
x
.
Cu 2.
101
1) Vi 2 m phng trnh tr thnh
0 2 5 2
2
+ x x
.
2
5 4.2.2 9 nn phng trnh c hai nghim
2
1
x
,
2
1
2
x
.
2) Phng trnh c bit thc
( ) ( ) 0 8 1 9 2 . 2 . 4 3
2
2
2
> + + + m m m m m vi mi
m
.
Do phng trnh lun c hai nghim
2 1
, x x
. Khi theo nh l Viet th

'

+
+
2
2
3
2 1
2 1
m
x x
m
x x
.
Biu thc A =
2 1
x x
=
( )
2
2 1
x x
=
( )
2 1
2
2 1
4 x x x x +
=
2
4
2
3
2
m m

,
_

+
=
( ) 8 1
2
1
9 2
2
1
2
2
+ + m m m
.
Do ( ) 0 1
2
m nn
( ) 2 2 8 8 1
2
+ m
, suy ra A
2
.
Du bng xy ra

1 m .
Vy gi tr nh nht ca A l
2
, t c khi 1 m .
Cu 3. 1) Ta c
3
9 25 4 9 5 2 a a a a a a a + +
2 ( 2) a a + v
2
2 ( 2) a a a a + +
nn P =
( )
( )
2 2
2
2
a a
a a
a
+

+
.
2) Gi vn tc can trong nc yn lng l
(km/h, 4) x x >
Vn tc ca n khi nc xui dng l 4 x + v thi gian ca n chy xui dng
l
48
4 x +
.
Vn tc ca n khi nc ngc dng l 4 x v thi gian ca n chy ngc
dng l
48
4 x
.
Theo gi thit ta c phng trnh
48 48
5
4 4 x x
+
+
(*)
(*)
2 2
48( 4 4) 5( 16) 5 96 80 0 x x x x x + +
Gii phng trnh ta c
0,8 x
(loi), 20 x (tha mn)
Vy vn tc ca n khi nc yn lng l 20 km/h
Cu 4.
102
1) Chng minh ABD cn
Xt ABD c BCDA v CA = CD nn
BC va l ng cao va l trung tuyn ca n.
Vy ABD cn ti B
2) Chng minh rng ba im D, B, F cng
nm trn mt ng thng.
V

CAE
= 90
0
, nn CE l ng knh ca (O).
Ta c CO l ng trung bnh ca tam gic
ABD
Suy ra BD // CO hay BD // CE (1)
Tng t CE l ng trung bnh ca tam gic
ADF.
C
O
D
F
B
A
E
Suy ra DF // CE (2). T (1) v (2) suy ra D, B, F cng nm trn mt ng thng.
3) Chng minh rng ng trn i qua ba im A, D, F tip xc vi ng trn (O).
Tam gic ADF vung ti A v theo tnh cht ca ng trung bnh DB = CE
= BF B l trung im ca DF. Do ng trn qua ba im A,D,F nhn
B lm tm v AB lm bn knh. Hn na, v OB = AB - OA nn ng trn
i qua ba im A, D, F tip xc trong vi ng trn (O) ti A.
Cu 5.
V cc s
c b a , ,
dng nn p dng bt ng thc Csi cho hai s ta c:
( )
2
) ( c b a
c b a
+ +
+

( )
c b a
a
c b a
a
c b
a
+ +

+
2
Tng t ta cng c:
c b a
b
a c
b
+ +

+
2
,
c b a
c
b a
c
+ +

+
2
Cng cc bt ng thc cng chiu trn ta c
2
2 2 2

+ +
+ +

+
+
+
+
+ c b a
c b a
b a
c
a c
b
c b
a
.
Du bng xy ra

'

+
+
+

b a c
a c b
c b a
0 c b a , khng tho mn.
Vy 2 >
+
+
+
+
+ b a
c
a c
b
c b
a
.
Li bnh:
103
Cu II.2
Cc bn tham kho thm mt li gii sau
Gi x
1
,

x
2
l cc nghim nu c ca phng trnh . T cng thc
1,2
2
b
x
a
t

suy ra :

2
1 2
( 1) 8
| | 2
| | 2
m
x x
a
+
, vi mi m. (*)
Kt qu (*) cho thy > 0 ,m ng thi c min|x
1
x
2
| = 2 , t
c khi m = 8.
Li gii gim bt ti a cc php ton, iu y ng hnh gim
bt nguy s sai st.
Cu IV.2
Vic chng minh ba im A, B, C thng hng thng c thc hin
bng cch chng minh mt trong ba iu tng ng sau :
AB + BC = AC (khi B thuc on thng AC).
Mt trong ba im y l nh mt gc bng 180
0
(chng hn

0
180 ABC
).
Mt trong ba im y l im chung ca hai on thng song
song (chng hnAB // BC).
Mt trong ba im y l im chung ca hai on thng cng to
vi ng thng () c sn mt gc bng nhau (chng hn

( , ) ( , ) AB AC ).
S 31
Cu 1: Tnh
a) A = 20 3 18 45 72 4.5 3 9.2 9.5 36.2 + + =
= 2 5 9 2 3 5 6 2 3 2 5 + .
b) B =
7 4 7 4 + +
1 7 1 7 ) 1 7 ( ) 1 7 ( 7 2 8 7 2 8 2
2 2
+ + + + + + B
14 7 2 2 B B
c) C =
1 2 1 2 + + x x x x
vi x > 1
C = 1 1 1 1 ) 1 1 ( ) 1 1 (
2 2
+ + + + x x x x
104
+) Nu x > 2 th C = 1 2 1 1 1 1 + + x x x
+) Nu x < 2, th C = 2 1 1 1 1 + + x x .
Cu 2: a) Hm s y = (2m - 1)x - m + 2 nghch bin trn R
khi v ch khi 2m - 1 > 0 <=> m >
2
1
b) th hm s i qua A (1; 2) khi: 2 = (2m - 1).1 - m + 2 <=> m = 1.
Vy hm s y = x + 1
Cu 3: Gi x, y l thi gian ngi th th nht v ngi th th 2 lm mt
mnh (x, y > 0, tnh bng gi).
- Mt gi mi ngi lm c
x
1
;
y
1
cng vic c 2 ngi lm c
x
1
+
y
1
=
16
1
. (v 2 ngi lm trong 16 gi th xong cng vic)
- Trong 3 gi ngi th nht lm c
x
3
(CV), 6 gi ngi 2 lm c
y
6

(CV) v c hai lm c
4
1
(CV) nu ta c
x
3
+
y
6
=
4
1
Do ta c h phng trnh:
1 1 1 3 3 3 3 1
x 24 x y 16 x y 16 y 16
3 6 1 3 6 1 1 1 1 y 48
x y 4 x y 4 x y 16

+ +



' ' ' '


+ + +


.
Vy ngi th nht hon thnh cng vic trong 24 gi
ngi th hai hon thnh cng vic trong 48 gi

Cu 4: a) Xt ABM vAMC
C gc A chung;

AMB MCB

( =
2
1
s cung MB)
=> AMB ~ ACM (g.g)
=>
AM
AB
AC
AM

=> AM
2
= AB.AC
b) T gic AMON c

M N +
= 180
0

105
D
K
I
B
O
N
A
C
M
(V

M N
= 90
0
tnh cht tip tuyn)
=> AMON l t gic ni tip c
- V OI BC (nh l ng knh v dy cung)
Xt t gic AMOI c

M I +
$
= 90
0
+ 90
0
= 180
0
=> AMOI l t gic ni tip c
c) Ta c OA MN ti K (v K trung im MN), MN ct AC ti D.
Xt t gic KOID c

K I +
$
= 180
0
=> t gic KOID ni tip ng trn tm O
1
=> O
1
nm trn ng trung trc ca DI m AD.AI = AK.AO = AM
2
=
AB.AC khng i (V A, B, C, I c nh).
Do AI khng i => AD khng i => D c nh.
Vy O
1
tm ng trn ngoi tipOIK lun thuc ng trung trc ca DI
c nh.
Cu 5:
Ta c:
x 1 2x 2 1
(2x 1)y x 1 y 2y 2y 1
2x 1 2x 1 2x 1
+ +
+ + +
+ + +
(*)
Xt pt (*): x, y nguyn th 2x +1 phi l c ca 1, do :
+ Hoc 2x +1 =1

x = 0, thay vo (*) c y = 1.
+ Hoc 2x +1 = -1

x = -1, thay vo (*) c y = 0


Vy pt cho c 2 nghim nguyn l: (0; 1) ; (-1; 0).
Li nhn.
Cu IV.c Lin h vi li bnh sau cu 4c 1
S 32
Cu 1: 1) P = 7 3 2 7 3 2 [ 7 3 2 ][ 7 3 2 ] ( )( ) ( ) ( ) + + +
=
2 2
7 3 2 7 3 4 3 4 4 3 ( ) ( )) ( ) + .
2) ng thng d v d
song song vi nhau khi v ch khi:
2 2
m 2 m 1 3 m 4
m 2
m 2 m 1 1 m 2
t

' ' '


Cu 2: x
2
+ (2m + 1) x + m
2
+ 1 = 0 (1)
a) Khi m = 1 ta c phng trnh: x
2
+ 3x + 2 = 0
V a = 1; b = 3; c = 2 => a - b + c = 0
106
Vy phng trnh c x
1
= - 1; x
2
= - 2
b) Phng trnh (1) c 2 nghim m khi v ch khi:
2 2
2
3
0 2m 1 4 m 1 0
m
4m 3 0
4
S 0 2m 1 0
2m 1 0 1
m
P 0
m 1 0
2
( ) ( )
( )

+ +


< + <
' ' ' '
+ >


>
>
+ >

3
m
4

.
Cu 3: Ta c: a
2
+ b
2
> 2ab = 1 (v ab = 1)
A = (a + b + 1)(a
2
+ b
2
) +
4
a b +
> 2(a + b + 1) +
b a +
4
= 2 + (a + b +
b a +
4
) + (a + b) > 2 + 4 + 2 = 8.
(a + b +
b a +
4
>
4
v a + b > 2 ab v p dng BT Csi cho 2 s
dng)
Du = khi v ch khi a = b =
2
1
.
Vy minA = 8.
Cu 4:
a) Xt t gic BHMK:

H K +
= 90
0
+ 90
0
= 180
0
=> T gic BHMK ni tip ng trn.
CM tng t c t gic CHMI cng ni tip c.
b) Ta c

B HMK C HMI + +
= 180
0
m

B C


HMK HMI
(1)

KBM BCM KBM KHM , (v 2 gc ni tip
cng chn cung MK v gc to bi tia tt ... v
gc ni tip cng chn cung BM).

HCM HIM
(gc to bi tia tip tuyn v gc ni
tip cng chn

HM
)


KHM HIM
(2).
T (1), (2) =>HMK ~IMH (g.g) =>
2
MH
MH
MK
MI
MH

= MI .MK
(pcm)
c) Ta c PB = PM; QC = QM; AB = AC (Theo t/c hai tip tuyn)
107
H
K
I
B
C
A
M
Xt chu vi APQ = AP + AQ + PQ = AP + AQ + PM + QM
= (AP + PB) + (AQ + QC) = AB + AC = 2AB khng i.
V A c nh v ng trn (O) cho trc nn chu vi APQ khng
ph thuc vo v tr ca im M (pcm).
Cu 5: Gi s h
5
2 2
x 2y a (1)
x y 1 (2)

'
+

c nghim l (x; y)
T (2) suy ra
x 1, y 1
. T (1) ta c:
5 5 2 2 2 2
x 2y x 2 y x 2 y ( x y ) ( y 2 y 1) 1 + + + + +
2 2
2 ( y 2 y 1) 2 ( y 1) 2 +

a 2
tri gi thit l
a 2 >
.
Suy ra h trn v nghim, pcm.
S 33
Cu 1: a)
x 3y 10 2x 6y 20 x 3y 10
2x y 1 2x y 1 y 3
+ + +

' ' '
+ +

x 3 3 10 x 1
y 3 y 3
( )
.
+

' '


b) Hm s y = (m + 2) x - 3 ng bin trn R khi v chi khi m + 2 > 0

m
> - 2.
Cu 2: a) A =
a 1 2 a 1 2 a
:
a 1
a 1 a(a 1) (a 1)
_ 1
+

1

+
+ + + +
, ]
=
2 2 2
( a 1) 1 2 a ( a 1) ( a 1)
: :
a 1 a 1
a 1 (a 1)( a 1) ( a 1)(a 1)
1


1
+ +
+ + + + +
]
.
=
2
2
( a 1) (a 1)( a 1)
. a 1
a 1
( a 1)
+ +
+
+

.
b) a = 2011 - 2 1 2010 ) 1 2010 ( 2010
2
a
Vy A = 2010 .
Cu 3: a) Vi k = -
2
1
ta c:
-
2
1
(x
2
- 4x + 3) + 2 (x - 1) = 0

x
2
- 8x + 7 = 0. V a + b + c = 1 + (- 8) + 7 = 0
Nn pt c nghim x
1
= 1; x
2
= 7
108
b) + Nu k = 0, phng trnh c dng 2(x - 1) = 0

x = 1
+ Nu k

0, phng trnh c dng: kx


2
+ 2(1 - 2k) x + 3k - 2 = 0
' = (1 - 2k)
2
- k(3k - 2) = 1- 4k + 4k
2
- 3k
2
+ 2k
= k
2
- 2k + 1 = (k - 1)
2
> 0 vi mi k.
Vy phng trnh c nghim vi mi k.
Cu 4:
a) Qua A v tip tuyn chung trong ct
BC ti M
Ta c MB = MA = MC (t/c 2 tip tuyn
ct nhau)

A
= 90
0
.
b) Gi s R > R. Ly N trung im
ca OO.
Ta c MN l ng trung bnh ca hnh thang vung OBCO
(OB // OC;

B C
= 90
0
) v tam gic AMN vung ti A.
C MN =
2
' R R +
; AN =
R R
2

. Khi MA
2
= MN
2
- AN
2
= RR
=> MA =
' RR
m BC = 2MA = 2
' RR
c) Ta c O, B, D thng hng (v

BAD
= 90
0
; OA = OB = OD)
BDC c

DBC
= 90
0
, BA CD, ta c: BD
2
= DA . DC (1)
ADE ~EDC (g.g) =>
DE
DA
DC
DE

=> DA . DC = DE
2
(2)
(1), (2) => BD = DE (pcm).
Cu 5:
Xt
2 1
+
=
2 1
2
2
2
1 2 1
2
2
2
1 2
2
2 1 1
2 ) ( 4 4 4 a a a a b b a a b a b a + + + +
(v a
1
a
2
> 2(b
1
+ b
2
)).
M 0 ) ( 2
2
2 1 2 1
2
2
2
1
+ a a a a a a ,
2 1
+
> 0
=> Tn ti
1

hoc
2

khng m => t nht mt trong 2 phng trnh cho


c nghim.
109
E
N
A
M
O O'
B
C
D
Li bnh:
Cu III.b
1) chng minh phng trnh c nghim khng ph thuc gi tr
ca k c hai cch gii.
Cch 1 ( ni li bnh sau cu 2(1) 24)
Xem k(x
2
4x 3) + 2(x 1) = 0 (*) l ph ng trnh
i v i n k . Th th (*) c nghi m khng ph thu c k khi
v ch khi x
2
4x 3 = 2(x 1) = 0 x = 1.
Cch 2 (Phng php cn v )
+ Phng trnh (*) c nghim vi mi x t phi c nghim vi k =
0.
+ Vi k = 0 ta c k(x
2
4x 3) + 2(x 1) x = 1.
Thay x = 1 vo (*) c 0k + 0 = 0 ngha l x = 1 l nghim ca (*) vi
mi k. Ta c iu phi chng minh.
2) Kt qu mt bi ton u phi ch c l p s. Ci quan trng hn
l cch ngh ra li gii chng nh th no, c bao nhiu con ng (cch
gii) i n kt qu :
Cu V : 1) Mu cht ca bi ton l chuyn ho hnh thc bi ton. C
th y l bit thay th vic chng minh t nht mt trong hai phng
trnh c nghim bng cch chng minh
1
+
2
0. S chuyn ho ny
gip kt ni thnh cng vi gi thit a
1
+ a
2
2(b
1
+ b
2
).
2) Mt cch hiu khc ca bi ton l :
Chng minh c hai phng trnh khng th cng v nghim. Vi
cch hiu ny ta chuyn ho thnh chng minh kh nng
1
+
2
< 0
khng th xy ra.
Tht vy: Nu
1
< 0 v
2
< 0 suy ra
1
+
2
< 0. iu ny s dn
ti mu thun vi a
1
+ a
2
2(b
1
+ b
2
). Bi ton c chng minh.
3) Cc cch chng minh bi ton trn cng l cch chng minh
trong nhi u ph ng trnh b c hai, t nh t c m t ph ng
trnh c nghi m.
4) Cng mt kiu t duy y bn d dng chng minh :
Vi mi gi tr ca m, phng trnh x
2
mx + m = 0 khng th c
hai nghim cng dng.
Tht vy :
+ Nu m = 0, phng trnh c nghim x = 0.
+ N u m < 0, ph ng trnh c nghi m hai nghi m
tri d u (do ac < 0).
110
+ Nu m > 0, nu c hai nghim x
1
, x
2
u m th x
1
+ x
2
< 0 suy ra
0
b
m
a
<
(!).
Mu thun vi m > 0.
Vy l bi ton c chng minh.
S 34
Cu 1: P =
1 1 1 1 + + a a
Nu a> 2 => 1 2 0 1 1 a P a
Nu 1< a < 2 => 1 1 a < 0 => P = 2
Cu 2: KX: x > 0; x

1.
1) Q =
x
x
x x
x x
x
x x
x
x 1
) 1 .( 4
4 . ) 1 (
1
) 1 ( ) 1 (
.
4
) 1 (
2 2 2 2

+
.
2) Q = - 3 3 x => 4x + 3 x - 1 = 0

x 1 (loai)
1
x
1
16
x
4

(tha
mn)
Cu 3: t
x
= t, c t
2
+ 2(m - 1)t + m + 1 = 0 (1)
Phng trnh c ng 2 nghim phn bit <=> (1) c 2 nghim khc du hoc
(1) c nghim kp t > 0.
+) (1) C 2 nghim khc du <=> m + 1 < 0 <=> m < -1
+) ' = 0 <=> m
2
- 3m = 0 <=>
m 0
m 3


Thay vo (1) xt th m = 0 tha mn, m = 3 b loi.
Vy m < - 1 hoc m = 0.
Cu 4: PT <=> 25 ) 1 ( 16 ) 1 ( 3
2 2
+ + + x x = 9 - (x - 1)
2
VT > 9; VP < 9 (v (x - 1)
2
> 0) nn:
PT <=>
VT 9
VP 9

'

<=> x = 1 (TM)
Cu 5: 1) Gi H l hnh chiu ca O trn
ng thng MN. Xt t gic OAMH

0 0
A H 180 (do A H 90 ) +
=> OAMH l t gic ni tip ng trn.
Tng t t gic OANH ni tip c
111
N
M
O A
B
H
=>

1 1 1 1
A M , B N (2 gc ni tip chn 1 cung)

0
1 1 1 1
A B M N 90 + + =>

AHB
= 90
0

=> MN l tip tuyn
2) Ta c AM = MH, BN = NH, theo h thc lng
trong tam vung, ta c:
AM. BN = MH . NH = OH
2
=
4
2
AB
(pcm)
3.
2
1

MON
S
OH . MN >
2
1
OH . AB (V AMNB l hnh thang vung)
Du = khi v ch khi MN = AB hay H l im chnh gia ca cung AB.

M, N song song vi AB

AM = BN =
AB
.
2

Vy
MON
S

nh nht khi v ch khi AM = BN =


AB
.
2

S 35
Cu 1: A =
2
x 3 (x 3)
x 3 x 3
+ +

+ +
=
1 khi x 3
1 khi x 3
>

'
<

Cu 2: a) Bnh phng hai v ta c:


x
2
- 2x + 4 = 4 <=> x(x - 2) = 0 <=> x = 0 hoc x = 2
b) ng thng (d) c phng trnh y = ax + b i qua im A (1; 2) v B (2;
0) khi v ch khi:
a b 2 a 2
2a b 0 b 4
+

' '
+

Vy y = - 2x + 4
Cu 3: a) Vi m = 2, ta c phng trnh
(x
2
- x - 2)(x - 1) = 0 <=>
2
x 1; x 2
x x 2 0
x 1
x 1 0



Vy phng trnh c 3 nghim x t 1; x = 2
b) V phng trnh (1) lun c nghim x
1
= 1 nn phng trnh (1) c 2 ng
nghim phn bit khi v ch khi:
- Hoc phng trnh f(x) = x
2
- x - m = 0 c nghim kp khc 1
112

1
0 1 4m 0 m 1
m
4
f (1) 0 1 1 m 0 4
m 0

+


' ' '



.
- Hoc phng trnh f(x) = x
2
- x - m = 0 c 2 nghim phn bit trong c
mt nghim bng 1.

1
0 1 4m 0 m
m 0.
4
f (1) 0 m 0
m 0

> + > >




' ' '



Vy phng trnh (1) c ng 2 nghim phn bit khi v ch khi m = -


4
1
; m = 0.
Cu 4:
a) V MA, MB l tip tuyn ca ng trn (O)
Nn MA OA; MB OB; M OI CD
(Theo nh l ng knh l dy cung).
Do

MAO MBO MIO
= 90
0
=>
3 im A, B, I
thuc ng trn ng knh MO hay
5 im M, A, I, O, B cng thuc mt
ng trn.
b) Ta c:

AIM AOM
(v 2 gc ni tip cng chn cung MA)

BIM BOM

(v 2 gc ni tip cng chn cung MB) m

AOM BOM
(tnh cht hai tip tuyn)
=>

AIM BIM
=> IM l phn gic ca gc AIB (pcm).
Cu 5:
4 4
3 3 2 2
x y 1 1
x y x y 2

'
+ +

( )
( )
T (1) suy ra:
4
x 1 x 1 . Tng t
y 1
(3).
2 2
2 x 1 x y 1 y 0 ( ) ( ) ( ) + (4), T (3) suy ra v tri ca (4) khng m.
nn
(4)
2
2
x 1 x 0 x 0 x 0 x 1 x 1
y 0 y 1 y 0 y 1
y 1 y 0
( )
; ; ;
( )


' ' ' ' '


.
Th li th h ch c 2 nghim l:
x 0 x 1
y 1 y 0
;

' '



113
I
C
O
B
M
D
A
S 36
Cu 1: a) P =
1 5 1 5 1 5 5 1 2 5 + + + +
.
b) x
2
+ 2x - 24 = 0
' = 1 + 24 = 25 =>
'
= 5
=> phng trnh c 2 nghim x
1
= - 1 + 5 = 4; x
2
= - 1 - 5 = - 6
Cu 2: a) P =
2 a a 1 7 a 3
a 3 a 3 ( a 3)( a 3)
+
+ +
+ +
=
) 3 )( 3 (
3 7 3 4 6 2
) 3 )( 3 (
3 7 ) 3 )( 1 ( ) 3 ( 2
+
+ + +

+
+ + +
a a
a a a a a
a a
a a a a a
=
3
3
) 3 )( 3 (
) 3 ( 3
) 3 )( 3 (
9 3
+

a
a
a a
a a
a a
a a

Vy P =
3 a
a 3 +
.
b) P < 1

3 a 3 9
1 3 a a 3 a 0 a
2 4
a 3
< < + < <
+
.
Cu 3: a) Vi m = 4 ta c x
4
- 5x
2
+ 4 = 0
t x
2
= t , vi t 0 ta c pt t
2
- 5t + 4 = 0 <=> t
1
= 1; t
2
= 4
T , ta c:
2
2
x 1 x 1
x 2
x 4
t

.
Vy phng trnh c 4 nghim
x 1; x 2. t t
b) x
4
- 5x
2
+ m = 0 (1) c dng f(y) = y
2
- 5y + m = 0 (2) (vi y = x
2
; y > 0)
Phng trnh (1) c ng 2 nghim phn bit <=> phng trnh (2):
1) Hoc c nghim kp khc 0 <=>
25
0 m 25
m
4
f (0) 0 4
m 0




' '

.
2) Hoc c 2 nghim khc du m 0 < .
Vy m =
4
25
hoc m < 0 th phng trnh (1) c ng 2 nghim phn bit
Cu 4: a)

FAB
= 90
0
(v AF AB)

BEC
= 90
0
(gc ni tip chn na ng trn)
114
D
M
E
O
F
A
C B
=>

BEF
= 90
0
. Do

FAB BEF +
= 180
0
Vy t gic ABEF ni tip ng trn.
b) Ta c:

AFB AEB
= (
2
1
s cung AB) (v
2 gc ni tip cng chn 1 cung)

AEB BMD
= (
2
1
s cung BD) (v 2 gc ni tip cng chn 1 cung)
Do

AFB BMD => AF // DM m FA AC => DM AC
c) ACF ~ ECB (g.g) =>
BC
CF
CE
AC

=> CE.CF = AC.BC (1)


ABD ~ AEC (g.g) =>
AC
AD
AE
AB

=> AD.AE = AC.AB (2)


(1), (2) => AD.AE + CE.CF = AC(AB + BC) = AC
2
(pcm)
Cu 5: Ta c y =
x
x x
x
x x
x x
+
+

+
+

) 1 (
1
2 ) 2 2 ( 1
1
2

= 2 + 1 + 2 2 3
1
.
1
2
2 3
1
1
2
+

+
x
x
x
x
x
x
x
x
(p dng BT Csi vi
2 s dng)
ng thc xy ra <=>
1 2
1
1
2

x
x
x
x
x
(loi nghim x = - 1 -
2
)
Vy gi tr nh nht ca y bng 3 + 2
2
khi x =
2
-1.
Li nhn.
Cu IV.c. Lin h vi Li bnh sau cu 4c, 6.
S 37
Cu 1: M =
1
) 1 (
1
) 1 (
3 3
+
+

+ +

x x
x x
x x
x x
+ x + 1
= 1
1
) 1 )( 1 (
1
) 1 )( 1 (
+ +
+
+ +

+ +
+ +
x
x x
x x x x
x x
x x x x
= x - x - x - x + x + 1 = x - 2 x + 1 = ( x - 1)
2

115
Cu 2: a)
3x 5y 18 3x 5y 18 11y 33 x 1
x 2y 5 3x 6y 15 x 2y 5 y 3



' ' ' '
+ + +

.
Vy h phng trnh c nghim duy nht (- 1; 3)
b) Hai ng thng (d) v (d) song song khi v ch khi:
3
a 3 a a
2
b 2 b
b 1

' '

.
Cu 3: a) Khi m = - 3, ta c phng trnh x
2
- 2x - 3 = 0
V a - b + c = 1 - (- 2) + (- 3) = 0 nn x
1
= - 1; x
2
= 3
b) Phng trnh c nghim

' > 0

1 - m > 0

m < 1
Khi theo h thc Vit, ta c: x
1
+ x
2
= 2 v x
1
x
2
= m (1)
2 2 2
1 2 1 2 1 2
2 2 2 2 2
1 2 1 2
x x (x x ) 2x x 1 1
1 1 1
x x x x (x x )
+ +
+
(2)
T (1), (2), ta c: 4 - 2m = m
2
<=> m
2
+ 2m - 4 = 0
' = 1 + 4 = 5 =>
'
= 5 nn m = -1 + 5 (loi); m

= - 1 - 5 (T/m v m < 1).
Vy gi tr m cn tm l: m 1 5
Cu 4: a) Ta c

ACK
= 90
0

(v gc ni tip chn na ng trn)
Nn CK AC m BH AC (v H trc tm)
=> CK // BH tng t c CH // BK
=> T gic BHCK l hbh (pcm)
b) OM BC => M trung im ca BC
(nh l ng knh v dy cung) => M l
trung im ca HK (v BHCK l hnh bnh
hnh) => pcm AHK c OM l ng trung
bnh => AH = 2.OM
c) Ta c

AC C BB C
= 90
0
=> t gic BCBC ni tip ng trn =>

AC B
=

ACB
m

ACB BAx
(Ax l tip tuyn ti A) => Ax // BC
OA Ax => OA BC. Do S
ABOC
=
2
1
R.BC
Tng t: S
BAOC
=
2
1
R.AC; S
CBOA
=
2
1
R.AB
116
M
H
O
B
A
C
K
ABC
S

=
2
1
R(AB + BC + CA)=
2
1
AA .BC <
2
1
(AO + OM).BC
=> AB + BC + CA, ln nht khi A, O, M thng hng <=> A l m
chnh gia cung ln BC.
Cu 5: y =
2
2 2
2
x x 1
y(x 2x 2) (x x 1) 0
x 2x 2
+ +
+ + + +
+ +

(y - 1)x
2
+ (2y - 1)x + (2y - 1) = 0 (1)
- Nu y = 1 th x = - 1
- Nu y

1 th (1) l phng trnh bc hai i vi x. (1) c nghim th


phi c
= (2y - 1)
2
- 4 (y - 1)(2y-1)

0
1 3
(2y 1)(2y 3) 0 y
2 2

.
1
y
2

khi x = 0. Vy min y =
1
2
..
S 38
Cu 1:
a) Ta c x
2
+
3
x x( x 1) x( x 1)(x x 1) + + +
nn P =
x
x x
x x
x x x x ) 1 2 (
1
1
) 1 )( 1 ( +
+
+
+ +
= x( x 1) 1 2 x 1 x x + + . Vy P =
x x
.
b) P = 0

x - x = 0

x ( x - 1) = 0

x = 0 (loi) ; x = 1 (t/m)
Vy x = 1 th P = 0
Cu 2: a) Ta c
2
1 x
= 1 - x. k:
x
< 1
Bnh phng hai v, ta c phng trnh h qu: 1 - x
2
= (1 - x)
2
.
<=> 2x
2
- 2x = 0 <=> 2x (x - 1) <=> x = 0 ; x = 1
Thay vo pt cho th li th c 2 nghim u tho mn.
b) k: x

0 v y

0.
H cho tng ng vi h phng trnh:
117
3 3 5
7 7
x 2
x 2 x y 2
x 2
3
4 3 2 1
4 3 y 3
1
1
y
x y
x y



' ' ' '

.
Vy h phng trnh c nghim (2; 3).
Cu 3: a) Vi m = - 1 ta c phng trnh:
x
2
+ 4x = 0 <=> x(x + 4) = 0 <=> x = 0 ; x = - 4
b) Phng trnh (1) c nghim khi ' > 0 <=> (m -1)
2
- (m+ 1) = m
2
- 3m =
m(m - 3) > 0
<=> m > 3 ; m < 0. (1)
Khi theo h thc Vit ta c: x
1
+ x
2
= 2(m - 1) v x
1
x
2
= m + 1 (2)
Ta c:
1 2
2 1
x x
x x
+
=
2 2 2
1 2 1 2 1 2
1 2 1 2
x x (x x ) 2x x
x x x x
+ +

.
nn
2
2
1 2 1 2 1 2
1 2 1 2
2 1 1 2
x x (x x ) 2x x
4 4 (x x ) 6x x
x x x x
+
+ +
(3)
T (2). (3) ta c: 4(m - 1)
2
= 6(m + 1) <=> 4m
2
- 8m + 4 = 6m + 6 <=> 2m
2
- 7m - 1 = 0

m
= 49 + 8 = 57 nn m =
4
57 7
< 0 ; m =
4
57 7 +
> 0.
i chiu k (1) th c 2 nghim u tho mn.
Cu 4: a) Ta c:

DBO DMO
= 90
0
(v gt)
=> 2 im B, M thuc ng trn ng knh
DO =>pcm
b) Chng minh tng t c 4 im O, C, E, M
cng thuc mt ng trn =>

MEO MCO

(v 2 gc ni tip cng chn cung MO)

MBO MDO
(v 2 gc ni tip cng chn
cung MO)
M

MBO MCO
(v BOC cn ti O)
=>

MEO MDO
=>DOE cn ti O
M MO DE nn MD = ME (pcm)
Cu 5: t
1
2
+ x
= t, vi t > 0, ta c t
2
- (x + 3) t + 3x = 0
118
E
D
A
B
C
M
Xem pt trn l pt bc 2 i vi t.
= (x + 3)
2
- 12x = (x - 3)
2
t
1
=
x
x x

+ +
2
3 3
; t
2
=
3
2
3 3

+ + x x
Do : - Hoc:
1
2
+ x
= x

2 2
x 0
x 1 x

'
+

v nghim.
- Hoc:
1
2
+ x
= 3

x
2
= 8

x =
2 2 t
Vy phng trnh c 2 nghim x =
2 2 t
.
S 39
Cu 1: (2 im)
1) Tnh: 48 - 2 75 + 108 = 16 . 3 - 2 25 . 3 + 36 . 3
= 4 3 - 10 3 + 6 3 = 0
2) Rt gn biu thc: P =
1 1 1
- . 1 -
1 - x 1 + x x
_ _

, ,
=
1 + x - 1 + x x - 1
1- x
x
_ _


, ,
=
2 x x - 1
.
1- x
x
=
- 2
1 + x
Cu 2:1) ng thng y = ax + b i qua 2 im M (3; 2) v N( 4; -1) nn:
2 = 3a + b

- 1 = 4a + b

'

a = - 3
b = 11

'

2) Gii h pt:
2x + 5y = 7
3x - y = 2

'

2x + 5y = 7
15x - 5y = 10

'

17y = 17
3x - y = 2

'

x = 1
y = 1

'

.
Cu 3:
1) Khi m = 2, phng trnh (1) tr thnh: x
2
- 4x -12 = 0
' = 16, pt cho c 2 nghim: x = - 2; x

= 6.
2) Phng trnh (1) c nghim

' 0

m
2
+ 6m
m 6; m 0
(2)
Khi , theo h thc Vi t ta c:
1 2
1 2
x + x = 2m
x x = - 6m

'

(3)
Phng trnh c 1nghim gp 2 ln nghim kia khi v ch khi:
2 2
1 2 2 1 1 2 2 1 1 2 1 2
x 2x ; x 2x (x 2x )(x 2x ) 0 5x x 2(x x ) 0 +
119

O
1
E
I
C
O
N
M
B A
2 2
1 2 1 2 1 2 1 2 1 2
5x x 2[(x x ) 2x x ] 0 9x x 2(x x ) 0 + +
(4)
T (3), (4), ta c:
2
27
54m 8m 0 m 0; m
4

(tha mn k (2))
Vy cc gi tr m cn tm l
27
m 0; m
4

.
Cu 4:
1. Theo gi thit MN AB ti I

0 0
ACB = 90 hay ECB = 90

0
EIB + ECB = 180
m y l hai gc i ca t gic IECB nn
t gic IECB l t gic ni tip.
2. Theo gi thit MN AB, suy ra A l im
chnh gia ca

MN
nn

AMN = ACM
(hai
gc ni tip chn hai cung bng nhau) hay

AME = ACM
, li c

CAM
l
gc chung do tam gic AME ng dng vi tam gic ACM
AM AE
=
AC AM

AM
2
= AE.AC.
3. Theo trn

AMN = ACM
AM l tip tuyn ca ng trn ngoi tip
ECM. Ni MB ta c

AMB
= 90
0
, do tm O
1
ca ng trn ngoi tip
ECM phi nm trn BM.
Ta thy NO
1
nh nht khi NO
1
l khong cch t N n BM

NO
1
BM. Gi O
1
l chn ng vung gc k t N n BM ta c O
1
l tm
ng trn ngoi tip ECM c bn knh l O
1
M.
Do khong cch t N n tm ng trn ngoi tip ECM l
nh nht th C phi l giao im ca ng trn (O
1
), bn knh O
1
M vi
ng trn (O) trong O
1
l hnh chiu vung gc ca N trn BM.
Cu 5: T 2x + 3y 6
2 2
y 2 - x - y x - 2
3 3

K = x
2
- 2x - y
2 2
2x 2 22 - 22
x - 2x + - 2 = (x - ) -
3 3 9 9

Suy ra : min K =
- 22
9
khi x =
2
3
; y =
14
9

Ta c : 2x
2
+ xy 4x ( x0)
120
( )
2
- y x + 2
xy
x - 2x - y - - y = 0
2 2

Suy ra : max K = 0 khi
y = 0
x = 0

'

hoc
y = 0
x = 2

'

Li bnh :
Cu V
Nhiu khi tm trc tip GTNN ca biu thc K tht kh khn. "Ci kh
l ci khn", ngi ta bc cu K qua biu thc B (b hn) theo s "b
dn": K B . Ri i tm GTNN ca B, t m suy ra GTNN ca biu
thc K. Cc mi lin h gia K v gi thit s ch dn chng ta tm n B.
+ Trong bi ton trn, thy trong biu thc K = x
2
2x y c cha y,
nn thun theo s "b dn" ta bin i :
2x + 3y 6
2
2
3
x
y
Thay y bi
2
2
3
x

ta c
2
2 22
3 9
K B x
_


,
.
Cng vy, i vi tm GTLN th vic bc cu phi theo s "ln
dn": K L
+ Trong cc gi thit khng th suy ra y h(x) tm L (ln hn) trong
s "ln dn" . Vy nn c biu thc L buc phi nh gi b phn
cn li x
2
2x g(x).
Ta c 2x + y 4
2
2
y
x

0 x


2
2
2
xy
x x
. ( y
( )
2
xy
g x
)
Thay x
2
2x bi
2
xy
ta c
( 2)
2
y
K L x +
.
Chc chn bn cn thc mc l bi ton c hai gi thit, th nhng khi
tm GTNN (GTLN) li s dng gi thit ny m khng s dng gi thit
kia ?
+ Trong qu trnh nh gi c th tm c nhiu biu thc B. Gi B
k
l
mt trong s cc biu thc B tm c v c minB
k
= . Th th cha
hn l GTNN ca K. Ch trong trng hp khi minB
k
= m ta cng
c K = B
k
(ho gii c du "=" trong s "ln hn") th mi c
minK = minB
k
= . Trong trng hp biu thc B
k
c gi l "kt".
121
Li gii ch thnh cng khi tm c "kt". Trong bi ton trn, s dng
gi thit cn li khng dn ti "kt".
Tnh hung cng tng t i vi vic tm biu thc L. Biu thc L dn
ti maxK cng c gi l "kt".
+ Trong bi ton trn, hnh thc cc gi thit cha ch dn "bt
mch" s dng gi thit ny hay gi thit kia. Nhiu bi ton phc tp c
th cn s kt hp ca tt c cc gi thit mi tm c "kt".
Mu cht ca bi ton tm GTNN, GTLN l tm "kt".
Nhn li kt ca cc trc :
+ Cu 5, 1, "kt" chnh l biu thc phi tm GTNN.
+ Cu 5, 11, "kt" l
3 3 6 1 8
( )
2 2 2
k
B x y x y
x y
_
_
+ + + + +

,
,
.
+ Cu 5, 32, "kt" l B
k
=
1
+
2
.
S 40
Cu 1. a) 3x + 4y = 2
3 1
y x
4 2
+
, nn h s gc ca ng thng d l
k =
3
4

.
b) d // d
1

2 2
3 1 1
m 1 m m
1
4 4 2
m
1 1 1 2
m m m
2 2 2

t



' ' '




.
Vy vi
1
m
2

th d
1
// d.
Cu 2. H phng trnh
ax by 3
bx ay 11
+

'

c nghim
x 3
y 1

'

nn
a.3 b( 1) 3
b.3 a( 1) 11
+

'

3a b 3
a 3b 11

'
+

9a 3b 9 10a 20
a 3b 11 a 3b 11



' '
+ +

a 2 a 2
3a b 3 b 3



' '


.
Cu 3.
a) Doac (1 3)(1 3) 1 3 2 0 + < nn phng trnh (1) lun c 2
nghim phn bit.
122
b) V
1 2
x , x
l 2 nghim ca phng trnh (1) nn theo h thc Vi-et, ta c:
1 2
2
x x
1 3
+
+
,
1 2
1 3
x x
1 3

+
.
Do :
1 2
1 2 1 2
x x 1 1 2 2(1 3)
S (1 3)
x x x x 2
1 3
+ +
+ +

.
v P =
2
1 2 1 2
1 1 1 1 3 (1 3) 4 2 3
. (2 3)
x x x x 2 2
1 3
+ + +
+

.
Vy phng trnh bc 2 cn tm l:
2
X (1 3)X (2 3) 0 + + + .
Cu 4.
a) Tam gic ADE cn ti A v
AD = AE. Li c:

1
A =

0 0 0
DAB EAB 90 60 30
Do

0 0 0
1
ADE AED (180 30 ) 75
2

.
b) T gi thit, d thy tam gic BEF
vung cn ti B, nn

0
1
E 45 .
T ta c:
2
1
2
3
x
1
1
M
O
F
E
D
C
A B

0 0 0 0
2 1
DEF DEA E E 75 60 45 180 + + + + suy ra 3 im D, E, F thng
hng, pcm.
c) Ta c:

1 1
B A (cng chn cung EM) suy ra

0
1
B 30 nn

0
2
B 30 .
M

3 2
E B nn

0
3
E 30 .
Vy

0 0 0
2 3
E E 60 30 90 + + hay ME EB. Mt khc BF EB do
ME // BF.
Cu 5. T (1) ta c:
3 2
x 2(y 1) 1 1 x 1 (3)
T (2) ta c:
2 2
2
2y
x 1 x 1 1 x 1
y 1

+
(4)
T (3) v (4), suy ra x = -1, thay vo h cho ta c y = 1.
Vy P = 2.
123
II - LP 10 THPT CHUYN
S 1
Cu 1:
a) t
2
x - t
x

(1), suy ra
2 2
2
4
x t 4
x
+ +
Khi phng trnh cho tr thnh: t
2
4t 5 = 0
t 1
t 5

.
Ln lt thay cc gi tr ca t vo (1) th phng trnh cho c 4 nghim:
x
1
= 1; x
2
= - 2;
3 4
5 33 5 33
x ; x
2 2
+

b) k: x - 2 (1)
t ( )
x + 5 a; x + 2 b a 0; b 0
(2)
Ta c: a
2
b
2
= 3; ( ) ( )
2
x 7x + 10 x + 5 x + 2 ab +
Thay vo phng trnh cho ta c:
(a b)(1 + ab) = a
2
b
2

(a b)(1 a)(1 b) = 0
a - b = 0
1 - a = 0
1 - b = 0

nn
x + 5 x + 2 (VN)
x = - 4
x + 5 1
x = - 1
x + 2 1

i chiu vi (1) suy ra phng trnh cho c nghim duy nht x = - 1.


Cu 2:
124
a) t
3
3
3
3
3
3
1 b
a
x
x a
b
b 1 c
y
c y b
c
1 a
z
a
z c



' '


, khi do abc = 1 nn xyz = 1 (1).
T bi suy ra
1 1 1
x y z
x y z
+ + + +

x + y + z = yz + xz + xy (2).
T (1) v (2) suy ra: xyz + (x + y + z) (xy + yz + zx) 1 = 0

(x 1)(y 1)(z 1) = 0.
Vy tn ti x =1 chng hn, suy ra a = b
3
, pcm.
b) t
3 3
84 84
1 a; 1 b
9 9
+

x = a + b; a
3
+ b
3
= 2; ab =
1
3

.
Ta c: x
3
= (a + b)
3
= a
3
+ b
3
+ 3ab(a + b)
Suy ra: x
3
= 2 x

x
3
+ x 2 = 0 ( )
( )
2
x - 1 x x + 2 0 +

x = 1. V x
2
+ x + 2 =
2
1 7
x + 0
2 4
_
+ >

,
. T suy ra iu phi chng minh.
Cu 3: p dng cc BT:
( )
2 2
a + b 2 a b + ; a + b + c
( )
2 2 2
3 a b c + +
(c suy ra t bt ng thc Bunhiacpski)
Ta c:
( )
( )
( )
( )
( )
( )
( )
2 2
2 2
2 2
1 + x 2x 2 1 x 2x 2 x + 1
1 + y 2y 2 1 y 2y 2 y + 1
1 + z 2z 2 1 z 2z 2 z + 1
x y z 3 x + y + z
+ + +
+ + +
+ + +
+ +
Li c: A =
2 2 2
1 x 1 y 1 z 2x 2y 2z + + + + + + + +
+
( ) ( )
2 2 x y z + +
( )
( )
( )
A 2 x + y + z + 3 2 2 3 x + y + z +
A 6 + 3 2
(do x + y + z 3). Du = xy ra khi v ch khi x = y = z = 1.
Vy maxA =
6 3 2. +
Cu 4:
125
a) Ta c:

0
ABO ACO 90
(tnh cht tip tuyn) (1)
AB = AC
2 2
OA OB
= R = OB = OC (2).
T (1) v (2) suy ra ABOC l hnh vung.
b) Theo bi ra ta c: AD + DE + AE = 2R (3).
Suy ra: DE = BD + CE (4).
V OM DE (M

DE) (5)
Trn tia i ca tia CA ly im F sao
cho CF = BD; suy ra BDO = COF
(c-g-c)

OD = OF; li c DE = FE nn ODE
= OFE (c-c-c)

OM = OC = R
(hai ng cao tng ng) (6). T (5) v
(6) suy ra DE l tip tuyn ca ng
trn (O;R).
c) t: AD = x; AE = y
ADE
1
S xy
2

(x, y > 0)
Ta c: DE
2 2 2 2
AD AE x + y + (nh l Pitago).
V AD + DE + AE = 2R
2 2
x + y + x y + = 2R (6)
p dng BT Csi cho hai s khng m ta c:
2 2
x + y 2 xy v x + y 2xy (7).
Du = xy ra khi v ch khi x = y.
T (6) v (7) suy ra: 2 xy 2xy 2R +
( )
xy 2 2 2R +
( )
2R
xy
2+ 2

xy
2
2R
3 2 2

S
ADE
( )
2
2
ADE
R
S 3 - 2 2 R
3 2 2

+
.
Vy max S
ADE
=
( )
2
3 2 2 R

x = y

ADE cn ti A.
Cu 5: Xt im A v hnh trn (C
1
) c tm A, bn knh bng 1.
126
R
F
M
y
x
E
D
C
B
O
A
C
2
C
1
C
B
A
- Nu tt c 98 im cn li u nm trong (C
1
) th hin nhin bi ton
c chng minh.
- Xt trng hp c im B nm ngoi (C
1
).
Ta c: AB > 1 (1)
V hnh trn (C
2
) tm B, bn knh bng 1.
+ Gi s C l mt im bt k khc A v B. Khi im C thuc mt
trong hai hnh trn
(C
1
) v (C
2
). Tht vy, gi s C khng thuc hai hnh trn ni trn.
Suy ra: AC > 1 v BC > 1 (2)
T (1) v (2) suy ra b 3 im A, B, C khng c hai im no c khong
cch nh hn 1 (v l v tri vi gi thit).
Chng t C (C
1
) hoc C (C
2
). Nh vy 99 im cho u thuc (C
1
)
v (C
2
).
Mt khc 99 = 49.2 + 1 nn theo nguyn tc Dirichle t phi c mt hnh trn
cha khng t hn 50 im.
S 2
Cu 1: a) Theo bi ra ta c:
) 2010 ( 2010 ) 2011 ( 2011 + + x y y x
+ Nu x + y - 2011 = 0 th y - x + 2010 = 0
x y 2010 2x 4021
x y 2011 2y 1



' '
+


x 2010, 5
y 0, 5

'

+ Nu y - x + 2010 = 0 th x + y - 2011 = 0, ta cng c kt qu nh trn.


+ Nu x + y - 2011

0 th
2011
2010
2010
2011
+
+

y x
x y
v l (v VP l s hu t,
VT l s v t)
Vy x = 2010,5 v y = 0,5 l cp s duy nht tho mn bi.
127
b) Ta c xy (z + 1) + y(z + 1) + x(z + 1) + (z + 1) = 2012
<=> (z + 1)(xy + y + x + 1) = 2012
<=> (z + 1)[x(y + 1)+(y + 1)] = 2012
<=> (x + 1)(y + 1)(z + 1) = 1.2.2.503 = 503.4.1 . Ch c 3 b sau tho mn:
x = 502, y = 1, z = 1 hoc x = 1005, y = 1, z = 0 hoc x = 2011, y = 0, z = 0.
Cu 2: a) iu kin: x > -1
t a = 1 + x ; b =
1
2
+ x x
Ta c: 2(a
2
+ b
2
) = 5ab <=> (2a - b)(2b - a) = 0 <=> b = 2a ; a = 2b
Do : 1) 2 1 + x =
1
2
+ x x
<=> 4(x + 1) = x
2
- x + 1
<=> x
2
- 5x - 3 = 0 <=> x
1
=
2
37 5
(loi); x
2
=
2
37 5+
2) 1 + x = 2
1
2
+ x x

2 2
x 1 4(x x 1) 4x 5x 3 0 + + + v
nghim.
Vy phng trnh c 2 nghim: x =
2
37 5+
b) V a, b, c

[0; 2] nn: (2 - a)(2 - b)(2 - c) > 0


<=> 8 - 4(a + b + c) + 2(ab + bc + ca) - abc > 0
<=> 2(ab + bc + ca) > 4(a + b + c) - 8 + abc
nn 2(ab + bc + ca) > 4 (v a + b + c = 3 v abc 0)
Suy ra (a + b + c)
2
- (a
2
+ b
2
+ c
2
) > 4
<=> a
2
+ b
2
+ c
2
5 (v (a + b + c)
2
= 9)
Du = xy ra khi mt trong 3 s a, b, c c mt s bng 2, mt s bng 0 v
mt s bng 1.
Cu 3: Gi s x =
q
p
(p, q

Z, q > 0) v (p, q) = 1
Ta c
2
2
6 n
q
p
q
p
+ +

,
_

(n

N) <=> p
2
= q(-P - 6q + n
2
q)
=> q l c ca p
2
nhng (p, q) = 1 => q = 1 lc x = p
=> p
2
+ p + 6 = n
2
(p, n

Z)
<=> (2p + 1)
2
+ 23 = 4n
2
<=> (2n)
2
- (2p + 1)
2
= 23
<=> (2n - 2p - 1)(2n + 2p + 1) = 23
128
Do 2n - 2p - 1 = 1 v 2n + 2p + 1 = 23 ; 2n - 2p - 1 = 23 v 2n + 2p + 1 =
1
(v 23

P v 2n + 2p + 1 > 0 v 2n - 2p - 1 > 0) <=> p = 5 (t/m) ; p = - 6


(t/m)
Vy s hu t x cn tm l 5 hoc 6
Cu 4:
a) T gic MNKB ni tip c (v

K N +
= 180
0
). T gic MNCI cng ni
tip c (v

MNC MIC
MNC = 90
0
)
=>

BNK BMK
,

INC IMC
(1)
(v 2 gc ni tip cng chn mt cung).
Mt khc

BMK IMC
(2)
(v

BMK KMC KMC IMC + +
do
cng b vi gc A ca tam gic ABC)
T (1), (2) suy ra

BNK
=

INC
nn 3 im
K, N, I thng hng.
P
S
K
N
I
Q
H
O
A
B
C
M
b) V

MAK MCN (v 2 gc ni tipcng chn cung BM)
=>
AK CN AB BK CN
cot g
MK MN MK MN

>
hay
AB BK CN
MK MK MN

(1)
Tng t c:
MN
BN
MI
AI

hay
AC CI BN
MI MI MN
+
(2)
M
IC BK
tg
MI MK

(

=

BMK IMC
) (3)
T (1), (2), (3) =>
AB AC BC
MK MI MN
+
(pcm)
c) Gi giao ca AH, MN vi ng trn (O) th t l Q, S => AQMS l hnh
thang cn (v AQ // MS => AS = QM). V HP // AS (P

MS)
=> HQMP l hnh thang cn, c BN l trc i xng (v Q v H i xng qua
BC)
=> N l trung im ca PM m HP // KN (v KN // AS do

SAC AIN
v
cng bng

NMC
) => KN i qua trung im ca HM (pcm).
129
Cu 5: a v bi ton tm P h phng trnh:
2 2
2 2
2x xy y p
x 2xy 3y 4

'
+ +

c
nghim.
H trn
2 2
2 2
8x 4xy 4y 4p (1)
px 2pxy 3py 4p (2)

'
+ +

. Ly (1) - (2), ta c:
(8 - p)x
2
- 2y(2 + p)x - (4 + 3p)y
2
= 0 (3)
- Nu y = 0 => (8 - p)x
2
= 0 <=> x = 0 hoc p = 8
p 0; p 8.
- Nu y

0 chia 2 v pt (3) cho y


2
ta c :
(8 - p)t
2
- 2(2 + p)t - (4 + 3p) = 0 (4) vi t =
y
x
.
+ Nu p = 8 th t = -
7
5
.
+ Nu p

8: Phng trnh (2) c nghim <=> ' = (2 + p)


2
+ (8 - p)(4 + 3p) > 0
<=> p
2
- 12p - 18 < 0 <=> 6 - 3 6 3 6 6 + p . Du = c xy ra.
Vy min P = 6 - 3 6 , max P = 6 +3 6 .
S 3
Cu 1: a) T gi thit ta c:
( ) ( )
2 2
a b c ab - b - ac + c
= - =
b - c a - c a - b a - b a - c
Nhn 2 v ca ng thc vi
1
b - c
ta c:
( )
( ) ( ) ( )
2 2
2
a ab - b - ac + c
=
a - b a - c b - c
b - c
Vai tr ca a, b, c nh nhau, thc hin hon v vng quanh gia a, b, c ta c:
( )
( ) ( ) ( )
2 2
2
b cb - c - ab + a
=
a - b a - c b - c
c - a
,
( )
( ) ( ) ( )
2 2
2
c ac - a - bc + b
=
a - b a - c b - c
a - b
Cng v vi v cc ng thc trn, ta c
2 2 2
a b c
+ + = 0
(b - c) (c - a) (a - b)

(pcm)
b) t
2 4 4
2010 = x 2010 = x ; 2010 = x . Thay vo ta c:
130
2
2 2
2 4
2
2 1
1 + +
x - x 1 + x
x x
A = + -
1 - x x 1 + x
_

,
=
2
2
2
2
1
1 +
x 1
-
x 1 + x
_

_ ,

,
2 2
1 1
= - = 0
x x
_ _

, ,
Cu 2: a) V a, b, c l di 3 cnh ca tam gic nn a, b, c > 0
p dng BT C-si ta c:
a
2
+ bc

2a
2 2
bc, b + ac 2b ac ; c + ab 2c ab .
Do
2 2 2
1 1 1 1 1 1 1
+ + + +
a + bc b + ac c + ab 2
a bc b ac c ab
_


,
=
a +b b + c c + a
+ +
1 ab + bc + ca 1 a + b + c
2 2 2
. . =
2 abc 2 abc 2abc

,
pcm.
Du bng xy ra khi v ch khi a = b = c, tc l tam gic cho l tam gic u.
b) iu kin x 0; y 0
Ta c: A = (x - 2 xy + y) + 2y - 2 x +1
( ) ( )
2
= [ x - y - 2 x - y + 1] - 2 y + 2y
( )
2
1 1
= x - y - 1 + (2y - 2 y + ) -
2 2
( ) ( )
2 2
1 1 1
= x - y - 1 + 2 y 1 - -
2 2 2

9
x =
x - y - 1 = 0
1
4
A= -
1 2
2 y - 1 = 0
y =
4




' '

Vy minA =
1

2

Cu 3: a) iu kin : 1 x 5
p dng BT Bunhiacpski ta c:
131
o
k
m
f
e
d
c
b
a
( ) ( )
( )
2
2 2
2 x - 1 + 3 5 - x 2 + 3 x - 1 + 5 - x = 13.4
2 x - 1 + 3 5 - x 2 13
Du bng xy ra khi v ch khi 3
29
x - 1 = 2 5 - x x =
13

Thay vo pt cho th li th tha mn..


Vy pt c nghim
29
x =
13
b) Xt ng thc: f(x) + 3f
2
1
= x
x
_

,
x 0 (1)
Thay x = 2 vo (1) ta c: f(2) + 3.
1
f
2
_

,
= 4.
Thay x =
1
2
vo (1) ta c:
1 1
f + 3.f(2) =
2 4
_

,
t f(2) = a,
1
f
2
_

,
= b ta c.
a + 3b = 4
1
3a + b =
4

'

. Gii h, ta c
13
a = -
32
Vy
13
f(2) = -
32
.
Cu 4:
Gi O l tm ca ng trn ngoi tip
lc gic u th A, O, D thng hng v
OK =
1
2
AB. V FM =
1
2
EF m EF = AB
do FM = OK
Ta li c AF = R

AF = OA v

AFM
=
120
0
.

0 0 0
AOK + AOB = 180 = AOK + 60 AOK = 120 .
Do : AFM = AOK (c.g.c)

0
AM = AK, MAK = 60 AMK u.
Cu 5:
132
o
h
d
c
b
a
Gi BH l ng cao ca ABO
Ta c 2S
AOB
= OA . BH
Nhng BH BO nn 2S
AOB
OA . OB
m OA.OB
2 2
OA + OB

2

Do 2S
AOB

2 2
OA + OB

2

Du = xy ra

OA OB v OA = OB
Chng minh tng t ta c:
2S
BOC

2 2
OB + OC

2
; 2S
COD

2 2
OC + OD

2

2S
AOD

2 2
OD + OA

2

Vy 2S = 2(S
AOB
+ S
BOC
+ S
COD
+ S
DOA
)
( )
2 2 2 2
2 OA + OB + OC + OD
2
Hay 2S OA
2
+ OB
2
+ OC
2
+ OD
2
Du bng xy ra khi v ch khi OA = OB = OC = OD
v

0
AOB = BOC = COD = DOA = 90 ABCD
l hnh vung tm O.
Li bnh:
Cu III.b
1) Chc chn bn s hi
1
2
x
t u m ra?
Gi A(x), B(x), P(x), Q(x), C(x) l cc a thc ca bin x v f(x) l
hm s c xc nh bi phng trnh
A(x).f[P(x)] + B(x).f[Q(x)] = C(x) (1)
tnh gi tr ca hm s f(x) ti im x = a ta lm nh sau
Bc 1: Gii phng trnh Q(x) = P(a) . (2)
Gi s x = b l mt nghim ca (2).
B c 2: Thay x = a, x = b vo ph ng trnh (1), v
t x = f(a), y = f(b). ta c h

( ) ( ) ( )
( ) ( ) ( )
A a x B a y C a
B b x A b y C b
+

'
+

(3)
133
Gii h phng trnh (3) ( l h phng trnh bc nht i vi hai
n x, y) .
Trong bi ton trn: A(x) = 1, B(x) = 3, P(x) = x, Q(x) =
1
x
, C(x) = x
2
, a = 2.
Phng trnh Q(x) = P(a)
1
2
x


1
2
x
, tc l
1
2
b
S
1
2
x
c ngh ra nh th .
2) Ch : Khng c n bi t ph ng trnh (2) c bao
nhiu nghi m. Ch c n bi t (c th l on) c m t
nghi m c a n l cho l i gi i thnh cng.
3) Mt s bi tp tng t
a) Tnh gi tr ca hm s f(x) ti x = 1 nu f(x) + 3.f(x) = 2 + 3x.
(vi x ).
b) Tnh gi tr ca hm s f(x) ti x = 3 nu
1
( )
1
f x f x
x
_
+

,

(vi 0 x 1).
c) Tnh gi tr ca hm s f(x) ti x = 2 nu
1 1
( 1) ( )
1
x f x f
x x
_
+

,
(vi 0 x 1).
S 4
Cu 1: a) T x
2
+ y
2
= 4

2xy = (x + y)
2
- 4 = (x + y + 2) (x + y - 2)
V x + y + 2 0 nn
xy x + y
= - 1
x + y + 2 2
(1)
p dng BT Bunhiacopski, ta c:
x + y
( )
2 2
2 x + y

x + y
2 2
(2)
T (1), (2) ta c:
xy
2 - 1
x + y + 2

. Du "="
2 2
x 0, y 0
khi x = y x = y = 2
x + y = 4

'

.
Vy maxA =
2 - 1
.
134
b) V x
2
+ y
2
+ z
2
= 2 nn:
2 2 2 2 2 2 2 2 2
2 2 2 2 2 2 2 2 2 2 2 2
2 2 2 x + y + z x + y + z x + y + z
+ + = + +
x + y y + z z + x x + y y + z z + x
=
2 2 2
2 2 2 2 2 2
z x y
+ + + 3
x + y y + z x + z
Ta c x
2
+ y
2
2xy
2 2
2 2
z z

x + y 2xy

,
Tng t
2 2
2 2
x x

y + z 2yz

,
2 2
2 2
y y

x + z 2xz

Vy
2
2 2
z

x + y
+
2
2 2
x

y + z
+
2
2 2
y
+ 3
x + z
2
z

2xy

+
2
x
2yz
+
2
y
2xz
+ 3


3 3 3
2 2 2 2 2 2
2 2 2 x + y + z
+ + + 3
x + y y + z z + x 2xyz

, pcm.
Cu 2: a) x
2
+ 9x + 20 = 2 3x + 10 (1) .iu kin:
10
x
3

(2)
(1)

(3x + 10 - 2 3x + 10 + 1) + (x
2
+ 6x + 9) = 0

( 3x + 10 - 1)
2
+ (x + 3)
2
= 0
3x + 10 - 1 = 0
x = - 3
x + 3 = 0


'

(tha mn k (2).
Vy phng trnh (1) c nghim x = -3.
b)
2 2 2
2 3
x y - 2x + y = 0
2x - 4x + 3 = - y

'

2
2
3 2
2x
y = (1)
x + 1
y = - 2 (x - 1) - 1

'

Ta c:
2
2
2x
1 y 1 - 1 y 1 (1)
1 + x

Mt khc: - 2 (x - 1)
2
- 1 - 1

y
3
- 1

y - 1 (2)
T (1) v (2)

y = - 1 nn x = 1. Thay vo h cho th li th tha mn.


Vy x = 1 v y = -1 l cc s cn tm.
Cu 3:
a) t
3
x = b > 0 v
3
y = c > 0 ta c x
2
= b
3
v y
2
= c
3

135
h
k
h'
o
m
c
b
a
Thay vo gt ta c
3 2 3 2
b + b c + c + bc = a

a
2
= b
3
+ b
2
c + c
3
+ bc
2
+
( )
2
2 2
2 b c b + c
a
2
= (b + c)
3

3 2
a = b + c
hay
3 2 2 3 2
3
x + y = a , pcm.
b) Gi s x
0
l mt nghim ca phng trnh, d thy
0
x 0
.
Suy ra
2
0
x
+ ax
0
+ b +
2
0 0
a 1
+ = 0
x x
2
0 0
2
0 0
1 1
x + + a x + + b = 0
x x
_


,
t x
0
+
2 2
0 0 0 0
2
0 0
1 1
= y x + = y - 2 , y 2
x x


2
0 0
y - 2 = - ay - b
p dng bt ng thc Bunhiacpxki ta c:
( )
( )
( ) ( )
2
2
2 2 2 2
0 0 0
y - 2 = ay + b a + b y + 1
2 2
2 2
0
2
0
(y 2)
a b
y 1

+
+
(1)
Ta chng minh
2 2
0
2
0
(y 2) 4
y 1 5

+
(2)
Thc vy: (2)
4 2 2 4 2
0 0 0 0 0
5(y 4y 4) 4(y 1) 5y 24y 16 0 + + +
2 2
0 0
4
5(y 4)(y ) 0
5

ng vi
y 2
nn (1) ng
T (1), (2) suy ra
2 2 2 2
4
a + b 5(a + b ) 4
5

, pcm.
Cu 4: t AH = x
Ta c

0
AMB = 90 (OA = OB = OM)
Trong vung AMB ta c MA
2
= AH . AB = 2Rx
(H l chn ng vung gc h t M xung BC)
Mt khc: MK
2
= OH
2
= (R - x)
2
(v MKOH l
hnh ch nht).
Theo bi ra ta c: 4Rx = 15(R - x)
2
.
Do H

AB

O x 2R
Phng trnh tr thnh: 15x
2
- 34Rx + 15R
2
= 0

(5x - 3R) (3x - 5R) = 0


3R 5R
x = ; x =
5 3

.
C 2 gi tr ny u tho mn
136
i
f
g
e
d
c
b
a
Vy ta tm c 2 im H v H

2 im M v M l giao im ca na
ng trn vi cc ng vung gc vi AB dng t H v H.
Cu 5:
Gi I l trung im ca CD.
Ni EF, EI, IF, ta c IE l ng
trung bnh ca BDC

IE // BC
M GF BC

IE GF (1)
Chng minh tng t EG IF (2)
T (1) v (2)

G l trc tm ca EIF

IG EF (3)
D chng minh EF // DC (4)
T (3) v (4)

IG DC
Vy DGC cn ti G

DG = GC
S 5
Cu 1: 1) Tr vo 2 v ca phng trnh vi 2x .
9x
x + 9
Ta c:
2
2
9x 18x
x - = 40 -
x + 9 x + 9
_

,

2
2 2
x 18x
+ - 40 = 0
x + 9 x + 9
_


,
(1)
t
2
x
x + 9
= y (2), phng trnh (1) tr thnh y
2
+ 18y - 40 = 0
(y + 20) (y - 2) = 0 y = -20 ; y = 2
Thay vo (2), ta c
2 2
2 2
x = - 20(x + 9) x +20x +180=0 (3)
x = 2(x + 9) = 0 x - 2x - 18 = 0 (4)




Phng trnh (3) v nghim, phng trnh (4) c 2 nghim l: x 1 19. t
Vy phng trnh cho c 2 nghim l: x 1 19. t
2) . iu kin
x > 3
x + 1
0
x - 1 x - 3


(*)
Phng trnh cho

x + 1
(x - 3) (x + 1) + 3(x - 3) = 4
x - 3
t t =
( )
2
x + 1
x - 3 t = (x - 3) (x + 1)
x - 3

137
f
k
i
e
o
h
n
m
b a
Phng trnh tr thnh: t
2
+ 3t - 4 = 0
t = 1; t = - 4
Ta c: (x -3)
1 1
1 (1) ; ( 3) 4 (2)
- 3 3
+ +

x x
x
x x
+ (1)
2
x 3
x 3
x 1 5
(x 3)(x 1) 1
x 2x 4 0
>
>

+
' '
+


. (t/m (*))
+ (2)
2
x 3
x 3
x 1 2 5
(x 3)(x 1) 16
x 2x 19 0
<
<


' '
+


. (t/m (*))
Vy phng trnh cho c 2 nghim l: x 1 5 ; x 1 2 5 + .
Cu 2: 1) iu kin: 1 - x
2
> 0

- 1 < x < 1

2 - 3x > 0

A 0
Vy A
2
=
2 2
2 2
25 - 30x + 9x (3 - 5x)
= +16 16
1 - x 1 - x
.
Du bng xy ra khi 3 - 5x = 0
3
x =
5

Vy minA = 4.
2) Chng minh:
2 2 2 2 2 2
a + b + b + c + c + a 2 (a + b + c)
(1)
S dng bt ng thc:
2 2 2
2(x y ) (x y) + + , ta c:
2 2 2 2 2
2(a + b ) (a b) 2. a + b a + b +
(2)
Tng t, ta c:
2 2
2. b + c b + c
(3) v

2 2
2. c + a c + a
(4)
Ly (2) + (3) + (4) theo tng v v rt gn, suy ra (1) ng, pcm.
Cu 3: (1) c nghim
2
y
x 4 0 x 2; x 2 (3)
(2)
2 2
(y 1) x 2x + c nghim
2
x 2x 0 2 x 0 (4)
T (3), (4) ta c: x = - 2, t ta c y = - 1. Vy h c nghim (- 2 ; - 1).
Cu 4: K MP // BD (P

AD)
MD ct AC ti K. Ni NP ct BD ti H.
Ta c
AM AP
=
AB AD
m
AM CM
= (gt)
AB CD
AP CN
= PN // AC
AD CD

Gi O l giao im
138
ca AC v BD. Ta c
BO CO MK OC
= , =
OD OA PK OA
v
NH OC
=
PH OA
. Suy ra:
NH MK
= KH // MN
PH PK

Cc t gic KENH, MFHK l hnh bnh hnh nn MF = KH v EN = KH


MF = EN

ME = NF
Cu 5: 1) T gic MEHF ni tipv

0
MEH + MFH = 180

0
AMB = 180 - EHF = EHA + FHB (1)
Ta c

MHF = MEF
(gc ni tip chn

MF
)
Li c

0
MHF + FHB = 90 = MEF + EMD

FHB = EMD (2)
T (1) v (2)

EHA = DMB
, Gi N l giao im ca MD vi ng trn (O)
ta c

DMB = NAB
(gc ni tip chn

NB
)

EHA = NAB
do AN // EH
m HE MA nn NA MA. hay

0
MAN = 90
AN l ng knh ca
ng trn. Vy MD i qua O c nh.
2) K DI MA, DK MB, ta c
MAD MAD
MBD MBH
S S AH AM . HE AD AM . DI
= = ; = =
BD S BM . DK BH S BM . HF
Vy
2
2
AH AD MA HE . DI
. = .
BD BH MB DK . HF
(1)
Ta c

HMB = FHB
(cng ph vi

MHF
) m

FHB = EMD
(CMT)

EFH = DIK
v

EHF = DMH
.
T gic MEHF ni tip nn

0
AMH = EFH EHF = 180 - AMB v
T gic MIDK ni tip nn

0
DMB = DIK IDK = 180 - AMB v

EFH = DIK EHF = IDK v
DIK HFE (g.g)
do
ID DK
suy ra =
HF HE

ID . HE = DK . HF
HE.DI
= 1
DK.HF

(2)
T (1), (2)
2
2
MA AH AD
= .
MB BD BH
.
S 6
139
Cu 1: Ta c: A =
1 - 2 2 - 3 24 - 25
+ + ... +
- 1 - 1 - 1
= - 1 + 2 - 2 + 3 - 3 + ... + 25 = - 1 + 5 = 4
Cu 2: a) T gi thit suy ra:
2 2 2 2 2 2
2 2 2 2 2 2 2 2 2 2 2 2
x x y y z z
- + - + - = 0
a a + b + c b a + b + c c a + b + c
_ _ _

, , ,
2 2 2
2 2 2 2 2 2 2 2 2 2 2 2
1 1 1 1 1 1
x - + y - + z - = 0
a a + b + c b a + b + c c a + b + c
_ _ _


, , ,
(*)
Do
2 2 2 2 2 2 2 2 2 2 2 2
1 1 1 1 1 1
- > 0; - > 0; - > 0
a a + b + c b a + b + c c a + b + c

Nn t (*) suy ra x = y = z = 0, do M = 0
b) x
3
= 2a +
3
2
2
a + 1 8a - 1
3 . a -
3 3
_ _

, ,
x

x
3
= 2a + 3x .
( )
3
3
1 - 2a

3

x
3
= 2a + x(1 - 2a)

x
3
+ (2a - 1) x - 2a = 0

(x - 1) (x
2
+ x + 2a) = 0
2
x - 1 = 0
x 1
1
x + x + 2a = 0 ( a > )
8
nn x l nguyn

v nghim do
mt s dung
Cu 3:
a) Ta c:
( ) ( )
4c 1 35 35
+ 2. > 0
4c + 57 1 + a 35 2b 1 + a 2b + 35

+
(1)
Mt khc
1 4c 35 1 4c 35
- -
1 + a 4c + 57 35 + 2b 1 + a 4c + 57 35 + 2b

1 4c 35 2b
- + 1 1 - =
1 +a 4c + 57 35 + 2b 35 + 2b

( ) ( )
2b 1 57 57
+ 2.
35 + 2b 1 + a 4c + 57 1 + a 4c + 57

> 0 (2)
140
Q
P
N
M H C
B
A
Ta c:
1 4c 35
1 - 1 - +
1 + a 4c + 57 35 + 2b

( ) ( )
a 57 35 35 . 57
+ 2.
1 + a 4c + 57 35 + 2b 4c + 57 35 + 2b

> 0 (3)
T (1), (2), (3) ta c:
( ) ( ) ( ) ( ) ( ) ( )
8abc 35 . 57
8 .
1 + a 4c + 57 2b + 35 1 + a 2b + 35 4c + 57

Do abc 35.57 = 1995.


Du = xy ra khi v ch khi a = 2, b = 35 v c =
57
2
.
Vy min (abc) = 1995.
b) t t =
A B C D
= = =
a b c d

A = ta, B = tb, C = tc, D = td.
t =
A + B + C + D
a + b + c + d
V vy
2 2 2 2
aA + bB + cC + dD = a t + b t + c t + d t
= (a + b + c + d)
A + B + C + D
t = (a + b + c + d)
a + b + c + d
= (a + b + c +d)(A + B + C + D)
Cu 4:
a) Xt ABC c PQ // BC
AQ QP
=
AB BC

Xt BAH c QM // AH
BQ QM
=
BA AH

Cng tng v ta c:
AQ BQ QP QM QP QM
+ = + 1 = +
AB AB BC AH BC AH

141
H
M
D
C
B
A
2
MNPQ
ABC
ABC
MNPQ
2S
QP QM QP QM
1 = + 4 . =
BC AH BC AH S
S
S .
2
_


,

ABC
MNPQ
S QP QM 1 BC
maxS = khi = = QP =
2 BC AH 2 2

Tc l khi PQ l ng trung bnh ca ABC, khi PQ i qua trung im


AH.
b) V
QP QM
1 = +
BC AH
m BC = AH
QP + QM
1 = QP + QM = BC
BC

Do chu vi (MNPQ) = 2BC (khng i)
Cu 5:
HCD ng dng vi ABM (g.g) m
AB = 2AM nn HC = 2HD.
t HD = x th HC = 2x. Ta c:
DH
2
= HM . HC hay x
2
= HM . 2x

HM = 0,5x; MC = 2,5x; AM = 2,5x; AH = 3x.


Vy AH = 3HD.
MC LC
Trang
- Li gii thiu ___________________________________________3
- A phn ti __________________________________________5
142
I Phn n thi tuyn sinh lp 10 THPT_______________________ 5
II n thi tuyn sinh lp 10 chuyn ton ___________________33
B- Phn li gii ________________________________________38
I Lp 10 THPT _______________________________________38
II Lp 10 chuyn ton_________________________________ 122
143

You might also like